Сходящаяся числовая последовательность: Предел числовой последовательности, сходящиеся и расходящиеся последовательности

Содержание

Предел числовой последовательности, сходящиеся и расходящиеся последовательности

Содержание:

Определение

Число $a$ называется пределом последовательности $\left\{x_{n}\right\}$ и обозначается $\lim _{n \rightarrow \infty} x_{n}=\lim _{n} x_{n}=a$, $x_{n} \underset{n \rightarrow \infty}{\longrightarrow} a$

Число $a$ называется пределом последовательности $\left\{x_{n}\right\}$ , если для любого $\epsilon>0$ существует номер $n_{0}=n_{0}(\epsilon)$ такой, что для любого $n>n_{0}$ выполняется неравенство $\left|x_{n}-a\right| \lt \epsilon$ :

$\lim _{n \rightarrow \infty} x_{n}=a \Leftrightarrow \forall \epsilon>0, \exists n_{0}=n_{0}(\epsilon) : \forall n>n_{0},\left|x_{n}-a\right| \lt \epsilon$

Определение

Целой частью $[x]$ некоторого числа $x$ называется наибольшее целое число, не превосходящее $x$

Пример

Задание. Найти целую часть чисел — 2,36; 2,36; 2.

Решение. $[-2,36]=-3,[2,36]=2,[2]=2$

Слишком сложно?

Предел числовой последовательности не по зубам? Тебе ответит эксперт через 10 минут!

Пример

Задание. Доказать равенство: $\lim _{n \rightarrow \infty} \frac{1}{n}=0$

Доказательство. Исходя из определения, 0 будет пределом последовательности $\frac{1}{n}$ , если для любого $\epsilon>0$ найдется такой номер $n_{0}=n_{0}(\epsilon)$, что для любого $n>n_{0}$ выполняется неравенство $\left|x_{n}-0\right| \lt \epsilon$:

$\left|x_{n}-a\right|=\left|\frac{1}{n}-0\right|=\left|\frac{1}{n}\right|=\frac{|1|}{|n|}=\frac{1}{n} \lt \epsilon \Rightarrow n>\frac{1}{\epsilon}$

В качестве $n_{0}$ возьмем $n_{0}=\left[\frac{1}{\epsilon}\right]+1$

Итак, для любого $n>n_{0}$ указано соответствующее значение $n_{0}$ , а тогда равенство $\lim _{n \rightarrow \infty} \frac{1}{n}=0$ доказано.{n+1}$ не имеет предел.

Доказательство. Пусть $a$ — предел рассматриваемой последовательности, то есть $\lim _{n \rightarrow \infty} x_{n}=a$. Рассмотрим $\epsilon=\frac{1}{10} \Rightarrow \exists n_{0}=n_{0}(\epsilon) \in N : n>n_{0} :\left|x_{n}-a\right| \lt \epsilon$

Пусть $n=2 k$ :

$\left|x_{2 k}-a\right| \lt \frac{1}{10} \Rightarrow|-1-a| \lt \frac{1}{10} \Rightarrow|1+a| \lt \frac{1}{10}$

Пусть $n=2 k+1$ :

$\left|x_{2 k+1}-a\right| \lt \frac{1}{10} \Rightarrow|1-a| \lt \frac{1}{10}$

Так как полученные выражения не равны, то данная последовательность предела не имеет.

Постоянная последовательность $\left\{x_{n}\right\}=\{c\}$ имеет предел, равный числу $c$ : $\lim _{n \rightarrow \infty} x_{n}=\lim _{n \rightarrow \infty} c=c$

Теорема

Сходящаяся последовательность имеет только один предел.

Последовательность на бесконечности

Последовательность $\left\{x_{n}\right\}$ имеет бесконечный предел, если для любого $\epsilon>0, \exists n_{0} \in N : n>n_{0} :$ $x_{n}>\epsilon : \lim _{n \rightarrow \infty} x_{n}=\infty$

Последовательность $\left\{x_{n}\right\}$ называется бесконечно малой, если $\lim _{n \rightarrow \infty} x_{n}=0$

Последовательность $\left\{x_{n}\right\}$ называется бесконечно большой, если для любого $\epsilon>0$ существует номер $n_{0}$ такое, что для любого $n>n_{0} :\left|x_{n}\right|>\epsilon$

Теорема

Пусть $\lim _{n \rightarrow \infty} x_{n}=a, \lim _{n \rightarrow \infty} y_{n}=b$ , тогда

а) $\lim _{n \rightarrow \infty}\left(x_{n}+y_{n}\right)=\lim _{n \rightarrow \infty} x_{n}+\lim _{n \rightarrow \infty} y_{n}=a+b$ ;

б) $\lim _{n \rightarrow \infty}\left(x_{n} \cdot y_{n}\right)=\lim _{n \rightarrow \infty} x_{n} \cdot \lim _{n \rightarrow \infty} y_{n}=a \cdot b$ ;

в) если $b \neq 0$ , то начиная с некоторого номера заданная последовательность $\lim _{n \rightarrow \infty} \frac{x_{n}}{y_{n}}=\frac{a}{b}$

Читать дальше: предельный переход в неравенствах.

Числовая последовательность — это… Что такое Числовая последовательность?

Последовательность

Числовая последовательность — это последовательность элементов числового пространства.

Числовые последовательности являются одним из основных объектов рассмотрения в математическом анализе.

Определение

Пусть множество — это либо множество вещественных чисел , либо множество комплексных чисел . Тогда последовательность элементов множества называется

числовой последовательностью.

Примеры

  • Функция является бесконечной последовательностью целых чисел. Начальные отрезки этой последовательности имеют вид .
  • Функция является бесконечной последовательностью рациональных чисел. Начальные отрезки этой последовательности имеют вид .
  • Функция, сопоставляющая каждому натуральному числу одно из слов «январь», «февраль», «март», «апрель», «май», «июнь», «июль», «август», «сентябрь», «октябрь», «ноябрь», «декабрь» (в порядке их следования здесь) представляет собой последовательность вида . В частности, пятым членом этой последовательности является слово «май».

Операции над последовательностями

На множестве всех последовательностей элементов множества можно определить арифметические и другие операции, если таковые определены на множестве . Такие операции обычно определяют естественным образом, т. е. поэлементно.


Например, так определяются арифметические операции для числовых последовательностей.

Суммой числовых последовательностей и называется числовая последовательность такая, что .

Разностью числовых последовательностей и называется числовая последовательность такая, что .

Произведением числовых последовательностей и называется числовая последовательность такая, что .

Частным числовой последовательности и числовой последовательности , все элементы которой отличны от нуля, называется числовая последовательность . Если в последовательности на позиции всё же имеется нулевой элемент, то результат деления на такую последовательность всё равно может быть определён, как последовательность .

Конечно, арифметические операции могут быть определены не только на множестве числовых последовательностей, но и на любых множествах последовательностей элементов множеств, на которых определены арифметические операции, будь то поля или даже кольца.

Подпоследовательности

Подпоследовательность последовательности — это последовательность , где — возрастающая последовательность элементов множества натуральных чисел.

Иными словами, подпоследовательность получается из последовательности удалением конечного или счётного числа элементов.

Примеры

  • Последовательность простых чисел является подпоследовательностью последовательности натуральных чисел.
  • Последовательность натуральных чисел, кратных 12, является подпоследовательностью последовательности чётных натуральных чисел.

Свойства

  • Всякая последовательность является своей подпоследовательностью.
  • Для всякой подпоследовательности верно, что .
  • Подпоследовательность сходящейся последовательности сходится к тому же пределу, что и исходная последовательность.
  • Если все подпоследовательности некоторой исходной последовательности сходятся, то их пределы равны.
  • Любая подпоследовательность бесконечно большой последовательности также является бесконечно большой.
  • Из любой неограниченной числовой последовательности можно выделить бесконечно большую подпоследовательность, все элементы которой имеют определённый знак.
  • Из любой числовой последовательности можно выделить либо сходящуюся подпоследовательность, либо бесконечно большую подпоследовательность, все элементы которой имеют определённый знак.

Предельная точка последовательности

Предельная точка последовательности

— это точка, в любой окрестности которой содержится бесконечно много элементов этой последовательности. Для сходящихся числовых последовательностей предельная точка совпадает с пределом.

Предел последовательности

Предел последовательности — это объект, к которому члены последовательности приближаются с ростом номера. Так в произвольном топологическом пространстве пределом последовательности называется элемент, в любой окрестности которого лежат все члены последовательности, начиная с некоторого. В частности для числовых последовательностей предел — это число, в любой окрестности которого лежат все члены последовательности начиная с некоторого.

Частичный предел последовательности

— это предел одной из её подпоследовательностей. У сходящихся числовых последовательностей он всегда совпадает с обычным пределом.

Верхний предел последовательности — это наибольшая предельная точка этой последовательности.

Нижний предел последовательности — это наименьшая предельная точка этой последовательности.

Некоторые виды последовательностей

  • Стационарная последовательность — это последовательность, все члены которой, начиная с некоторого, равны.
    стационарная

Ограниченные и неограниченные последовательности

В предположении о линейной упорядоченности множества элементов последовательности можно ввести понятия ограниченных и неограниченных последовательностей.

Критерий ограниченности числовой последовательности

Числовая последовательность является ограниченной тогда и только тогда, когда существует такое число, что модули всех членов последовательности не превышают его.

ограниченная
Свойства ограниченных последовательностей

Бесконечно большие и бесконечно малые последовательности

Свойства бесконечно малых последовательностей

Бесконечно малые последовательности отличаются целым рядом замечательных свойств, которые активно используются в математическом анализе, а также в смежных с ним и более общих дисциплинах.

  • Сумма двух бесконечно малых последовательностей сама также является бесконечно малой последовательностью.
  • Разность двух бесконечно малых последовательностей сама также является бесконечно малой последовательностью.
  • Алгебраическая сумма любого конечного числа бесконечно малых последовательностей сама также является бесконечно малой последовательностью.
  • Произведение ограниченной последовательности на бесконечно малую последовательность есть бесконечно малая последовательность.
  • Произведение любого конечного числа бесконечно малых последовательностей есть бесконечно малая последовательность.
  • Любая бесконечно малая последовательность ограничена.
  • Если стационарная последовательность является бесконечно малой, то все её элементы, начиная с некоторого, равны нулю.
  • Если вся бесконечно малая последовательность состоит из одинаковых элементов, то эти элементы — нули.
  • Если — бесконечно большая последовательность, не содержащая нулевых членов, то существует последовательность , которая является бесконечно малой. Если же всё же содержит нулевые элементы, то последовательность всё равно может быть определена, начиная с некоторого номера , и всё равно будет бесконечно малой.
  • Если — бесконечно малая последовательность, не содержащая нулевых членов, то существует последовательность , которая является бесконечно большой. Если же всё же содержит нулевые элементы, то последовательность всё равно может быть определена, начиная с некоторого номера , и всё равно будет бесконечно большой.

Сходящиеся и расходящиеся последовательности

  • Сходящаяся последовательность — это последовательность элементов множества , имеющая предел в этом множестве.
  • Расходящаяся последовательность — это последовательность, не являющаяся сходящейся.
Свойства сходящихся последовательностей
  • Всякая бесконечно малая последовательность является сходящейся. Её предел равен нулю.
  • Удаление любого конечного числа элементов из бесконечной последовательности не влияет ни на сходимость, ни на предел этой последовательности.
  • Любая сходящаяся последовательность элементов хаусдорфова пространства имеет только один предел.
  • Любая сходящаяся последовательность ограничена. Однако не любая ограниченная последовательность сходится.
  • Последовательность сходится тогда и только тогда, когда она является ограниченной и при этом её верхний и нижний пределы совпадают.
  • Если последовательность сходится, но не является бесконечно малой, то, начиная с некоторого номера, определена последовательность , которая является ограниченной.
  • Сумма сходящихся последовательностей также является сходящейся последовательностью.
  • Разность сходящихся последовательностей также является сходящейся последовательностью.
  • Произведение сходящихся последовательностей также является сходящейся последовательностью.
  • Частное двух сходящихся последовательностей определено, начиная с некоторого элемента, если только вторая последовательность не является бесконечно малой. Если частное двух сходящихся последовательностей определено, то оно представляет собой сходящуюся последовательность.
  • Если сходящаяся последовательность ограничена снизу, то никакая из её нижних граней не превышает её предела.
  • Если сходящаяся последовательность ограничена сверху, то её предел не превышает ни одной из её верхних граней.
  • Если для любого номера члены одной сходящейся последовательности не превышают членов другой сходящейся последовательности, то и предел первой последовательности также не превышает предела второй.
  • Если все элементы некоторой последовательности, начиная с некоторого номера, лежат на отрезке между соответствующими элементами двух других сходящихся к одному и тому же пределу последовательностей, то и эта последовательность также сходится к такому же пределу.
  • Любую сходящуюся последовательность можно представить в виде , где — предел последовательности , а — некоторая бесконечно малая последовательность.
  • Всякая сходящаяся последовательность является фундаментальной. При этом фундаментальная числовая последовательность всегда сходится (как и любая фундаментальная последовательность элементов полного пространства).

Монотонные последовательности

Монотонная последовательность — это невозрастающая, либо неубывающая последовательность. При этом предполагается, что на множестве, из которого берутся элементы последовательности, введено отношение порядка.

Фундаментальные последовательности

Фундаментальная последовательность (сходящаяся в себе последовательность, последовательность Коши) — это последовательность элементов метрического пространства, в которой для любого наперёд заданного расстояния найдётся такой элемент, расстояние от которого до любого из следующих за ним элементов не превышает заданного. Для числовых последовательностей понятия фундаментальной и сходящейся последовательностей эквивалентны, однако в общем случае это не так.

Вариации и обобщения

Примечания

См. также

Страница не найдена — ПриМат

© 2012-2016: Нохум-Даниэль Блиндер (11), Анастасия Лозинская (10), Денис Стехун (8), Валентин Малявко (8), Елизавета Савицкая (8), Игорь Любинский (8), Юлия Стерлянко (8), Александр Базан (7), Анна Чалапчий (7), Константин Берков (7), Олег Шпинарев (7), Максим Швандт (6), Людмила Рыбальченко (6), Кирилл Волков (6), Татьяна Корнилова (6), Влад Радзивил (6), Елизавета Снежинская (5), Вадим Покровский (5), Даниил Радковский (5), Влад Недомовный (5), Александр Онищенко (5), Андрей Метасов (5), Денис Базанов (5), Александр Ковальский (5), Александр Земсков (5), Марина Чайковская (5), Екатерина Шибаева (5), Мария Корень (5), Анна Семененко (5), Мария Илларионова (5), Сергей Черкес (5), Алиса Ворохта (5), Валерия Заверюха (5), Никита Савко (4), Кондрат Воронов (4), Алина Зозуля (4), Иван Чеповский (4), Артем Рогулин (4), Игорь Чернега (4), Даниил Кубаренко (4), Ольга Денисова (4), Татьяна Осипенко (4), Яков Юсипенко (4), Ольга Слободянюк (4), Руслан Авсенин (4), Екатерина Фесенко (4), Дмитрий Заславский (4), Алина Малыхина (4), Андрей Лисовой (4), Полина Сорокина (4), Кирилл Демиденко (4), Дмитрий Стеценко (4), Александр Рапчинский (4), Святослав Волков (4), Иван Мясоедов (4), Владислав Стасюк (4), Алёна Гирняк (4), Николай Царев (4), Валентин Цушко (4), Павел Жуков (4), Роман Бронфен-Бова (4), Артём Романча (4), Анна Шохина (4), Иван Киреев (4), Виктор Булгаков (3), Дмитрий Мороз (3), Богдан Павлов (3), Игорь Вустянюк (3), Андрей Яроцкий (3), Лаура Казарян (3), Екатерина Мальчик (3), Анатолий Осецимский (3), Иван Дуков (3), Дмитрий Робакидзе (3), Вячеслав Зелинский (3), Данила Савчак (3), Дмитрий Воротов (3), Стефания Амамджян (3), Валерия Сиренко (3), Георгий Мартынюк (3), Виктор Иванов (3), Вячеслав Иванов (3), Валерия Ларикова (3), Евгений Радчин (3), Андрей Бойко (3), Милан Карагяур (3), Александр Димитриев (3), Иван Василевский (3), Руслан Масальский (3), Даниил Кулык (3), Стас Коциевский (3), Елизавета Севастьянова (3), Павел Бакалин (3), Антон Локтев (3), Андрей-Святозар Чернецкий (3), Николь Метри (3), Евелина Алексютенко (3), Константин Грешилов (3), Марина Кривошеева (3), Денис Куленюк (3), Константин Мысов (3), Мария Карьева (3), Константин Григорян (3), Колаев Демьян (3), Станислав Бондаренко (3), Ильдар Сабиров (3), Владимир Дроздин (3), Кирилл Сплошнов (3), Карина Миловская (3), Дмитрий Козачков (3), Мария Жаркая (3), Алёна Янишевская (3), Александра Рябова (3), Дмитрий Байков (3), Павел Загинайло (3), Томас Пасенченко (3), Виктория Крачилова (3), Таисия Ткачева (3), Владислав Бебик (3), Илья Бровко (3), Максим Носов (3), Филип Марченко (3), Катя Романцова (3), Илья Черноморец (3), Евгений Фищук (3), Анна Цивинская (3), Михаил Бутник (3), Станислав Чмиленко (3), Катя Писова (3), Дмитрий Дудник (3), Дарья Кваша (3), Игорь Стеблинский (3), Артем Чернобровкин (3), Яна Колчинская (2), Юрий Олейник (2), Кирилл Бондаренко (2), Елена Шихова (2), Татьяна Таран (2), Наталья Федина (2), Настя Кондратюк (2), Никита Гербали (2), Сергей Запорожченко (2), Николай Козиний (2), Георгий Луценко (2), Владислав Гринькив (2), Александр Дяченко (2), Анна Неделева (2), Никита Строгуш (2), Настя Панько (2), Кирилл Веремьев (2), Даниил Мозгунов (2), Андрей Зиновьев (2), Андрей Данилов (2), Даниил Крутоголов (2), Наталия Писаревская (2), Дэвид Ли (2), Александр Коломеец (2), Александра Филистович (2), Евгений Рудницкий (2), Олег Сторожев (2), Евгения Максимова (2), Алексей Пожиленков (2), Юрий Молоканов (2), Даниил Кадочников (2), Александр Колаев (2), Александр Гутовский (2), Павел Мацалышенко (2), Таня Спичак (2), Радомир Сиденко (2), Владислав Шиманский (2), Илья Балицкий (2), Алина Гончарова (2), Владислав Шеванов (2), Андрей Сидоренко (2), Александр Мога (2), Юлия Стоева (2), Александр Розин (2), Надежда Кибакова (2), Майк Евгеньев (2), Евгений Колодин (2), Денис Карташов (2), Александр Довгань (2), Нина Хоробрых (2), Роман Гайдей (2), Антон Джашимов (2), Никита Репнин (2), Инна Литвиненко (2), Яна Юрковская (2), Гасан Мурадов (2), Богдан Подгорный (2), Алексей Никифоров (2), Настя Филипчук (2), Гук Алина (2), Михаил Абабин (2), Дмитрий Калинин (2), Бриткариу Ирина (2), Никита Шпилевский (2), Алексей Белоченко (2), Юлиана Боурош (2), Никита Семерня (2),

Сходящиеся последовательности | Рефераты KM.RU

Последовательность, у которой существует предел, называется сходящейся. Последовательность не являющаяся сходящейся называется расходящейся.

Определение: Последовательность {xn} называется сходящейся, если существует такое число а, что последовательность {xn-а} является бесконечно малой. При этом число а называется пределом последовательности {xn}.

В соответствии с этим определением всякая бесконечно малая последовательность является сходящейся и имеет своим пределом число ноль.

Можно, также, дать еще одно определение сходящейся последовательности: Последовательность {xn} называется сходящейся, если существует такое число а, что для любого положительного числа e можно указать номер N такой, что при n³ N все элементы xn этой последовательности удовлетворяют неравенству:

|xn-a|<e .


При этом число а называется пределом последовательности.

Некоторые свойства сходящихся последовательностей:

ТЕОРЕМА: Сходящаяся последовательность имеет только один предел.

Доказательство: Пусть a и b – пределы сходящейся последовательности {xn}. Тогда, используя специальное представление для элементов xn сходящейся последовательности {xn}, получим xn=а+a n, xn=b+b n, где a n и b n – элементы бесконечно малых последовательностей {a n} и {b n}.

Вычитая данные соотношения, найдем a n-b n=b-a. Так как все элементы бесконечно малой последовательности {a n-b n} имеют одно и то же постоянное значение b-a, то (по теореме: Если все элементы бесконечно малой последовательности {a n} равны одному и тому же числу с, то с=0) b-a=0, т.е. b=a. Теорема доказана.

ТЕОРЕМА: Сходящаяся последовательность ограничена.

Доказательство: Пусть {xn} — сходящаяся последовательность и а – ее предел. Представим ее в следующем виде:

xn=а+a n,


где a n— элемент бесконечно малой последовательности. Так как бесконечно малая последовательность {a n} ограничена (по теореме: Бесконечно малая последовательность ограничена.), то найдется такое число А, что для всех номеров n справедливо неравенство |a n|£ А. Поэтому | xn | £ |a| + A для всех номеров n, что и означает ограниченность последовательности {xn}. Теорема доказана.

Ограниченная последовательность может и не быть сходящейся. Например, последовательность 1, -1, 1, -1, … — ограничена , но не является сходящейся. В самом деле, если бы эта последовательность сходилась к некоторому числу а, то каждая из последовательностей {xn-a} и {xn+1-a} являлась бы бесконечно малой. Но тогда (по теореме: Разность бесконечно малых последовательностей есть бесконечно малая последовательность.) {(xn-a) – (xn+1-a)}={xn– xn+1} была бы бесконечно малой, что невозможно т.к. |xn– xn+1| = 2 для любого номера n.

ТЕОРЕМА: Сумма сходящихся последовательностей {хn} и {yn} есть сходящаяся последовательность, предел которой равен сумме пределов последовательностей {хn} и {yn}.

Доказательство: Пусть а и b – соответственно пределы последовательностей {хn} и {yn}. Тогда:

xn=а+a n, yn=b+b n,


где {a n} и {b n) – бесконечно малые последовательности. Следовательно, (хn + yn) — (а + b) =a n+b n.

Таким образом, последовательность {(хn + yn) — (а + b)} бесконечно малая, и поэтому последователдьность {хn + yn} сходится и имеет своим пределом число а+b. Теорема доказана.

ТЕОРЕМА: Разность сходящихся последовательностей {хn} и {yn} есть сходящаяся последовательность, предел которой равен разности пределов последовательностей {хn} и {yn}.


Доказательство: Пусть а и b – соответственно пределы последовательностей {хn} и {yn}.Тогда:

xn=а+a n, yn=b+b n,


где {a n} и {b n) – бесконечно малые последовательности. Следовательно, (хn — yn) — (а — b) =a n-b n.

Таким образом, последовательность {(хn — yn) — (а — b)} бесконечно малая, и поэтому последователдьность {хn — yn} сходится и имеет своим пределом число а-b. Теорема доказана.

ТЕОРЕМА: Произведение сходящихся последовательностей {хn} и {yn} есть сходящаяся последовательность, предел которой равен произведению пределов последовательностей {хn} и {yn}.

Доказательство: Пусть а и b – соответственно пределы последовательностей {хn} и {yn}, то xn=а+a n, yn=b+b n и xyn=a× b+a× b n+b× a n+a n× b n. Следовательно,

xyn-а× b=a× b n+b× a n+a n× b n.


(в силу теоремы: Произведение ограниченной последовательности на бесконечно малую есть бесконечно малая последовательность.) последовательность {a× b n+b× a n+a n× b n} бесконечно малая, и поэтому последовательность {xyn-а× b} тоже бесконечно малая, а значит последовательность {xyn} сходится и имеет своим пределом число а× b. Теорема доказана.

ЛЕММА: Если последовательность {yn} сходится и имеет отличный от ноля предел b, то, начиная с некоторого номера, определена последовательность , которая является ограниченной.

Доказательство: Пусть . Так как b¹ 0, то e >0. Пусть N – номер, соответствующий этому e , начиная с которого выполняется неравенство:

|yn-b|<e или |yn-b|<


из этого неравенства следует, что при n³ N выполняется неравенство |yn|>. Поэтому при n³ N имеем . Следовательно, начиная с этого номера N, мы можем рассматривать последовательность , и эта последовательность ограничена. Лемма доказана.

ТЕОРЕМА: Частное двух сходящихся последовательностей {xn} и {yn} при условии, что предел {yn} отличен от ноля, есть сходящаяся последовательность, предел которой равен частному пределов последовательностей {xn} и {yn}.

Доказательство: Из доказанной ранее леммы следует, что, начиная с некоторого номера N, элементы последовательности {yn} отличны от ноля и последовательность ограничена. Начиная с этого номера, мы и будем рассматривать последовательность . Пусть а и b – пределы последовательностей {xn} и {yn}. Докажем, что последовательность бесконечно малая. В самом деле, так как xn=а+a n, yn=b+b n, то

.

Так как последовательность ограничена, а последовательность бесконечно мала, то последовательность бесконечно малая. Теорема доказана.

Итак, теперь можно сказать, что арифметические операции над сходящимися последовательностями приводят к таким же арифметическим операциям над их пределами.

ТЕОРЕМА: Если элементы сходящейся последовательности {xn}, начиная с некоторого номера, удовлетворяют неравентству xb (xb), то и предел а этой последовательности удовлетворяет неравенству а³ b (a£ b).

Доказательство: Пусть все элементы xn, по крайней мере начиная с некоторого номера, удовлетворяют неравенству xb. Предположим, что а<b. Поскольку а – предел последовательности {xn}, то для положительного e =b-a можно указать номер N такой, что при n³ N выполняется неравенство

|xn-a|<b-a.

Это неравенство эквивалентно

-(b-a)<xn-a<b-a

Используя правое из этих неравенств мы получим xn<b, а это противоречит условию теоремы. Случай xb рассматривается аналогично. Теорема доказана.

Элементы сходящейся последовательности {xn} могут удовлетворять строгому неравенству xn>b, однако при этом предел а может оказаться равным b. Например, если xn=1/n, то xn>0, однако .

Следствие 1: Если элементы xn и уn у сходящихся последовательностей {xn} и {yn}, начиная с некоторого номера, удовлетворяют неравенству xn £ уn, то их пределы удовлетворяют аналогичному неравенству

.

Элементы последовательности {yn-xn} неотрицательны, а поэтому неотрицателен и ее предел . Отсюда следует, что

.

Следствие 2: Если все элементы сходящейся последовательности {xn} находятся на сегменте [a,b], то и ее предел с также находится на этом сегменте.

Это выполняется, так как а£ xb, то a£ c£ b.

ТЕОРЕМА: Пусть {xn} и {zn}- сходящиеся последовательности, имеющие общий предел а. Пусть, кроме того, начиная с некоторого номера, элементы последовательности {yn}удовлетворяют неравенствам xyzn. Тогда последовательность {yn} сходится и имеет предел а.

Доказательство: достаточно доказать, что {yn-a} является бесконечно малой. Обозначим через N’ номер, начиная с которого, выполняются неравенства, указанные в условии теоремы. Тогда, начиная с этого же номера, будут выполнятся также неравенства xn-а £ yn-а £ zn-а. Отсюда следует, что при n³ N’ элементы последовательности {yn-a} удовлетворяют неравенству

|yn-a| £ max {|xn-a|, |zn-a|}.


Так как и , то для любого e >0 можно указать номера N1 и N2 такие, что при n³ N1 |xn-a|<e , а при n³ N2 |zn-a|<e . Итак последовательность {yn-a} бесконечно малая. Теорема доказана.

Итак, мы показали неравенства, которым удовлетворяют элементы сходящихся последовательностей, в пределе переходят в соответствующие неравенства для пределов этих последовательностей.

ПРИМЕРЫ

  1. Последовательность сходится и имеет своим пределом ноль. Ведь каково бы ни было e >0, по свойству Архимеда вещественных чисел существует такое натуральное число ne , что ne >. Поэтому для всех n³ ne , а это означает, что .
  2. Последовательность сходится и
, что следует из того, что

, и того, что .

ЗАДАЧИ

ЗАДАЧА № 1

Пусть числовая последовательность а1, а2, а3, … удовлетворяет условию

(m, n = 1, 2, 3, … ),

тогда последовательность

,…

должна либо расходиться к , причем предел этой последовательности будет равен ее нижней грани.

РЕШЕНИЕ:

Видим частный случай теоремы у M. Fekete. Достаточно рассмотреть случай, когда нижняя грань a конечна. Пусть e >0 и a +e . Всякое целое число n может быть представлено в форме n=qm+r, где r=0 или 1, или 2, …, или m-1. Полагая единообразие а0=0, имеем:

an=aqm+r£ am+am+…+am+ar=qam+ar,

,

ЗАДАЧА № 2

Пусть числовая последовательность а1, а2, а3, … удовлетворяет условию

тогда существует конечный предел

,

причем

(n = 1, 2, 3, … ).

РЕШЕНИЕ:

Из неравенств 2am-1<a2m<2am+1 получаем:

(*)

Ряд

сходится, ибо в силу неравенства (*) он мажорируется сходящимся рядом:

|a1|+2-1+2-2+2-3+…

запишем целое число n по двоичной системе:

n=2m+e 12m-1+e 22m-2+…+e m (e 1, e 2, …, e m = 0 или 1)

согласно предположению

.

Применяя теорему (1) для данных:

s0=0, s1=, sm-1=, sm=, …, pn0=0, pn1=, …, pn, m-1=,

, pn, m+1=0, …,

заключаем, что . Наконец, в силу (*) имеем:

.

 

ЗАДАЧА № 3

Если общий член ряда, не являющегося ни сходящимся, ни расходящимся в собственном смысле, стремится к нулю, то частичные суммы этого ряда расположены всюду плотно между их нижним и верхним пределами lim inf и lim sup.

РЕШЕНИЕ:

Нам достаточно рассмотреть случай, когда частичные суммы s1, s2, …, sn, … ограничены. Пусть , , l — целое положительное число, l>2 и .

Разобьем числовую прямую на l интервалов точками

-¥ , m+d , m+2d , …, M-2d , M-d , +¥ .

Выберем такое N, чтобы для n>N выполнялось неравенство |sn-sn+1|<d . Пусть, далее, sn1 (n1>N) лежит в первом интервале и sn2 (n2> n1) – в последнем. Тогда числа конечной последовательности не смогут “перепрыгнуть” ни один из l-2 промежуточных интервалов длиной d . Аналогично рассуждаем и в том случае, когда последовательность будет не “медленно восходящей”, а “медленно нисхожящей”.

ЗАДАЧА № 4

Пусть для последовательности t1, t2, … , tn, … существует такая последовательность стремящихся к нулю положительных чисел …, что для каждого n

.


Тогда числа t1, t2, … , tn, …лежат всюду плотно между их нижним и верхним пределами.

РЕШЕНИЕ:

Существуют в сколь угодно большом удалении конечные последовательности , произвольно медленно нисходящие от верхнего предела последовательности к ее нижнему пределу.

ЗАДАЧА № 5

Пусть v1, v2, … , vn, … — положительные числа, v1 £ v2 £ v3 … Совокупность предельных точек последовательности

, …

заполняет замкнутый интервал (длина которого равна нулю, если эта последовательность стремится к пределу).

РЕШЕНИЕ:

ЗАДАЧА № 6

Числовая последовательность, стремящаяся к , имеет наименьший член.

РЕШЕНИЕ:

Какое бы число мы ни задали, слева от него будет находиться лишь конечное число членов последовательности, а среди конечного множества чисел существует одно или несколько наименьших.

ЗАДАЧА № 7

Сходящаяся последовательность имеет либо наибольший член, либо наименьший, либо и тот и другой.

РЕШЕНИЕ:

При совпадении верхней и нижней граней рассматриваемой последовательности теорема тривиальна. Пусть поэтому они различны. Тогда по крайней мере одна из них отличается от предела последовательности. Она и будет равна наибольшему, соответственно наименьшему, члену последовательности.

ЗАДАЧА № 8

Пусть l1, l2, l3, … , lm, … — последовательность положительных чисел и , тогда существует бесконечно много номеров n, для которых ln меньше всех предшествующих ему членов последовательности l1, l2, l3, … , ln-1.

РЕШЕНИЕ:

Пусть задано целое положительное число m и h – наименьшее из чисел l1, l2, l3, … , lm; h >0. Согласно предположению в рассматриваемой последовательности существуют члены, меньше чем h . Пусть n – наименьший номер, для которого ln<h . Тогда:

n>m; ln<l1, ln<l2, …, ln<ln-1.

ЗАДАЧА № 9

Пусть l1, l2, l3, … , lm, … — последовательность положительных чисел и , тогда существует бесконечно много номеров n, для которых ln превосходит все следующие за ним члены ln+1, ln+2, ln+3,…

ЗАДАЧА № 10

Пусть числовые последовательности

l1, l2, l3, … , lm, … (lm>0),

s1, s 2, s 3, … , s m, … (s1>0, sm+1>sm, m=1, 2, 3, …)

обладают тем свойством, что

, .

Тогда существует бесконечно много номеров n, для которых одновременно выполняются неравенства

ln>ln+1, ln>ln+2, ln>ln+3, …

lnsn>ln-1sn-1, lnsn>ln-2sn-2, … lnsn>l1s1,

РЕШЕНИЕ:

Будем называть lm “выступающим” членом последовательности, если lm больше всех последующих членов. Согласно предположению в первой последовательности содержится бесконечно много выступающих членов; пусть это будут:

,…

Каждый невыступающий член lv заключается (для v>n1) между двумя последовательными выступающими членами, скажем nr-1<v<nr. Имеем последовательно:

,

значит

(*)

отсюда заключаем, что

Действительно, в противном случае , значит, в силу (*) и вся последовательность
l1s1, l2s2, … были бы ограничены, что противоречит предположению. Теперь пусть задано целое положительное число m и h – наименьшее из чисел ,… ; h >0. Согласно предположению в рассматриваемой последовательности существуют члены, меньше чем h . Пусть k – наименьший номер, для которого <h . Тогда:

k>m; .

ЗАДАЧА № 11

Если числовая последовательность ,… стремится к и А превышает ее наименьший член, то существует такой номер n (возможно несколько таких), n³ 1, что n отношений


все не больше А, а бесконечное множество отношений

,…

все не меньше А.

РЕШЕНИЕ:

Имеем . Пусть минимум последовательности

L0-0, L1-A, L2-2A, L3-3A, …

Будет Ln-nA; тогда

Ln-u-(n-u)A³ Ln-nA; Ln+v-(n+v)A³ Ln-nA,

u=1, 2, …, n; v=1, 2, 3, …; n=0 исключено в силу предложений относительно А.

ЗАДАЧА № 12

Пусть относительно числовой последовательности l1, l2, l3, … , lm, … предполагается лишь, что

.

Пусть, далее, А>l1. Тогда существует такой номер n, n ³ 1, что одновременно выполняются все неравенства

.

Если А® ¥ , то также n® ¥ .

РЕШЕНИЕ:

Пусть

l1+l2+l3+…+lm=Lm, m=1, 2, 3, …; L0=0.

Так как L1-A<0, то L0-0 не является минимумом в предыдущем решении. ln+1³ A; поэтому ln+1, а следовательно и n должны стремиться к бесконечности одновременно с А.

ЗАДАЧА № 13

Пусть числовая последовательность l1, l2, l3, … , lm, … удовлетворяет условиям

,


Пусть, далее, l1>A>0. Тогда существует такой номер n, n ³ 1, что одновременно выполняются все неравенства

.

Если А® 0, то также n® 0.

РЕШЕНИЕ:

Положим

l1+l2+l3+…+lm=Lm, m=1, 2, 3, …; L0=0.

Тогда . Последовательность

L0-0, L1-A, L2-2A, L3-3A, …, Lm-mA, …

стремится к -¥ . Пусть ее наибольший член будет Ln-nA. Тогда интересующие нас неравенства будут выполняться для этого номера n.

В последовательности L0, L1, …, Lm, … содержится бесконечно много членов, превышающих все предыдущие. Пусть Ls будет один из них. Тогда числа:

все положительны: коль скоро А меньше наименьшего из них, соответствующий А номер n больше или равен s. Точки (n, Ln) должны быть обтянуты теперь бесконечным выпуклым сверху полигоном.

Дата добавления: 22.10.2000

Числовые последовательности

Рассмотрим ряд натуральных чисел: 1, 2, 3, , n – 1, n,  .

Если заменить каждое натуральное число n в этом ряду некоторым числом an, следуя некоторому закону, то получим новый ряд чисел:

a1, a2, a3, , an–1, an, ,

кратко обозначаемый и называемыйчисловой последователь- ностью. Величина an называется общим членом числовой последовательности. Обычно числовая последовательность задается некоторой формулой an = f(n) позволяющей найти любой член последовательности по его номеру n; эта формула называется формулой общего члена. Заметим, что задать числовую последовательность формулой общего члена не всегда возможно; иногда последовательность задается путем описания ее членов.

По определению, последовательность всегда содержит бесконечное множество элементов: любые два разных ее элемента отличаются, по крайней мере, своими номерами, которых бесконечно много.

Числовая последовательность является частным случаем функции. Последовательность является функцией, определенной на множестве натуральных чисел и принимающей значения в множестве действительных чисел, т. е. функцией вида f : NR.

Последовательность называетсявозрастающей (убывающей), если для любого n  N Такие последовательности называютсястрого монотонными.

Иногда в качестве номеров удобно использовать не все натуральные числа, а лишь некоторые из них (например, натуральные числа, начиная с некоторого натурального числа n0). Для нумерации также возможно использование не только натуральных, но и других чисел, например, n = 0, 1, 2,  (здесь в качестве еще одного номера к множеству натуральных чисел добавлен ноль). В таких случаях, задавая последовательность, указывают, какие значения принимают номера n.

Если в некоторой последовательности для любого n  N то последовательность называетсянеубывающей (невозрастающей). Такие последовательности называются монотонными.

Пример 1. Числовая последовательность 1, 2, 3, 4, 5, … является рядом натуральных чисел и имеет общий член an = n.

Пример 2. Числовая последовательность 2, 4, 6, 8, 10, … является рядом четных чисел и имеет общий член an = 2n.

Пример 3. 1.4, 1.41, 1.414, 1.4142, … − числовая последовательность приближенных значений с увеличивающейся точностью.

В последнем примере невозможно дать формулу общего члена последовательности.

Пример 4. Записать первых 5 членов числовой последовательности по ее общему члену . Для вычисленияa1 нужно в формулу для общего члена an вместо n подставить 1, для вычисления a2 − 2 и т. д. Тогда имеем:

Тест 6. Общим членом последовательности 1, 2, 6, 24, 120,  является:

1)

2)

3)

4)

Тест 7. Общим членом последовательности является:

1)

2)

3)

4)

Тест 8. Общим членом последовательности является:

1)

2)

3)

4)

Предел числовой последовательности

Рассмотрим числовую последовательность, общий член которой приближается к некоторому числу А при увеличении порядкового номера n. В этом случае говорят, что числовая последовательность имеет предел. Это понятие имеет более строгое определение.

Число А называется пределом числовой последовательности:

(1)

если для любого  > 0 найдется такое число n0 = n0(), зависящее от , что приn > n0.

Это определение означает, что А есть предел числовой последовательности, если ее общий член неограниченно приближается к А при возрастании n. Геометрически это значит, что для любого  > 0 можно найти такое число n0, что, начиная с n > n0, все члены последовательности расположены внутри интервала (А – , А + ). Последовательность, имеющая предел, называется сходящейся; в противном случае – расходящейся.

Числовая последовательность может иметь только один предел (конечный или бесконечный) определенного знака.

Пример 5. Гармоническая последовательность имеет пределом число 0. Действительно, для любого интервала (–; +) в качестве номера N0 можно взять какое-либо целое число, больше . Тогда для всехn > n0 >имеем

Пример 6. Последовательность 2, 5, 2, 5,  является расходящейся. Действительно, никакой интервал длины, меньшей, например, единицы, не может содержать всех членов последовательности, начиная с некоторого номера.

Последовательность называется ограниченной, если существует такое число М, что для всехn. Всякая сходящаяся последовательность ограничена. Всякая монотонная и ограниченная последовательность имеет предел. Всякая сходящаяся последовательность имеет единственный предел.

Пример 7. Последовательность является возрастающей и ограниченной. Она имеет предел=е.

Число e называется числом Эйлера и приблизительно равно 2,718 28.

Тест 9. Последовательность 1, 4, 9, 16,  является:

1) сходящейся;

2) расходящейся;

3) ограниченной;

4) арифметической прогрессией;

5) геометрической прогрессией.

Тест 10. Последовательность является:

1) сходящейся;

2) расходящейся;

3) ограниченной;

4) арифметической прогрессией;

5) геометрической прогрессией.

Тест 11. Последовательность не является:

1) сходящейся;

2) расходящейся;

3) ограниченной;

4) гармонической.

Тест 12. Предел последовательности, заданной общим членом равен:

1) 1;

2) 0;

3) e;

4) .

Узнаем как доказать, что последовательность сходится? Основные свойства сходящихся последовательностей

Для многих людей математический анализ представляет собой лишь набор непонятных цифр, значков и определений, далёких от реальной жизни. Однако, мир, в котором существуем мы, построен на числовых закономерностях, выявление которых помогает не просто познавать окружающий мир и решать его сложные проблемы, но и упрощать бытовые практические задачи. Что имеет в виду математик, когда говорит, что числовая последовательность сходится? Об этом следует поговорить подробнее.

Представим себе матрёшек, которые помещаются одна в другой. Размеры их, записанные в виде цифр, начиная с большей и кончая меньшей из них, формируют последовательность. Если вообразить бесконечное количество подобных ярких фигурок, то получившийся ряд окажется фантастически длинным. Это сходящаяся числовая последовательность. И стремится она к нулю, так как размеры каждой последующей матрёшки, катастрофически уменьшаясь, постепенно превращаются в ничто. Таким образом, легко можно объяснить: что такое бесконечно малое.

Похожим примером может стать дорога, уходящая вдаль. А визуальные размеры автомобиля, уезжающего по ней от наблюдателя, постепенно сокращаясь, превращаются в бесформенное пятнышко, напоминающее точку. Таким образом, машина, как некий объект, удаляясь в неизвестном направлении, становится бесконечно маленькой. Параметры указанного тела никогда не будут нулевыми в прямом смысле этого слова, но неизменно стремятся к этой величине в конечном пределе. Поэтому данная последовательность сходится снова к нулю.

Рассчитаем всё по каплям

Вообразим теперь житейскую ситуацию. Больному врач прописал принимать микстуру, начиная с десяти капель в день и прибавляя по две в каждые последующие сутки. И так доктор предложил продолжать до тех пор, пока не кончится содержимое пузырька с лекарством, объём которого составляет 190 капель. Из изложенного следует, что количество таковых, расписанное по дням составит следующий числовой ряд: 10, 12, 14 и так далее.

Как выяснить время прохождения всего курса и количество членов последовательности? Здесь, конечно, можно подсчитывать капли примитивным образом. Но гораздо легче, учитывая закономерность, воспользоваться формулой суммы арифметической прогрессии с шагом d = 2. И с применением такого метода выяснить, что количество членов числового ряда равно 10. При этом а10 = 28. Номер члена указывает на количество дней приёма лекарства, а 28 соответствует числу капель, которые больной должен употребить в последний день. Данная последовательность сходится? Нет, потому что, несмотря на то, что снизу она ограничена числом 10, а сверху – 28, такой числовой ряд не имеет предела, в отличие от предыдущих примеров.

В чём разница?

Попробуем теперь уточнить: когда числовой ряд оказывается сходящейся последовательностью. Определение такого рода, как можно заключить из вышеописанного, напрямую связано с понятием конечного предела, наличие которого и выявляет суть вопроса. Так в чём принципиальное отличие ранее приведённых примеров? И почему в последнем из них число 28 не может считаться пределом числового ряда Xn = 10 + 2(n-1)?

Для выяснения этого вопроса рассмотрим другую последовательность, заданную нижеуказанной формулой, где n принадлежит множеству натуральных чисел.

Данное сообщество членов представляет собой набор обыкновенных дробей, числитель которых 1, а знаменатель постоянно увеличивается: 1, ½ …

Причём каждый последующий представитель этого ряда по расположению на числовой прямой всё больше приближается к 0. А это значит, что появляется такая окрестность, где точки скучиваются вокруг нуля, который и является пределом. И чем ближе они к нему, тем плотнее становится их концентрация на числовой прямой. А расстояние между ними катастрофически сокращается, превращаясь в бесконечно малое. Это признак того, что последовательность сходится.

Подобным же образом разноцветные прямоугольники, изображённые на рисунке, при удалении в пространстве визуально располагаются кучнее, в гипотетическом пределе превращаясь в ничтожно малые.

Бесконечно большие последовательности

Разобрав определение сходящейся последовательности, перейдём теперь к противоположным примерам. Многие из них были известны человеку с самых древних времён. Простейшими вариантами расходящихся последовательностей являются ряды натуральных и чётных чисел. Они по-другому именуются бесконечно большими, так как члены их, постоянно увеличиваясь, всё больше приближаются к положительной бесконечности.

Примерами таковых также могут служить любая из арифметических и геометрических прогрессий с шагом и знаменателем соответственно больше нуля. Расходящимися последовательностями считаются, к тому же, числовые ряды, которые и вовсе не имеют предела. К примеру, Xn= (-2)n-1.

Последовательность Фибоначчи

Практическая польза указанных ранее числовых рядов для человечества несомненна. Но существует огромное множество и других замечательных примеров. Одним из них является последовательность Фибоначчи. Каждый из её членов, которые начинаются с единицы, представляет собой сумму предыдущих. Первыми двумя её представителями являются 1 и 1. Третий 1+1=2, четвёртый 1+2=3, пятый 2+3=5. Далее, согласно этой же логике, следуют числа 8, 13, 21 и так далее.

Данный ряд чисел неограниченно возрастает и не имеет конечного предела. Зато он обладает ещё одним замечательным свойством. Отношение каждого предыдущего числа к последующему всё более приближается по своему значению к 0, 618. Здесь можно уяснить разницу между сходящейся и расходящейся последовательностью, ведь если составить ряд из полученных частных от делений, указанный числовой строй будет иметь конечный предел равный 0,618.

Последовательность коэффициентов Фибоначчи

Указанный выше числовой ряд широко используется в практических целях для технического анализа рынков. Но этим не ограничиваются его возможности, которые знали и умели применять на практике ещё в глубокой древности египтяне и греки. Это доказывают построенные ими пирамиды и Парфенон. Ведь число 0, 618 является постоянным коэффициентом хорошо известного в старину золотого сечения. Согласно этому правилу, любой произвольный отрезок возможно поделить так, что отношение между его частями будет совпадать с отношением между большим из отрезков и общей длиной.

Построим ряд из указанных отношений и попытаемся проанализировать данную последовательность. Числовой ряд получится следующим: 1; 0,5; 0,67; 0,6; 0,625; 0,615; 0,619 и так далее. Продолжая, таким образом можно убедиться, что предел сходящейся последовательности действительно будет 0,618. Однако, необходимо заметить и прочие свойства этой закономерности. Здесь цифры как бы идут вразнобой, а вовсе не в порядке возрастания или убывания. Это означает, что данная сходящаяся последовательность монотонной не является. О том, почему это так и пойдёт разговор далее.

Монотонность и ограниченность

Члены числового ряда с увеличением номера могут чётко убывать (если x1>x2>x3>…>xn>…) или возрастать (если x1<x2<x3<…<xn<…). В этом случае последовательность называется строго монотонной. Могут наблюдаться также другие закономерности, где числовой ряд будет неубывающим и невозрастающим (x1≥x2≥x3≥…≥xn≥… или x1≤x2≤x3≤…≤xn≤…), тогда сходящаяся последовательно монотонна тоже, только уже не в строгом смысле. Хорошим примером первого из указанных вариантов может служить числовой ряд, задаваемый следующей формулой.

Расписав числа данного ряда можно заметить, что любой из его членов, неограниченно приближаясь к 1, никогда не превысит этого значения. В этом случае говорят об ограниченности сходящейся последовательности. Подобное бывает всякий раз, когда находится такое положительное число М, которое оказывается всегда больше любого из членов ряда по модулю. Если числовой ряд обладает признаками монотонности и имеет предел, а следовательно – сходится, то он обязательно наделён таким свойством. Причём обратное не обязательно должно быть верным. Об этом говорит теорема об ограниченности сходящейся последовательности.

Применение подобных наблюдений на практике оказывается очень полезным. Приведём конкретный пример, исследовав свойства последовательности Xn = n/n+1, и докажем её сходимость. То, что она монотонна легко показать, так как (xn+1 – xn) есть число положительное при любых значениях n. Предел последовательности равен числу 1, а значит, соблюдаются все условия вышеуказанной теоремы, называемой также теоремой Вейерштрасса. Теорема об ограниченности сходящейся последовательности утверждает, что если она имеет предел, то в любом случае оказывается ограниченной. Однако, приведём следующий пример. Числовой ряд Xn = (-1)n является ограниченным снизу числом -1 и сверху 1. Но данная последовательность не является монотонной, не имеет предела и поэтому не сходится. То есть из ограниченности не всегда следует наличие предела и сходимости. Чтобы это выполнялось необходимо совпадение нижнего и верхнего предела, как в случае коэффициентов Фибоначчи.

Числа и законы Вселенной

Простейшими вариантами сходящейся и расходящейся последовательности являются, пожалуй, числовые ряды Xn = n и Xn = 1/n. Первая из них представляет собой натуральный ряд чисел. Она же является, как уже говорилось, бесконечно большой. Вторая сходящаяся последовательность ограничена, а члены её по величине приближаются к бесконечно малому. Каждая из этих формул олицетворяет одну из сторон многогранной Вселенной, помогая человеку на языке цифр и знаков представить себе и просчитать нечто непознаваемое, недоступное для ограниченного восприятия.

Законы мироздания, начиная от ничтожно малого и кончая невероятно большим, выражает также золотой коэффициент 0,618. Учёные считают, что он заложен в основу сути вещей и используется природой для формирования её частей. Упомянутые уже нами ранее отношения между последующим и предыдущим членами ряда Фибоначчи, не завершают на этом демонстрацию удивительных свойств этого уникального ряда. Если рассмотреть частное от деления предыдущего члена на последующей через один, то получим ряд 0,5; 0, 33; 0,4; 0,375; 0,384; 0,380; 0,382 и так далее. Интересно то, что эта ограниченная последовательность сходится, монотонной она не является, но отношение крайних от определённого члена соседних чисел всегда приблизительно оказывается равным 0,382, что тоже может быть использовано в архитектуре, техническом анализе и других отраслях.

Существуют и другие интересные коэффициента ряда Фибоначчи, все они играют в природе особую роль, а также применяются человеком в практических целях. Математики уверены, что Вселенная развивается по некоей «золотой спирали», формируемой из указанных коэффициентов. С их помощью возможно рассчитать многие явления, происходящие на Земле и в космосе, начиная от роста численности определённых бактерий и кончая движением далёких комет. Подобным же законам подчиняется, как выясняется, код ДНК.

Убывающая геометрическая прогрессия

Существует теорема, утверждающая единственность предела сходящейся последовательности. Это значит, что двух и более пределов у неё существовать не может, что несомненно важно для нахождения её математических характеристик.

Рассмотрим некоторые случаи. Любой числовой ряд, составленный из членов арифметической прогрессии, является расходящимся, за исключением случая с нулевым шагом. Это же касается геометрической прогрессии, знаменатель которой больше 1. Пределами таких числовых рядов являются «плюс» или «минус» бесконечности. Если же знаменатель меньше -1, то никакого предела вообще не существует. Возможны и другие варианты.

Рассмотрим числовой ряд, задаваемой формулой Xn = (1/4)n-1. С первого взгляда легко понять, что эта сходящаяся последовательность ограничена, потому что является строго убывающей и никаким образом не способна принимать отрицательные значения.

Распишем некоторое число её членов в ряд.

Получится: 1; 0,25; 0,0625; 0,015625; 0,00390625 и так далее. Достаточно совсем несложных расчётов, чтобы понять, как быстро данная геометрическая прогрессия со знаменателей 0<q<1 уменьшается. В то время как знаменатель членов неограниченно возрастает, сами они превращаются в бесконечно малое. Это значит, что предел числового ряда равен 0. Данный пример ещё раз демонстрирует ограниченность сходящейся последовательности.

Фундаментальные последовательности

Огюстен Луи Коши, французский учёный, явил миру много работ связанных с математическим анализом. Он дал определения таким его понятиям, как дифференциал, интеграл, предел и непрерывность. Исследовал он также основные свойства сходящихся последовательностей. Для того, чтобы понять суть его идей, необходимо обобщить некоторые важные детали.

В самом начале статьи было показано, что есть такие последовательности, для которых существует окрестность, где точки, изображающие члены определённого ряда на числовой прямой, начинают скучиваться, выстраиваясь всё плотнее. При этом расстояние между ними при увеличении номера очередного представителя всё уменьшается, превращаясь в бесконечно малое. Таким образом, оказывается, что в данной окрестности группируется бесконечное число представителей данного ряда, в то время, как за её пределами их насчитывается конечное количество. Такие последовательности именуются фундаментальными.

Знаменитый критерий Коши, созданный французским математиком, однозначно указывает, что наличия подобного свойства достаточно, чтобы доказать, что последовательность сходится. Верно также обратное.

Следует заметить, что данное заключение французского математика представляет по большей части чисто теоретический интерес. Его применение на практике считается достаточно сложным делом, поэтому для выяснения сходимости рядов гораздо важнее доказать существование у последовательности конечного предела. В противном же случае она считается расходящейся.

При решении задач следует также учитывать основные свойства сходящихся последовательностей. Они представлены ниже.

Бесконечные суммы

Такие знаменитые учёные древности, как Архимед, Евклид, Евдокс использовали суммы бесконечных числовых рядов для вычисления длин кривых, объёмов тел и площадей фигур. В частности, именно таким образом удалось узнать площадь параболического сегмента. Для этого была использована сумма числового ряда геометрической прогрессии с q=1/4. Подобным способом находились объёмы и площади других произвольных фигур. Данный вариант назывался методом «исчерпывания». Идея заключалось в том, что исследуемое сложное по формам тело разбивалось на части, которые представляли собой фигуры с легко измеряемыми параметрами. По этой причине нетрудно было вычислить их площади и объёмы, потом же они складывались.

Кстати, похожие задачи очень знакомы современным школьникам и встречаются в заданиях ЕГЭ. Уникальный способ, найденный ещё далёкими предками, является и на сегодняшний день самым простейшим вариантом решения. Даже если частей, на которые разбивается числовая фигура, всего две или три, сложение их площадей всё равно представляет собой сумму числового ряда.

Гораздо позднее древнегреческих учёных Лейбниц и Ньютон, основываясь на опыте мудрых предшественников, познавали закономерности интегрального вычисления. Знания свойств последовательностей помогали им решать дифференциальные и алгебраические уравнения. В настоящее время созданная усилиями многих поколений талантливых учёных теория рядов даёт шанс решить огромное количество математических и практических проблем. А изучение числовых последовательностей составляет основную задачу, решаемую математическим анализом с момента его создания.

Как доказать что последовательность сходится

Для многих людей математический анализ представляет собой лишь набор непонятных цифр, значков и определений, далёких от реальной жизни. Однако, мир, в котором существуем мы, построен на числовых закономерностях, выявление которых помогает не просто познавать окружающий мир и решать его сложные проблемы, но и упрощать бытовые практические задачи. Что имеет в виду математик, когда говорит, что числовая последовательность сходится? Об этом следует поговорить подробнее.

Что такое бесконечно малое?

Представим себе матрёшек, которые помещаются одна в другой. Размеры их, записанные в виде цифр, начиная с большей и кончая меньшей из них, формируют последовательность. Если вообразить бесконечное количество подобных ярких фигурок, то получившийся ряд окажется фантастически длинным. Это сходящаяся числовая последовательность. И стремится она к нулю, так как размеры каждой последующей матрёшки, катастрофически уменьшаясь, постепенно превращаются в ничто. Таким образом, легко можно объяснить: что такое бесконечно малое.

Похожим примером может стать дорога, уходящая вдаль. А визуальные размеры автомобиля, уезжающего по ней от наблюдателя, постепенно сокращаясь, превращаются в бесформенное пятнышко, напоминающее точку. Таким образом, машина, как некий объект, удаляясь в неизвестном направлении, становится бесконечно маленькой. Параметры указанного тела никогда не будут нулевыми в прямом смысле этого слова, но неизменно стремятся к этой величине в конечном пределе. Поэтому данная последовательность сходится снова к нулю.

Рассчитаем всё по каплям

Вообразим теперь житейскую ситуацию. Больному врач прописал принимать микстуру, начиная с десяти капель в день и прибавляя по две в каждые последующие сутки. И так доктор предложил продолжать до тех пор, пока не кончится содержимое пузырька с лекарством, объём которого составляет 190 капель. Из изложенного следует, что количество таковых, расписанное по дням составит следующий числовой ряд: 10, 12, 14 и так далее.

Как выяснить время прохождения всего курса и количество членов последовательности? Здесь, конечно, можно подсчитывать капли примитивным образом. Но гораздо легче, учитывая закономерность, воспользоваться формулой суммы арифметической прогрессии с шагом d = 2. И с применением такого метода выяснить, что количество членов числового ряда равно 10. При этом а10 = 28. Номер члена указывает на количество дней приёма лекарства, а 28 соответствует числу капель, которые больной должен употребить в последний день. Данная последовательность сходится? Нет, потому что, несмотря на то, что снизу она ограничена числом 10, а сверху – 28, такой числовой ряд не имеет предела, в отличие от предыдущих примеров.

В чём разница?

Попробуем теперь уточнить: когда числовой ряд оказывается сходящейся последовательностью. Определение такого рода, как можно заключить из вышеописанного, напрямую связано с понятием конечного предела, наличие которого и выявляет суть вопроса. Так в чём принципиальное отличие ранее приведённых примеров? И почему в последнем из них число 28 не может считаться пределом числового ряда Xn = 10 + 2(n-1)?

Для выяснения этого вопроса рассмотрим другую последовательность, заданную нижеуказанной формулой, где n принадлежит множеству натуральных чисел.

Данное сообщество членов представляет собой набор обыкновенных дробей, числитель которых 1, а знаменатель постоянно увеличивается: 1, ½ …

Причём каждый последующий представитель этого ряда по расположению на числовой прямой всё больше приближается к 0. А это значит, что появляется такая окрестность, где точки скучиваются вокруг нуля, который и является пределом. И чем ближе они к нему, тем плотнее становится их концентрация на числовой прямой. А расстояние между ними катастрофически сокращается, превращаясь в бесконечно малое. Это признак того, что последовательность сходится.

Подобным же образом разноцветные прямоугольники, изображённые на рисунке, при удалении в пространстве визуально располагаются кучнее, в гипотетическом пределе превращаясь в ничтожно малые.

Бесконечно большие последовательности

Разобрав определение сходящейся последовательности, перейдём теперь к противоположным примерам. Многие из них были известны человеку с самых древних времён. Простейшими вариантами расходящихся последовательностей являются ряды натуральных и чётных чисел. Они по-другому именуются бесконечно большими, так как члены их, постоянно увеличиваясь, всё больше приближаются к положительной бесконечности.

Примерами таковых также могут служить любая из арифметических и геометрических прогрессий с шагом и знаменателем соответственно больше нуля. Расходящимися последовательностями считаются, к тому же, числовые ряды, которые и вовсе не имеют предела. К примеру, Xn = (-2) n -1 .

Последовательность Фибоначчи

Практическая польза указанных ранее числовых рядов для человечества несомненна. Но существует огромное множество и других замечательных примеров. Одним из них является последовательность Фибоначчи. Каждый из её членов, которые начинаются с единицы, представляет собой сумму предыдущих. Первыми двумя её представителями являются 1 и 1. Третий 1+1=2, четвёртый 1+2=3, пятый 2+3=5. Далее, согласно этой же логике, следуют числа 8, 13, 21 и так далее.

Данный ряд чисел неограниченно возрастает и не имеет конечного предела. Зато он обладает ещё одним замечательным свойством. Отношение каждого предыдущего числа к последующему всё более приближается по своему значению к 0, 618. Здесь можно уяснить разницу между сходящейся и расходящейся последовательностью, ведь если составить ряд из полученных частных от делений, указанный числовой строй будет иметь конечный предел равный 0,618.

Последовательность коэффициентов Фибоначчи

Указанный выше числовой ряд широко используется в практических целях для технического анализа рынков. Но этим не ограничиваются его возможности, которые знали и умели применять на практике ещё в глубокой древности египтяне и греки. Это доказывают построенные ими пирамиды и Парфенон. Ведь число 0, 618 является постоянным коэффициентом хорошо известного в старину золотого сечения. Согласно этому правилу, любой произвольный отрезок возможно поделить так, что отношение между его частями будет совпадать с отношением между большим из отрезков и общей длиной.

Построим ряд из указанных отношений и попытаемся проанализировать данную последовательность. Числовой ряд получится следующим: 1; 0,5; 0,67; 0,6; 0,625; 0,615; 0,619 и так далее. Продолжая, таким образом можно убедиться, что предел сходящейся последовательности действительно будет 0,618. Однако, необходимо заметить и прочие свойства этой закономерности. Здесь цифры как бы идут вразнобой, а вовсе не в порядке возрастания или убывания. Это означает, что данная сходящаяся последовательность монотонной не является. О том, почему это так и пойдёт разговор далее.

Монотонность и ограниченность

Члены числового ряда с увеличением номера могут чётко убывать (если x1>x2>x3>…>xn>…) или возрастать (если x1 n является ограниченным снизу числом -1 и сверху 1. Но данная последовательность не является монотонной, не имеет предела и поэтому не сходится. То есть из ограниченности не всегда следует наличие предела и сходимости. Чтобы это выполнялось необходимо совпадение нижнего и верхнего предела, как в случае коэффициентов Фибоначчи.

Числа и законы Вселенной

Простейшими вариантами сходящейся и расходящейся последовательности являются, пожалуй, числовые ряды Xn = n и Xn = 1/n. Первая из них представляет собой натуральный ряд чисел. Она же является, как уже говорилось, бесконечно большой. Вторая сходящаяся последовательность ограничена, а члены её по величине приближаются к бесконечно малому. Каждая из этих формул олицетворяет одну из сторон многогранной Вселенной, помогая человеку на языке цифр и знаков представить себе и просчитать нечто непознаваемое, недоступное для ограниченного восприятия.

Законы мироздания, начиная от ничтожно малого и кончая невероятно большим, выражает также золотой коэффициент 0,618. Учёные считают, что он заложен в основу сути вещей и используется природой для формирования её частей. Упомянутые уже нами ранее отношения между последующим и предыдущим членами ряда Фибоначчи, не завершают на этом демонстрацию удивительных свойств этого уникального ряда. Если рассмотреть частное от деления предыдущего члена на последующей через один, то получим ряд 0,5; 0, 33; 0,4; 0,375; 0,384; 0,380; 0,382 и так далее. Интересно то, что эта ограниченная последовательность сходится, монотонной она не является, но отношение крайних от определённого члена соседних чисел всегда приблизительно оказывается равным 0,382, что тоже может быть использовано в архитектуре, техническом анализе и других отраслях.

Существуют и другие интересные коэффициента ряда Фибоначчи, все они играют в природе особую роль, а также применяются человеком в практических целях. Математики уверены, что Вселенная развивается по некоей «золотой спирали», формируемой из указанных коэффициентов. С их помощью возможно рассчитать многие явления, происходящие на Земле и в космосе, начиная от роста численности определённых бактерий и кончая движением далёких комет. Подобным же законам подчиняется, как выясняется, код ДНК.

Убывающая геометрическая прогрессия

Существует теорема, утверждающая единственность предела сходящейся последовательности. Это значит, что двух и более пределов у неё существовать не может, что несомненно важно для нахождения её математических характеристик.

Рассмотрим некоторые случаи. Любой числовой ряд, составленный из членов арифметической прогрессии, является расходящимся, за исключением случая с нулевым шагом. Это же касается геометрической прогрессии, знаменатель которой больше 1. Пределами таких числовых рядов являются «плюс» или «минус» бесконечности. Если же знаменатель меньше -1, то никакого предела вообще не существует. Возможны и другие варианты.

Рассмотрим числовой ряд, задаваемой формулой Xn = (1/4) n -1 . С первого взгляда легко понять, что эта сходящаяся последовательность ограничена, потому что является строго убывающей и никаким образом не способна принимать отрицательные значения.

Распишем некоторое число её членов в ряд.

Получится: 1; 0,25; 0,0625; 0,015625; 0,00390625 и так далее. Достаточно совсем несложных расчётов, чтобы понять, как быстро данная геометрическая прогрессия со знаменателей 0 15 ноября, 2017

Последовательность $left
ight>$ называется сходящейся, если существует такое число $a in R$ такое, что последовательность $left-a
ight>$ является бесконечно малой последовательностью.

Число $a$ называется пределом последовательности $left
ight>$ и обозначается $lim _x_=lim _x_=a$, $x_underset <longrightarrow>a$

Число $a$ называется пределом последовательности $left
ight>$ , если для любого $epsilon>0$ существует номер $n_<0>=n_<0>(epsilon)$ такой, что для любого $n>n_<0>$ выполняется неравенство $left|x_-a
ight| 0, exists n_<0>=n_<0>(epsilon) : forall n>n_<0>,left|x_-a
ight| Определение

Целой частью $[x]$ некоторого числа $x$ называется наибольшее целое число, не превосходящее $x$

Задание. Найти целую часть чисел – 2,36; 2,36; 2.

Решение. $[-2,36]=-3,[2,36]=2,[2]=2$

Задание. Доказать равенство: $lim _ frac<1>=0$

Доказательство. Исходя из определения, 0 будет пределом последовательности $frac<1>$ , если для любого $epsilon>0$ найдется такой номер $n_<0>=n_<0>(epsilon)$, что для любого $n>n_<0>$ выполняется неравенство $left|x_-0
ight| frac<1><epsilon>$

В качестве $n_<0>$ возьмем $n_<0>=left[frac<1><epsilon>
ight]+1$

Итак, для любого $n>n_<0>$ указано соответствующее значение $n_<0>$ , а тогда равенство $lim _ frac<1>=0$ доказано.

Сходящиеся и расходящиеся последовательности

Последовательность, которая имеет предел, называется сходящейся; иначе – расходящейся.

Задание. Доказать, что последовательность $x_=(-1)^$ не имеет предел.

Доказательство. Пусть $a$ – предел рассматриваемой последовательности, то есть $lim _ x_=a$. Рассмотрим $epsilon=frac<1> <10>Rightarrow exists n_<0>=n_<0>(epsilon) in N : n>n_ <0>:left|x_-a
ight| Теорема

Сходящаяся последовательность имеет только один предел.

(Необходимый признак сходимости последовательности).

Последовательность на бесконечности

Последовательность $left
ight>$ имеет бесконечный предел, если для любого $epsilon>0, exists n_ <0>in N : n>n_ <0>:$ $x_>epsilon : lim _x_=infty$

Последовательность $left
ight>$ называется бесконечно малой, если $lim _x_=0$

Последовательность $left
ight>$ называется бесконечно большой, если для любого $epsilon>0$ существует номер $n_<0>$ такое, что для любого $n>n_ <0>:left|x_
ight|>epsilon$

в) если $b
eq 0$ , то начиная с некоторого номера заданная последовательность $lim _ frac>>=frac$

Определение подпоследовательности

Определение
Подпоследовательность последовательности – это последовательность , полученная из , удалением ряда ее членов без изменения порядка следования членов.

То есть подпоследовательность состоит из членов исходной последовательности с номерами , где – строго монотонная последовательность натуральных чисел.

Также можно сказать, что подпоследовательность последовательности – это подмножество множества , сохраняющее порядок следования членов.

Свойства подпоследовательностей

Далее мы используем понятие расширенного множества действительных чисел . Выражение означает, что a является или действительным числом, или элементом , или элементом .
См. «Бесконечно удаленные точки и их свойства».

1. Свойство подпоследовательностей сходящейся последовательности
Если последовательность сходится к числу , то и любая ее подпоследовательность сходится к этому же числу.
Доказательство ⇓

2. Свойство последовательности, все подпоследовательности которой сходятся к одному числу
Если любая подпоследовательность последовательности содержит подпоследовательность, сходящуюся к одному и тому же числу , то и сама последовательность сходится к этому числу:
.
Доказательство ⇓

3. Свойство эквивалентности сходимости последовательности и всех ее подпоследовательностей
Последовательность сходится тогда и только тогда, когда любая ее подпоследовательность сходится к одному числу .

Свойство 3 является следствием свойств 1 и 2.

Частичный предел последовательности

4. Теорема Больцано – Вейерштрасса
Из любой последовательности действительных чисел можно выделить подпоследовательность, сходящуюся к числу .

Определение частичного предела последовательности
Точка называется частичным пределом последовательности , если существует подпоследовательность , сходящаяся к точке a .

Произвольная последовательность может иметь конечное или бесконечное число частичных пределов.

5. Свойство частичного предела последовательности
Точка является частичным пределом последовательности тогда и только тогда, когда в любой окрестности точки a содержится бесконечное число членов последовательности.
Доказательство ⇓

Верхний и нижний частичный предел последовательности

Определение
Верхний (нижний) частичный предел последовательности – это число , которое является наибольшим (наименьшим) частичным пределом последовательности. Верхний и нижний частичные пределы обозначаются, соответственно, так:
.

6. Теорема о существовании верхнего и нижнего частичных пределов
У любой последовательности существует как верхний, так и нижний частичный пределы, принадлежащие расширенному множеству действительных чисел .
Доказательство ⇓

Рассмотрим множество частичных пределов последовательности. Эта теорема утверждает, что верхняя и нижняя грани этого множества являются ее элементами. То есть множество частичных пределов последовательности замкнуто, оно содержит свою границу. Для произвольного множества это может не выполняться. Например, для открытого интервала не существует наибольшего и наименьшего элемента, поскольку и верхняя грань b и нижняя a не принадлежит этому множеству.

Если последовательность не ограничена сверху, то ее верхний частичный предел равен плюс бесконечности:
.
Соответственно, если последовательность не ограничена снизу, то
.

Если последовательность ограничена, то ее верхний и нижний частичные пределы конечны.

7. Свойство верхнего и нижнего частичных пределов
Пусть – ограниченная последовательность. Пусть a – ее верхний (нижний) частичный предел. Тогда, для любого , в интервале содержится бесконечное число членов последовательности, а в полуинтервале – конечное или пустое множество.
Доказательство ⇓

8. Теорема о неравенстве между верхним и нижним частичными пределами
Верхний и нижний частичные пределы последовательности удовлетворяют неравенству:
.
Частичные пределы равны друг другу тогда и только тогда, когда существует предел последовательности:
.
Доказательство ⇓

9. Связь верхних и нижних пределов между последовательностями n> и <–xn>.
Имеют место очевидное равенство:
.

10. Свойства верхних и нижних пределов суммы последовательностей
Верхний и нижний частичные пределы от суммы последовательностей удовлетворяют следующим неравенствам:
;
,
где последовательности и ограничены.
Доказательство ⇓

11. Свойство верхних пределов произведения последовательностей
Пусть последовательность сходится к конечному положительному числу:
.
И пусть – любая последовательность. Тогда
.
Отсюда
.
Доказательство ⇓

Применяя равенство
,
можно получить другие подобные соотношения.

Доказательство свойств и теорем

Далее перечислены определения и свойства, которые мы будем использовать при доказательстве свойств подпоследовательностей.

Определение окрестности точки. Окрестностью конечной точки называется любой открытый интервал, содержащий эту точку: , где и – произвольные положительные числа.
См. «Определение окрестности точки».
Окрестностью точки называется множество ;
Окрестностью точки называется множество ,
где M – произвольное действительное число.
См. «Окрестности бесконечно удаленных точек».

Также мы будем использовать следующие обозначения для ε -окрестностей точек:
;
;
.

Определение предела последовательности. Точка является пределом последовательности , если для любой окрестности этой точки существует такое натуральное число N , что все элементы последовательности с номерами n > N принадлежат этой окрестности.
См. «Универсальное определение предела последовательности».

Свойство (*) предела последовательности. Для того, чтобы точка являлась пределом последовательности , необходимо и достаточно, чтобы за пределами любой окрестности этой точки находилось конечное число членов последовательности или пустое множество.
См. «Определение бесконечно большой последовательности: Свойство 1».

Доказательство свойства подпоследовательностей сходящейся последовательности

Формулировка ⇑ Действительно, поскольку последовательность сходится к числу a , то, согласно свойству (*) ⇑, за пределами любой окрестности точки a находится конечное число членов последовательности или пустое множество. Но, поскольку подпоследовательность получается из последовательности путем вычеркивания ряда ее членов, то за пределами любой окрестности точки a может находиться только конечное число членов подпоследовательности (или пустое множество). Согласно свойству (*) ⇑ это означает, что точка a является пределом подпоследовательности.

Доказательство свойства последовательности, все подпоследовательности которой сходятся к одному числу

Формулировка ⇑ Допустим противное. Пусть последовательность не сходится к числу a . Тогда существует такая окрестность точки a , вне которой имеется бесконечное число членов (см. «Определение отсутствия предела последовательности»). Составим из этих членов подпоследовательность . Из нее нельзя выделить подпоследовательность, сходящуюся к a , поскольку все члены подпоследовательности находятся за пределами окрестности .

Мы получили противоречие, так как по условию теоремы, из любой подпоследовательности можно выделить подпоследовательность, сходящуюся к числу a .
Свойство доказано.

Доказательство свойства частичного предела последовательности

Пусть в любой окрестности точки a содержится бесконечное число членов последовательности . Покажем, что из нее можно выделить подпоследовательность, сходящуюся к a .

Возьмем произвольную окрестность точки a : ⇑. В качестве первого члена подпоследовательности возьмем любой член последовательности, принадлежащий этой окрестности.

Возьмем более узкую окрестность: и выберем из нее второй член с номером .

И так далее. Поскольку любая окрестность точки a содержит бесконечное число членов последовательности, то мы на k – ом шаге можем выбрать член последовательности , принадлежащий окрестности с номером .

Так как член подпоследовательности с номером k принадлежит окрестности , то эта подпоследовательность сходится к числу a . Действительно, для любого имеется такой номер , что все члены подпоследовательности с номерами принадлежат ε – окрестности точки a .

Пусть теперь точка a является частичным пределом последовательности . Это означает, что существует подпоследовательность , сходящаяся к точке a . Тогда по свойству сходящихся последовательностей, в любой окрестности точки a находится бесконечное число членов подпоследовательности.

Доказательство теоремы о существовании верхнего и нижнего частичных пределов

Пусть у нас имеется некоторая последовательность . Докажем, что у нее существует верхний частичный предел.

Пусть последовательность неограниченна сверху. То есть для любого числа M существует член последовательности , превышающий M : . В свою очередь существует член последовательности , превышающий : . Продолжая подобные рассуждения мы приходим к выводу, что существует бесконечное число членов последовательности, превышающих M . Поскольку это утверждение справедливо для любого числа M , то в любой окрестности точки содержится бесконечное число членов последовательности . Тогда по свойству 4 ⇑, из последовательности можно выделить подпоследовательность, сходящуюся к .

В этом случае точка является верхним частичным пределом последовательности.

Пусть последовательность ограничена сверху и при этом любой отрезок содержит только конечное число членов последовательности.

В этом случае последовательность сходится к . Точка является единственным частичным пределом последовательности – как верхним, так и нижним.

Пусть последовательность ограничена сверху и при этом существует отрезок , содержащий бесконечное число членов последовательности.

В этом случае поступаем как при доказательстве теоремы Больцано – Вейерштрасса, применяя систему вложенных отрезков. Делим отрезок пополам. Если правый отрезок содержит бесконечное число членов последовательности, то следующим отрезком будет . В противном случае выбираем левый отрезок . Из отрезка выбираем первый член подпоследовательности .

Затем делим отрезок пополам. Если в правой половине бесконечное число членов последовательности, то выбираем ее. В противном случае выбираем левую половину. Получаем отрезок . Из него выбираем второй член подпоследовательности с номером . И так далее.

В результате получаем систему вложенных отрезков

и подпоследовательность . Поскольку длины отрезков стремятся к нулю, то согласно лемме о вложенных отрезках, существует единственная точка , принадлежащая всем отрезкам. Поскольку , то .

Поскольку мы выбирали самые правые отрезки с бесконечным числом членов, то точка c является верхним частичным пределом последовательности.

Аналогичным способом можно доказать, что у последовательности существует нижний частичный предел. Для этого сначала рассматриваем последовательность, неограниченную снизу. В конце рассматриваем последовательность, ограниченную снизу и имеющую бесконечное число членов в отрезке . Только здесь, при делении отрезков, мы выбираем левый отрезок, если он содержит бесконечное число членов последовательности.

Доказательство свойства верхнего и нижнего частичных пределов

Пусть точка a является верхним (нижним) частичным пределом последовательности . Тогда, согласно свойству 4 ⇑, в любой окрестности этой точки, в том числе и в интервале , содержится бесконечное число членов последовательности.

Докажем, что в полуинтервале содержится конечное число членов последовательности. Допустим противное, что в этом полуинтервале содержится бесконечное число членов. Тогда по теореме Больцано – Вейерштрасса ⇑ из них можно выделить сходящуюся подпоследовательность. Согласно свойствам неравенств, предел b этой подпоследовательности удовлетворяет неравенству , то есть больше (меньше) a . Возникает противоречие, поскольку a является верхним (нижним) частичным пределом последовательности.

Доказательство теоремы о неравенстве между верхним и нижним частичными пределами

Пусть последовательность сходится к числу a : .
Тогда согласно свойству 1 ⇑?, любая ее подпоследовательность сходится к этому же числу. Поэтому .

Пусть . И пусть a – конечное число. Согласно свойству 6 ⇑, для любого , интервалу не принадлежат только конечное число членов последовательности. Тогда согласно свойству (*) ⇑,
.

Пусть . Тогда, для любого конечного числа M , неравенство выполняется только для конечного числа членов . Отсюда .
Пусть . Тогда неравенство выполняется только для конечного числа членов . Поэтому .

Доказательство свойств верхних и нижних пределов суммы последовательностей

Докажем, что .
Из последовательности выберем подпоследовательность , сходящуюся к ее верхнему частичному пределу:
.
Из выберем сходящуюся подпоследовательность . Далее из выберем сходящуюся подпоследовательность .

Тогда последовательность является подпоследовательностью по отношению к . Согласно свойству 1 ⇑, их пределы равны:
.
Также и последовательность является подпоследовательностью по отношению к . Поэтому она сходится.

Докажем второе неравенство:
.
Умножим первое неравенство на – 1 :
.
Применим свойство 8 ⇑:
.

Доказательство свойства верхних пределов произведения последовательностей

Из последовательности выберем подпоследовательность , сходящуюся к ее верхнему частичному пределу:
.
По условию, последовательность сходится к числу a . Тогда и ее подпоследовательность , согласно свойству 1 ⇑, также сходится к числу a . По свойству предела произведения последовательностей, последовательность сходится и
.
Поскольку , то
(10.1) .

Аналогично предыдущему, из последовательности выберем подпоследовательность , сходящуюся к ее верхнему частичному пределу:
.
Из выберем сходящуюся подпоследовательность . Тогда
;
(10.2) .

Из (10.1) и (10.2) следует, что
.
Свойство доказано.

Использованная литература:
С.М. Никольский. Курс математического анализа. Том 1. Москва, 1983.

Автор: Олег Одинцов . Опубликовано: 21-12-2017

5.1 Последовательности — Исчисление Том 2

Цели обучения

  • 5.1.1 Найдите формулу для общего члена последовательности.
  • 5.1.2 Вычислить предел последовательности, если он существует.
  • 5.1.3 Определение сходимости или расхождения заданной последовательности.

В этом разделе мы вводим последовательности и определяем, что значит сходиться или расходиться последовательность. Мы показываем, как найти пределы сходящихся последовательностей, часто используя свойства пределов для функций, которые обсуждались ранее.Мы завершаем этот раздел теоремой о монотонной сходимости, инструментом, который мы можем использовать для доказательства сходимости определенных типов последовательностей.

Терминология последовательностей

Чтобы работать с этой новой темой, нам нужны новые термины и определения. Во-первых, бесконечная последовательность — это упорядоченный список чисел вида

. a1, a2, a3,…, an,… .a1, a2, a3,…, an,….

Каждое из чисел в последовательности называется термином. Символ nn называется индексной переменной для последовательности. Мы используем обозначение

{an} n = 1∞, или просто {an}, {an} n = 1∞, или просто {an},

для обозначения этой последовательности.Аналогичное обозначение используется для наборов, но последовательность — это упорядоченный список, а набор — неупорядоченный. Поскольку конкретное число anan существует для каждого положительного целого числа n, n, мы также можем определить последовательность как функцию, домен которой является набором положительных целых чисел.

Рассмотрим бесконечный упорядоченный список

2,4,8,16,32,… .2,4,8,16,32,….

Это последовательность, в которой первый, второй и третий члены задаются формулами a1 = 2, a1 = 2, a2 ​​= 4, a2 = 4 и a3 = 8. a3 = 8. Вы, вероятно, видите, что термины в этой последовательности имеют следующий шаблон:

a1 = 21, a2 = 22, a3 = 23, a4 = 24 и a5 = 25.a1 = 21, a2 = 22, a3 = 23, a4 = 24 и a5 = 25.

Предполагая, что этот шаблон продолжается, мы можем записать n-й член в последовательности по явной формуле an = 2n.an = 2n. Используя это обозначение, мы можем записать эту последовательность как

{2n} n = 1∞или {2n}. {2n} n = 1∞или {2n}.

В качестве альтернативы мы можем описать эту последовательность по-другому. Поскольку каждый член вдвое больше предыдущего, эту последовательность можно определить рекурсивно, выразив n-й член anan в терминах предыдущего члена an-1.an-1. В частности, мы можем определить эту последовательность как последовательность {an} {an}, где a1 = 2a1 = 2 и для всех n≥2, n≥2 каждый член anan определяется рекуррентным соотношениемan = 2an − 1.ан = 2ан − 1.

Определение

Бесконечная последовательность {an} {an} — это упорядоченный список чисел в форме

a1, a2,…, an,… .a1, a2,…, an,….

Нижний индекс nn называется индексной переменной последовательности. Каждое число анан является членом последовательности. Иногда последовательности определяются явными формулами, и в этом случае an = f (n) an = f (n) для некоторой функции f (n) f (n), определенной над положительными целыми числами. В других случаях последовательности определяются с помощью рекуррентного отношения. В рекуррентном отношении один член (или несколько) последовательности задается явно, а последующие термины определяются в терминах более ранних терминов в последовательности.

Обратите внимание, что индекс не обязательно должен начинаться с n = 1n = 1, но может начинаться с других целых чисел. Например, последовательность, заданная явной формулой an = f (n) an = f (n), может начинаться с n = 0, n = 0, и в этом случае последовательность будет

a0, a1, a2,… .a0, a1, a2,….

Аналогичным образом, для последовательности, определенной рекуррентным соотношением, член a0a0 может быть задан явно, а члены anan для n≥1n≥1 могут быть определены в терминах an − 1.an − 1. Поскольку последовательность {an} {an} имеет ровно одно значение для каждого положительного целого числа n, n, ее можно описать как функцию, домен которой является набором положительных целых чисел.В результате имеет смысл обсудить график последовательности. График последовательности {an} {an} состоит из всех точек (n, an) (n, an) для всех натуральных чисел n.n. На рисунке 5.2 показан график {2n}. {2n}.

Рисунок 5.2 Построенные точки представляют собой график последовательности {2n}. {2n}.

Часто встречаются два типа последовательностей, которым даны специальные имена: арифметические последовательности и геометрические последовательности. В арифметической последовательности разница между каждой парой последовательных членов одинакова.Например, рассмотрим последовательность

3,7,11,15,19,… .3,7,11,15,19,….

Вы можете видеть, что разница между каждой последовательной парой терминов составляет 4,4. Если предположить, что этот шаблон продолжается, эта последовательность является арифметической последовательностью. Его можно описать с помощью рекуррентного соотношения

{a1 = 3an = an − 1 + 4forn≥2. {a1 = 3an = an − 1 + 4forn≥2.

Обратите внимание, что

a2 = 3 + 4a3 = 3 + 4 + 4 = 3 + 2 · 4a4 = 3 + 4 + 4 + 4 = 3 + 3 · 4. a2 = 3 + 4a3 = 3 + 4 + 4 = 3 + 2 · 4a4 = 3 + 4 + 4 + 4 = 3 + 3 · 4.

Таким образом, последовательность также может быть описана с помощью явной формулы

ан = 3 + 4 (n − 1) = 4n − 1.ан = 3 + 4 (n − 1) = 4n − 1.

В общем, арифметическая последовательность — это любая последовательность вида an = cn + b.an = cn + b.

В геометрической последовательности соотношение каждой пары следующих друг за другом членов одинаково. Например, рассмотрим последовательность

2, -23,29, -227,281,… .2, -23,29, -227,281,….

Мы видим, что отношение любого члена к предыдущему члену равно -13.-13. Если предположить, что этот узор продолжается, эта последовательность представляет собой геометрическую последовательность. Его можно рекурсивно определить как

а1 = 2ан = -13 · ан-1forn≥2.а1 = 2ан = -13 · ан-1forn≥2.

Альтернативно, с

a2 = −13 · 2a3 = (- 13) (- 13) (2) = (- 13) 2 · 2a4 = (- 13) (- 13) (- 13) (2) = (- 13) 3 · 2 , a2 = −13 · 2a3 = (- 13) (- 13) (2) = (- 13) 2 · 2a4 = (- 13) (- 13) (- 13) (2) = (- 13) 3 · 2,

мы видим, что последовательность может быть описана с помощью явной формулы

an = 2 (−13) n−1.an = 2 (−13) n − 1.

Последовательность {2n} {2n}, которую мы обсуждали ранее, представляет собой геометрическую последовательность, где отношение любого члена к предыдущему члену составляет 2,2. В общем, геометрическая последовательность — это любая последовательность вида an = crn.an = crn.

Пример 5.1

Поиск явных формул

Найдите явную формулу для n-го члена последовательности для каждой из следующих последовательностей.

  1. −12,23, −34,45, −56,… −12,23, −34,45, −56,…
  2. 34,97,2710,8113,24316,… 34,97,2710,8113,24316,…
Решение
  1. Во-первых, обратите внимание, что последовательность чередуется с отрицательной на положительную. Нечетные члены в последовательности отрицательны, а четные — положительны.Следовательно, n-й член включает множитель (−1) n. (- 1) n. Затем рассмотрим последовательность числителей {1,2,3,…} {1,2,3,…} и последовательность знаменателей {2,3,4,…}. {2,3,4,…}. Мы видим, что обе эти последовательности являются арифметическими последовательностями. N-й член в последовательности числителей равен n, n, а n-й член в последовательности знаменателей равен n + 1.n + 1. Следовательно, последовательность может быть описана явной формулой
    an = (- 1) nnn + 1.an = (- 1) nnn + 1.
  2. Последовательность числителей 3,9,27,81,243,… 3,9,27,81,243,… представляет собой геометрическую последовательность.Числитель n-го члена равен 3n3n. Последовательность знаменателей 4,7,10,13,16,… 4,7,10,13,16,… является арифметической последовательностью. Знаменатель n-го члена равен 4 + 3 (n − 1) = 3n + 1,4 + 3 (n − 1) = 3n + 1. Следовательно, мы можем описать последовательность явной формулой an = 3n3n + 1.an = 3n3n + 1.

КПП 5.1

Найдите явную формулу для n-го члена последовательности {15, −17,19, −111,…}. {15, −17,19, −111,…}.

Пример 5.2

Определяется отношениями повторения

Найдите явную формулу для каждой из следующих рекурсивно определенных последовательностей.

  1. a1 = 2, a1 = 2, an = −3an − 1an = −3an − 1 для n≥2n≥2
  2. a1 = 12, a1 = 12, an = an − 1 + (12) nan = an − 1 + (12) n для n≥2n≥2
Решение
  1. Выписывая первые несколько терминов, получаем
    a1 = 2a2 = −3a1 = −3 (2) a3 = −3a2 = (- 3) 22a4 = −3a3 = (- 3) 32.a1 = 2a2 = −3a1 = −3 (2) a3 = −3a2 = ( −3) 22a4 = −3a3 = (- 3) 32.
    В целом
    an = 2 (−3) n−1.an = 2 (−3) n − 1.
  2. Запишите несколько первых терминов:
    a1 = 12a2 = a1 + (12) 2 = 12 + 14 = 34a3 = a2 + (12) 3 = 34 + 18 = 78a4 = a3 + (12) 4 = 78 + 116 = 1516.a1 = 12a2 = a1 + (12) 2 = 12 + 14 = 34a3 = a2 + (12) 3 = 34 + 18 = 78a4 = a3 + (12) 4 = 78 + 116 = 1516.
    Из этого шаблона мы выводим явную формулу
    an = 2n − 12n = 1−12n.an = 2n − 12n = 1−12n.

КПП 5.2

Найдите явную формулу для последовательности, определенной рекурсивно, такой что a1 = −4a1 = −4 и an = an − 1 + 6.an = an − 1 + 6.

Предел последовательности

Фундаментальный вопрос, который возникает относительно бесконечных последовательностей, — это поведение членов при увеличении nn. Поскольку последовательность — это функция, определенная на натуральных числах, имеет смысл обсудить предел членов при n → ∞.п → ∞. Например, рассмотрим следующие четыре последовательности и их различное поведение при n → ∞n → ∞ (см. Рисунок 5.3):

  1. {1 + 3n} = {4,7,10,13,…}. {1 + 3n} = {4,7,10,13,…}. Члены 1 + 3n1 + 3n становятся сколь угодно большими при n → ∞.n → ∞. В этом случае мы говорим, что 1 + 3n → ∞1 + 3n → ∞ при n → ∞.n → ∞.
  2. {1− (12) n} = {12,34,78,1516,…}. {1− (12) n} = {12,34,78,1516,…}. Слагаемые 1− (12) n → 11− (12) n → 1 при n → ∞.n → ∞.
  3. {(−1) n} = {- 1,1, −1,1,…}. {(- 1) n} = {- 1,1, −1,1,…}. Члены чередуются, но не приближаются к одному значению при n → ∞.п → ∞.
  4. {(−1) nn} = {- 1,12, −13,14,…}. {(- 1) nn} = {- 1,12, −13,14,…}. Члены этой последовательности также чередуются, но (−1) nn → 0 (−1) nn → 0 при n → ∞.n → ∞.
Рисунок 5.3 (a) Члены последовательности становятся сколь угодно большими при n → ∞.n → ∞. (b) Члены последовательности приближаются к 11 при n → ∞.n → ∞. (c) Члены в последовательности чередуются от 11 до −1−1 при n → ∞.n → ∞. (d) Члены в последовательности чередуются между положительными и отрицательными значениями, но приближаются к 00 при n → ∞.n → ∞.

Из этих примеров мы видим несколько возможностей поведения членов последовательности при n → ∞.п → ∞. В двух последовательностях члены стремятся к конечному числу при n → ∞.n → ∞. В двух других последовательностях термины нет. Если члены последовательности приближаются к конечному числу LL при n → ∞, n → ∞, мы говорим, что последовательность является сходящейся последовательностью, а действительное число LL является пределом последовательности. Мы можем дать здесь неформальное определение.

Определение

Для данной последовательности {an}, {an}, если члены anan становятся сколь угодно близкими к конечному числу LL, когда nn становится достаточно большим, мы говорим, что {an} {an} — сходящаяся последовательность, а LL — предел последовательности .В этом случае пишем

limn → ∞an = L. limn → ∞an = L.

Если последовательность {an} {an} не сходится, мы говорим, что это расходящаяся последовательность.

Из рисунка 5.3 мы видим, что члены в последовательности {1− (12) n} {1− (12) n} становятся произвольно близкими к 11, когда nn становится очень большим. Мы заключаем, что {1− (12) n} {1− (12) n} — сходящаяся последовательность и ее предел равен 1.1. Напротив, из рисунка 5.3 мы видим, что члены в последовательности 1 + 3n1 + 3n не приближаются к конечному числу по мере того, как nn становится больше.Мы говорим, что {1 + 3n} {1 + 3n} расходящаяся последовательность.

В неформальном определении предела последовательности мы использовали термины «произвольно близкие» и «достаточно большие». Хотя эти фразы помогают проиллюстрировать значение сходящейся последовательности, они несколько расплывчаты. Чтобы быть более точным, мы теперь представляем более формальное определение предела для последовательности и графически показываем эти идеи на рисунке 5.4.

Определение

Последовательность {an} {an} сходится к действительному числу LL, если для всех ε> 0, ε> 0 существует целое число NN такое, что | an − L | <ε | an − L | <ε, если n≥ Н.n≥N. Число LL является пределом последовательности, и мы пишем

limn → ∞an = Лоран → L.limn → ∞an = Loran → L.

В этом случае мы говорим, что последовательность {an} {an} является сходящейся последовательностью. Если последовательность не сходится, это расходящаяся последовательность, и мы говорим, что предела не существует.

Заметим, что сходимость или расходимость последовательности {an} {an} зависит только от того, что происходит с членами anan при n → ∞.n → ∞. Следовательно, если конечное число членов b1, b2,…, bNb1, b2,…, bN помещается перед a1a1 для создания новой последовательности

b1, b2,…, bN, a1, a2,…, b1, b2,…, bN, a1, a2,…,

эта новая последовательность будет сходиться, если {an} {an} сходится, и расходиться, если {an} {an} расходится.Кроме того, если последовательность {an} {an} сходится к L, L, эта новая последовательность также сходится к L.L.

Рисунок 5.4 По мере увеличения nn члены anan становятся ближе к LL Для значений n≥N, n≥N расстояние между каждой точкой (n, an) (n, an) и линией y = Ly = L меньше, чем ε.ε.

Как определено выше, если последовательность не сходится, она называется расходящейся последовательностью. Например, последовательности {1 + 3n} {1 + 3n} и {(−1) n} {(- 1) n}, показанные на рисунке 5.4, расходятся. Однако разные последовательности могут отличаться по-разному.Последовательность {(−1) n} {(- 1) n} расходится, потому что члены чередуются между 11 и −1, −1, но не приближаются к одному значению при n → ∞.n → ∞. С другой стороны, последовательность {1 + 3n} {1 + 3n} расходится, поскольку слагаемые 1 + 3n → ∞1 + 3n → ∞ при n → ∞.n → ∞. Скажем, что последовательность {1 + 3n} {1 + 3n} расходится на бесконечность, и запишем limn → ∞ (1 + 3n) = ∞.limn → ∞ (1 + 3n) = ∞. Важно понимать, что это обозначение не означает, что существует предел последовательности {1 + 3n} {1 + 3n}. На самом деле последовательность расходится. Запись о том, что предел равен бесконечности, предназначена только для предоставления дополнительной информации о том, почему последовательность расходится.Последовательность также может расходиться до отрицательной бесконечности. Например, последовательность {−5n + 2} {- 5n + 2} расходится до отрицательной бесконечности, поскольку −5n + 2 → −∞ − 5n + 2 → −∞ при n → −∞.n → −∞. Запишем это как limn → ∞ (−5n + 2) = → −∞.limn → ∞ (−5n + 2) = → −∞.

Поскольку последовательность — это функция, домен которой является набором положительных целых чисел, мы можем использовать свойства пределов функций, чтобы определить, сходится ли последовательность. Например, рассмотрим последовательность {an} {an} и связанную функцию ff, определенную для всех положительных действительных чисел, таких что f (n) = anf (n) = an для всех целых чисел n≥1.n≥1. Поскольку область определения последовательности является подмножеством области определения f, f, если существует limx → ∞f (x) limx → ∞f (x), то последовательность сходится и имеет тот же предел. Например, рассмотрим последовательность {1n} {1n} и связанную с ней функцию f (x) = 1x.f (x) = 1x. Поскольку функция ff, определенная на всех действительных числах x> 0x> 0, удовлетворяет условию f (x) = 1x → 0f (x) = 1x → 0 при x → ∞, x → ∞, последовательность {1n} {1n} должна удовлетворять 1n → 01n → 0 при n → ∞.n → ∞.

Теорема 5.1

Предел последовательности, определяемой функцией

Рассмотрим последовательность {an} {an} такую, что an = f (n) an = f (n) для всех n≥1.n≥1. Если существует действительное число LL такое, что

limx → ∞f (x) = L, limx → ∞f (x) = L,

затем сходится {an} {an} и

limn → ∞an = L. limn → ∞an = L.

Мы можем использовать эту теорему для вычисления limn → ∞rnlimn → ∞rn для 0≤r≤1.0≤r≤1. Например, рассмотрим последовательность {(1/2) n} {(1/2) n} и связанную с ней экспоненциальную функцию f (x) = (1/2) x.f (x) = (1/2) x. Поскольку limx → ∞ (1/2) x = 0, limx → ∞ (1/2) x = 0, заключаем, что последовательность {(1/2) n} {(1/2) n} сходится и ее предел составляет 0,0. Аналогично, для любого действительного числа rr такого, что 0≤r <1,0≤r <1, ​​limx → ∞rx = 0, limx → ∞rx = 0, и, следовательно, последовательность {rn} {rn} сходится.С другой стороны, если r = 1, r = 1, то limx → ∞rx = 1, limx → ∞rx = 1, и поэтому предел последовательности {1n} {1n} равен 1.1. Если r> 1, r> 1, limx → ∞rx = ∞, limx → ∞rx = ∞, и поэтому мы не можем применить эту теорему. Однако в этом случае, так же как функция rxrx неограниченно растет при n → ∞, n → ∞, члены rnrn в последовательности становятся сколь угодно большими при n → ∞, n → ∞, и мы заключаем, что последовательность {rn} {rn} расходится до бесконечности, если r> 1.r> 1.

Мы суммируем эти результаты относительно геометрической последовательности {rn}: {rn}:

rn → 0, если 0 1.rn → 0, если 0 1.

Далее в этом разделе мы рассмотрим случай, когда r <0.r <0.

Теперь рассмотрим несколько более сложные последовательности. Например, рассмотрим последовательность {(2/3) n + (1/4) n}. {(2/3) n + (1/4) n}. Члены в этой последовательности более сложные, чем в других последовательностях, которые мы обсуждали, но, к счастью, предел этой последовательности определяется пределами двух последовательностей {(2/3) n} {(2/3) n} и {( 1/4) n}. {(1/4) n}. Как мы описываем в следующих алгебраических предельных законах, поскольку {(2/3) n} {(2/3) n} и {1/4) n} {1/4) n} сходятся к 0,0, последовательность {(2/3) n + (1/4) n} {(2/3) n + (1/4) n} сходится к 0 + 0 = 0.0 + 0 = 0. Так же, как мы смогли оценить предел, включающий алгебраическую комбинацию функций ff и gg, глядя на пределы ff и gg (см. Введение в пределы), мы можем оценить предел последовательности, члены которой являются алгебраическими комбинациями функций anan и bnbn, оценив пределы {an} {an} и {bn}. {bn}.

Теорема 5.2

Алгебраические предельные законы

Для заданных последовательностей {an} {an} и {bn} {bn} и любого действительного числа c, c, если существуют такие константы AA и BB, что limn → ∞an = Alimn → ∞ an = A и limn → ∞bn = B, limn → ∞bn = B, тогда

  1. limn → ∞c = набег → ∞c = c
  2. limn → ∞can = climn → ∞an = cAlimn → ∞can = climn → ∞an = cA
  3. limn → ∞ (an ± bn) = limn → ∞an ± limn → ∞bn = A ± Blimn → ∞ (an ± bn) = limn → ∞an ± limn → ∞bn = A ± B
  4. limn → ∞ (an · bn) = (limn → ∞an) · (limn → ∞bn) = A · Blimn → ∞ (an · bn) = (limn → ∞an) · (limn → ∞bn) = A · B
  5. limn → ∞ (anbn) = limn → ∞anlimn → ∞bn = AB, limn → ∞ (anbn) = limn → ∞anlimn → ∞bn = AB, если B ≠ 0B ≠ 0 и каждое bn ≠ 0.bn ≠ 0.
Проба

Докажем часть iii.

Пусть ϵ> 0.ϵ> 0. Поскольку limn → ∞an = A, limn → ∞an = A, существует постоянное натуральное число N1N1 такое, что | ан-А | <ε2 | ан-А | <ε2 для всех n≥N1.n≥N1. Поскольку limn → ∞bn = B, limn → ∞bn = B, существует постоянная N2N2 такая, что | bn − B | <ε / 2 | bn − B | <ε / 2 для всех n≥N2.n≥N2. Пусть NN будет большим из N1N1 и N2.N2. Следовательно, для всех n≥N, n≥N

| (an + bn) — (A + B) | ≤ | an − A | + | bn − B | <ε2 + ε2 = ε. | (An + bn) - (A + B) | ≤ | an− А | + | bn − B | <ε2 + ε2 = ε.

Алгебраические предельные законы позволяют нам оценивать пределы для многих последовательностей. Например, рассмотрим последовательность {1n2}. {1n2}. Как было показано ранее, limn → ∞1 / n = 0. limn → ∞1 / n = 0. Аналогично, для любого положительного целого числа k, k мы можем заключить, что

limn → ∞1nk = 0. limn → ∞1nk = 0.

В следующем примере мы используем этот факт вместе с законами пределов для оценки пределов для других последовательностей.

Пример 5.3

Определение сходимости и определение пределов

Для каждой из следующих последовательностей определите, сходится ли последовательность.Если он сходится, найдите его предел.

  1. {5−3n2} {5−3n2}
  2. {3n4−7n2 + 56−4n4} {3n4−7n2 + 56−4n4}
  3. {2nn2} {2nn2}
  4. {(1 + 4n) n} {(1 + 4n) n}
Решение
  1. Мы знаем, что 1 / n → 0,1 / n → 0. Используя этот факт, заключаем, что
    limn → ∞1n2 = limn → ∞ (1n) .limn → ∞ (1n) = 0. limn → ∞1n2 = limn → ∞ (1n) .limn → ∞ (1n) = 0.
    Следовательно,
    limn → ∞ (5−3n2) = limn → ∞5−3limn → ∞1n2 = 5−3 · 0 = 5.limn → ∞ (5−3n2) = limn → ∞5−3limn → ∞1n2 = 5−3 · 0 = 5.
    Последовательность сходится и ее предел равен 5.5.
  2. Вынося n4n4 из числителя и знаменателя и используя приведенные выше законы пределов, получаем
    limn → ∞3n4−7n2 + 56−4n4 = limn → ∞3−7n2 + 5n46n4−4 = limn → ∞ (3−7n2 + 5n4) limn → ∞ (6n4−4) = (limn → ∞ (3) −limn → ∞7n2 + limn → ∞5n4) (limn → ∞6n4 − limn → ∞ (4)) = (limn → ∞ (3) −7 · limn → ∞1n2 + 5 · limn → ∞1n4) (6 · limn → ∞1n4 − limn → ∞ (4)) = 3−7 · 0 + 5 · 06 · 0−4 = −34.limn → ∞3n4−7n2 + 56−4n4 = limn → ∞3−7n2 + 5n46n4−4 = limn → ∞ (3−7n2 + 5n4) limn → ∞ (6n4−4) = (limn → ∞ (3) −limn → ∞7n2 + limn → ∞5n4) (limn → ∞6n4 − limn → ∞ (4)) = (limn → ∞ (3) −7 · limn → ∞1n2 + 5 · limn → ∞1n4) (6 · limn → ∞1n4 − limn → ∞ (4)) = 3−7 · 0 + 5 · 06 · 0 −4 = −34.
    Последовательность сходится, и ее предел равен −3 / 4. − 3/4.
  3. Рассмотрим связанную функцию f (x) = 2x / x2f (x) = 2x / x2, определенную для всех действительных чисел x> 0.x> 0. Поскольку 2x → ∞2x → ∞ и x2 → ∞x2 → ∞ при x → ∞, x → ∞, примените правило Л’Опиталя и запишите
    limx → ∞2xx2 = limx → ∞2xln22x Возьмите производные числителя и знаменателя. = limx → ∞2x (ln2) 22 Возьмите производные снова. = ∞.limx → ∞2xx2 = limx → ∞2xln22x Возьмите производные числителя и знаменателя. = limx → ∞2x (ln2) 22 Снова возьмем производные. = ∞.
    Делаем вывод, что последовательность расходится.
  4. Рассмотрим функцию f (x) = (1 + 4x) xf (x) = (1 + 4x) x, определенную для всех действительных чисел x> 0.x> 0. Эта функция имеет неопределенный вид 1∞1∞ при x → ∞.x → ∞. Пусть
    y = limx → ∞ (1 + 4x) x.y = limx → ∞ (1 + 4x) x.
    Теперь, взяв натуральный логарифм обеих частей уравнения, получим
    ln (y) = ln [limx → ∞ (1 + 4x) x]. ln (y) = ln [limx → ∞ (1 + 4x) x].
    Поскольку функция f (x) = lnxf (x) = lnx непрерывна в своей области определения, мы можем поменять местами предел и натуральный логарифм. Следовательно,
    ln (y) = limx → ∞ [ln (1 + 4x) x].ln (y) = limx → ∞ [ln (1 + 4x) x].
    Используя свойства логарифмов, запишем
    limx → ∞ [ln (1 + 4x) x] = limx → ∞xln (1 + 4x) .limx → ∞ [ln (1 + 4x) x] = limx → ∞xln (1 + 4x).
    Поскольку правая часть этого уравнения имеет неопределенный вид ∞ · 0, ∞ · 0, перепишите ее как дробь, чтобы применить правило Л’Опиталя. Написать
    limx → ∞xln (1 + 4x) = limx → ∞ln (1 + 4 / x) 1 / x.limx → ∞xln (1 + 4x) = limx → ∞ln (1 + 4 / x) 1 / x.
    Поскольку правая часть теперь имеет неопределенную форму 0 / 0,0 / 0, мы можем применить правило Л’Опиталя. Делаем вывод, что
    limx → ∞ln (1 + 4 / x) 1 / x = limx → ∞41 + 4 / x = 4.limx → ∞ln (1 + 4 / x) 1 / x = limx → ∞41 + 4 / x = 4.
    Следовательно, ln (y) = 4ln (y) = 4 и y = e4.y = e4. Следовательно, поскольку limx → ∞ (1 + 4x) x = e4, limx → ∞ (1 + 4x) x = e4, мы можем заключить, что последовательность {(1 + 4n) n} {(1 + 4n) n} сходится на e4.e4.

КПП 5.3

Рассмотрим последовательность {(5n2 + 1) / en}. {(5n2 + 1) / en}. Определите, сходится ли последовательность. Если он сходится, найдите его предел.

Напомним, что если ff — непрерывная функция при значении L, L, то f (x) → f (L) f (x) → f (L) при x → L.х → L. Эта идея применима и к последовательностям. Предположим, что последовательность an → L, an → L и функция ff непрерывна в L.L. Тогда f (an) → f (L) .f (an) → f (L). Это свойство часто позволяет нам находить пределы для сложных последовательностей. Например, рассмотрим последовательность 5−3n2,5−3n2. Из Примера 5.3а. мы знаем последовательность 5−3n2 → 5.5−3n2 → 5. Поскольку xx — непрерывная функция при x = 5, x = 5,

limn → ∞5−3n2 = limn → ∞ (5−3n2) = 5. limn → ∞5−3n2 = limn → ∞ (5−3n2) = 5.

Теорема 5.3

Непрерывные функции, определенные на сходящихся последовательностях

Рассмотрим последовательность {an} {an} и предположим, что существует действительное число LL такое, что последовательность {an} {an} сходится к L.L. Предположим, что ff — непрерывная функция в LL. Тогда существует целое число NN такое, что ff определено при всех значениях anan для n≥N, n≥N, и последовательность {f (an)} {f (an)} сходится к f (L) f (L) (рисунок 5.5).

Проба

Пусть ϵ> 0.ϵ> 0. Поскольку ff непрерывно в L, L, существует такое δ> 0δ> 0, что | f (x) −f (L) | <ε | f (x) −f (L) | <ε, если | x − L | <б. | х - L | <б. Поскольку последовательность {an} {an} сходится к L, L, существует такое NN, что | an − L | <δ | an − L | <δ для всех n≥N.n≥N. Следовательно, для всех n≥N, n≥N, | an − L | <δ, | an − L | <δ, откуда следует | f (an) −f (L) | <ε.| f (an) - f (L) | <ε. Мы заключаем, что последовательность {f (an)} {f (an)} сходится к f (L) .f (L).

Рисунок 5.5 Поскольку ff является непрерывной функцией, поскольку входы a1, a2, a3,… a1, a2, a3,… приближаются к L, L, выходы f (a1), f (a2), f (a3),… f ( a1), f (a2), f (a3),… приблизиться к f (L) .f (L).

Пример 5.4

Пределы, включающие непрерывные функции, определенные на сходящихся последовательностях

Определите, сходится ли последовательность {cos (3 / n2)} {cos (3 / n2)}. Если он сходится, найдите его предел.

Решение

Поскольку последовательность {3 / n2} {3 / n2} сходится к 00 и cosxcosx непрерывна при x = 0, x = 0, мы можем заключить, что последовательность {cos (3 / n2)} {cos (3 / n2 )} сходится и

limn → ∞cos (3n2) = cos (0) = 1.limn → ∞cos (3n2) = cos (0) = 1.

КПП 5.4

Определите, сходится ли последовательность {2n + 13n + 5} {2n + 13n + 5}. Если он сходится, найдите его предел.

Другая теорема, касающаяся пределов последовательностей, является расширением теоремы сжатия для пределов, обсуждаемых во введении в пределы.

Теорема 5.4

Теорема сжатия для последовательностей

Рассмотрим последовательности {an}, {an}, {bn}, {bn} и {cn}. {Cn}. Предположим, что существует целое число NN такое, что

an≤bn≤cn для всехn≥N.an≤bn≤cn для всехn≥N.

Если существует действительное число LL такое, что

limn → ∞an = L = limn → ∞cn, limn → ∞an = L = limn → ∞cn,

, то {bn} {bn} сходится и limn → ∞bn = Llimn → ∞bn = L (рисунок 5.6).

Проба

Пусть ε> 0.ε> 0. Поскольку последовательность {an} {an} сходится к L, L, существует целое число N1N1 такое, что | an − L | <ε | an − L | <ε для всех n≥N1.n≥N1. Аналогично, поскольку {cn} {cn} сходится к L, L, существует целое число N2N2 такое, что | cn − L | <ε | cn − L | <ε для всех n≥N2.n≥N2. По предположению существует целое число NN такое, что an≤bn≤cnan≤bn≤cn для всех n≥N.n≥N. Пусть MM будет наибольшим из N1, N2, N1, N2 и N.N. Мы должны показать, что | bn − L | <ε | bn − L | <ε для всех n≥M.n≥M. Для всех n≥M, n≥M,

−ε <- | an − L | ≤an − L≤bn − L≤cn − L≤ | cn − L | <ε. − ε <- | an − L | ≤an − L≤bn − L≤cn− L≤ | cn − L | <ε.

Следовательно, −ε

Рисунок 5.6. Каждый член bnbn удовлетворяет условию an≤bn≤cnan≤bn≤cn, а последовательности {an} {an} и {cn} {cn} сходятся к одному пределу, поэтому последовательность {bn} {bn} должна сходиться к такой же предел.

Пример 5.5

Использование теоремы сжатия

Используйте теорему сжатия, чтобы найти предел каждой из следующих последовательностей.

  1. {cosnn2} {cosnn2}
  2. {(−12) n} {(- 12) n}
Решение
  1. Так как −1≤cosn≤1−1≤cosn≤1 для всех целых чисел n, n, имеем
    −1n2≤cosnn2≤1n2. − 1n2≤cosnn2≤1n2.
    Поскольку −1 / n2 → 0−1 / n2 → 0 и 1 / n2 → 0,1 / n2 → 0, мы заключаем, что cosn / n2 → 0cosn / n2 → 0.
  2. с
    −12n≤ (−12) n≤12n − 12n≤ (−12) n≤12n
    для всех натуральных чисел n, n, −1 / 2n → 0−1 / 2n → 0 и 1 / 2n → 0,1 / 2n → 0, мы можем заключить, что (−1/2) n → 0.(-1/2) п → 0.

КПП 5.5

Найдите limn → ∞2n − sinnn.limn → ∞2n − sinnn.

Используя идею из Примера 5.5b. заключаем, что rn → 0rn → 0 для любого действительного числа rr такого, что −1 rn → 0, если | r | <1 rn → 0, если | r | <1

(5.1)

rn → 1ifr = 1rn → 1ifr = 1

(5.2)

rn → ∞ifr> 1rn → ∞ifr> 1

(5,3)

{rn} расходится ifr≤ − 1 {rn} расходится ifr≤ − 1

(5.4)

Ограниченные последовательности

Теперь обратим наше внимание на одну из наиболее важных теорем, касающихся последовательностей: теорему о монотонной сходимости. Прежде чем сформулировать теорему, нам нужно ввести некоторую терминологию и мотивацию. Начнем с определения того, что означает ограниченность последовательности.

Определение

Последовательность {an} {an} ограничена сверху, если существует действительное число MM такое, что

для всех натуральных чисел n.п.

Последовательность {an} {an} ограничена снизу, если существует вещественное число MM такое, что

для всех натуральных чисел n.n.

Последовательность {an} {an} является ограниченной последовательностью, если она ограничена сверху и ограничена снизу.

Если последовательность не ограничена, это неограниченная последовательность.

Например, последовательность {1 / n} {1 / n} ограничена выше, потому что 1 / n≤11 / n≤1 для всех положительных целых чисел n.n. Он также ограничен снизу, потому что 1 / n≥01 / n≥0 для всех натуральных чисел n. Следовательно, {1 / n} {1 / n} — ограниченная последовательность. С другой стороны, рассмотрим последовательность {2n}. {2n}. Поскольку 2n≥22n≥2 для всех n≥1, n≥1, последовательность ограничена снизу. Однако последовательность не ограничена сверху. Следовательно, {2n} {2n} — неограниченная последовательность.

Теперь обсудим связь между ограниченностью и сходимостью. Предположим, что последовательность {an} {an} неограничена. Тогда он не ограничен сверху, не ограничен снизу или и тем, и другим. В любом случае есть члены anan, которые сколь угодно велики по величине с увеличением nn.В результате последовательность {an} {an} не может сходиться. Следовательно, ограниченность — необходимое условие сходимости последовательности.

Теорема 5.5

Сходящиеся последовательности ограничены

Если последовательность {an} {an} сходится, то она ограничена.

Обратите внимание, что ограниченность последовательности не является достаточным условием для сходимости последовательности. Например, последовательность {(−1) n} {(- 1) n} ограничена, но последовательность расходится, потому что последовательность колеблется между 11 и −1−1 и никогда не приближается к конечному числу.Теперь обсудим достаточное (но не необходимое) условие сходимости ограниченной последовательности.

Рассмотрим ограниченную последовательность {an}. {An}. Предположим, что последовательность {an} {an} возрастает. То есть a1≤a2≤a3… .a1≤a2≤a3…. Поскольку последовательность возрастает, члены не колеблются. Следовательно, есть две возможности. Последовательность могла расходиться до бесконечности, а могла сходиться. Однако, поскольку последовательность ограничена, она ограничена сверху и последовательность не может расходиться до бесконечности. Мы заключаем, что {an} {an} сходится.Например, рассмотрим последовательность

{12,23,34,45,…}. {12,23,34,45,…}.

Поскольку эта последовательность возрастает и ограничена сверху, она сходится. Далее рассмотрим последовательность

{2,0,3,0,4,0,1, −12, −13, −14,…}. {2,0,3,0,4,0,1, −12, −13, −14 ,…}.

Несмотря на то, что последовательность не увеличивается для всех значений n, n, мы видим, что −1/2 <−1/3 <−1/4 , в конечном итоге возрастает.Поскольку последовательность ограничена сверху, она сходится. Верно также и то, что если последовательность убывает (или со временем убывает) и ограничена снизу, она также сходится.

Определение

Последовательность {an} {an} возрастает для всех n≥n0n≥n0, если

an≤an + 1 для всех n≥n0.an≤an + 1 для всехn≥n0.

Последовательность {an} {an} убывает для всех n≥n0n≥n0, если

an≥an + 1 для всех n≥n0.an≥an + 1 для всехn≥n0.

Последовательность {an} {an} является монотонной последовательностью для всех n≥n0n≥n0, если она увеличивается для всех n≥n0n≥n0 или убывает для всех n≥n0.n≥n0.

Теперь у нас есть необходимые определения, чтобы сформулировать теорему о монотонной сходимости, которая дает достаточное условие сходимости последовательности.

Теорема 5.6

Теорема о монотонной сходимости

Если {an} {an} — ограниченная последовательность и существует натуральное число n0n0 такое, что {an} {an} монотонно для всех n≥n0, n≥n0, то { an} {an} сходится.

Доказательство этой теоремы выходит за рамки этого текста. Вместо этого мы предоставляем график, который интуитивно показывает, почему эта теорема имеет смысл (рисунок 5.7).

Рисунок 5.7 Поскольку последовательность {an} {an} возрастает и ограничена сверху, она должна сходиться.

В следующем примере мы показываем, как теорему о монотонной сходимости можно использовать для доказательства сходимости последовательности.

Пример 5.6

Использование теоремы о монотонной сходимости

Для каждой из следующих последовательностей используйте теорему о монотонной сходимости, чтобы показать, что последовательность сходится, и найдите ее предел.

  1. {4nn!} {4nn!}
  2. {an} {an} определяется рекурсивно, так что
    a1 = 2andan + 1 = an2 + 12an для всех n≥2.a1 = 2andan + 1 = an2 + 12an для всех n≥2.
Решение
  1. Выписывая первые несколько терминов, мы видим, что
    {4nn!} = {4,8,323,323,12815,…}. {4nn!} = {4,8,323,323,12815,…}.
    Сначала сроки увеличиваются. Однако после третьего срока сроки сокращаются. Фактически, члены уменьшаются для всех n≥3.n≥3. Мы можем показать это следующим образом.
    an + 1 = 4n + 1 (n + 1)! = 4n + 1 · 4nn! = 4n + 1 · an≤anifn≥3.an + 1 = 4n + 1 (n + 1)! = 4n + 1 · 4nn ! = 4n + 1 · an≤anifn≥3.
    Следовательно, последовательность убывает для всех n≥3.n≥3. Далее, последовательность ограничена снизу числом 00, поскольку 4n / n! ≥04n / n! ≥0 для всех натуральных чисел n.n. Следовательно, по теореме о монотонной сходимости последовательность сходится.
    Чтобы найти предел, воспользуемся тем, что последовательность сходится, и положим L = limn → ∞an.L = limn → ∞an. Обратите внимание на это важное наблюдение. Рассмотрим limn → ∞an + 1.limn → ∞an + 1. С
    г. {an + 1} = {a2, a3, a4,…}, {an + 1} = {a2, a3, a4,…}, единственная разница между последовательностями {an + 1} {an + 1} и {an} {an} состоит в том, что {an + 1} {an + 1} опускает первый член.Поскольку конечное число членов не влияет на сходимость последовательности,
    limn → ∞an + 1 = limn → ∞an = L. limn → ∞an + 1 = limn → ∞an = L.
    Объединив этот факт с уравнением

    и взяв предел обеих частей уравнения
    limn → ∞an + 1 = limn → ∞4n + 1an, limn → ∞an + 1 = limn → ∞4n + 1an,
    можно сделать вывод, что
  2. Выписка первых нескольких терминов,
    {2,54,4140,32813280,…}. {2,54,4140,32813280,…}.
    мы можем предположить, что последовательность убывает и ограничена снизу единицей.1. Чтобы показать, что последовательность ограничена снизу числом 1,1, мы можем показать, что
    an2 + 12an≥1.an2 + 12an≥1.
    Чтобы показать это, сначала перепишите
    an2 + 12an = an2 + 12an.an2 + 12an = an2 + 12an.
    Поскольку a1> 0a1> 0 и a2a2 определяется как сумма положительных членов, a2> 0.a2> 0. Аналогично все члены an> 0.an> 0. Следовательно,

    тогда и только тогда, когда

    Переписывая неравенство an2 + 1≥2anan2 + 1≥2an как an2−2an + 1≥0, an2−2an + 1≥0 и используя тот факт, что
    an2−2an + 1 = (an − 1) 2≥0an2−2an + 1 = (an − 1) 2≥0
    поскольку квадрат любого действительного числа неотрицателен, мы можем заключить, что
    an2 + 12an≥1.an2 + 12an≥1.
    Чтобы показать, что последовательность убывает, мы должны показать, что an + 1≤anan + 1≤an для всех n≥1.n≥1. Поскольку 1≤an2,1≤an2, то
    an2 + 1≤2an2.an2 + 1≤2an2.
    Разделив обе части на 2an, 2an, получаем
    an2 + 12an≤an.an2 + 12an≤an.
    Используя определение an + 1, an + 1, мы заключаем, что
    an + 1 = an2 + 12an≤an.an + 1 = an2 + 12an≤an.
    Поскольку {an} {an} ограничен снизу и убывает по теореме о монотонной сходимости, он сходится.
    Чтобы найти предел, пусть L = limn → ∞an.L = limn → ∞an. Тогда, используя рекуррентное соотношение и тот факт, что limn → ∞an = limn → ∞an + 1, limn → ∞an = limn → ∞an + 1, имеем
    limn → ∞an + 1 = limn → ∞ (an2 + 12an), limn → ∞an + 1 = limn → ∞ (an2 + 12an),
    и, следовательно,

    Умножая обе части этого уравнения на 2L, 2L, мы приходим к уравнению

    Решая это уравнение относительно L, L, мы заключаем, что L2 = 1, L2 = 1, откуда L = ± 1. L = ± 1. Поскольку все члены положительны, предел L = 1.L = 1.

КПП 5,6

Рассмотрим последовательность {an} {an}, определенную рекурсивно так, что a1 = 1, a1 = 1, an = an − 1/2.ан = ан − 1/2. Используйте теорему о монотонной сходимости, чтобы показать, что эта последовательность сходится, и найдите ее предел.

Студенческий проект

Числа Фибоначчи

Числа Фибоначчи рекурсивно определяются последовательностью {Fn} {Fn}, где F0 = 0, F0 = 0, F1 = 1F1 = 1 и для n≥2, n≥2,

Fn = Fn − 1 + Fn − 2. Fn = Fn − 1 + Fn − 2.

Здесь мы рассмотрим свойства чисел Фибоначчи.

  1. Запишите первые двадцать чисел Фибоначчи.
  2. Найдите замкнутую формулу для последовательности Фибоначчи, выполнив следующие шаги.
    1. Рассмотрим рекурсивно определенную последовательность {xn} {xn}, где xo = cxo = c и xn + 1 = axn.xn + 1 = axn. Покажите, что эту последовательность можно описать замкнутой формулой xn = canxn = can для всех n≥0.n≥0.
    2. Используя результат части а. в качестве мотивации ищите решение уравнения
      Fn = Fn − 1 + Fn − 2Fn = Fn − 1 + Fn − 2
      вида Fn = cλn.Fn = cλn. Определите, какие два значения λλ позволяют FnFn удовлетворять этому уравнению.
    3. Рассмотрим два решения из части b: λ1λ1 и λ2.λ2.Пусть Fn = c1λ1n + c2λ2n. Fn = c1λ1n + c2λ2n. Используйте начальные условия F0F0 и F1F1 для определения значений констант c1c1 и c2c2 и запишите замкнутую формулу Fn.Fn.
  3. Используйте ответ в 2 c. чтобы показать, что
    limn → ∞Fn + 1Fn = 1 + 52.limn → ∞Fn + 1Fn = 1 + 52.
    Число ϕ = (1 + 5) / 2ϕ = (1 + 5) / 2 известно как золотое сечение (рисунок 5.8 и рисунок 5.9).

    Рис. 5.8. Семена подсолнечника имеют спиралевидные узоры, изгибающиеся влево и вправо. Количество спиралей в каждом направлении всегда является числом Фибоначчи — всегда.(кредит: модификация работы Эсдраса Кальдерана, Wikimedia Commons)


    Рис. 5.9 Пропорция золотого сечения присутствует во многих известных образцах искусства и архитектуры. Древнегреческий храм, известный как Парфенон, был спроектирован с такими пропорциями, и соотношение снова проявляется во многих мелких деталях. (кредит: модификация работы TravelingOtter, Flickr)

Раздел 5.1 Упражнения

Найдите первые шесть членов каждой из следующих последовательностей, начиная с n = 1.п = 1.

1.

an = 1 + (- 1) nan = 1 + (- 1) n для n≥1n≥1

2.

an = n2−1an = n2−1 для n≥1n≥1

3.

a1 = 1a1 = 1 и an = an − 1 + nan = an − 1 + n для n≥2n≥2

4.

a1 = 1, a1 = 1, a2 = 1a2 = 1 и an + 2 = an + an + 1an + 2 = an + an + 1 для n≥1n≥1

5.

Найдите явную формулу для anan, где a1 = 1a1 = 1 и an = an − 1 + nan = an − 1 + n для n≥2.n≥2.

6.

Найдите формулу anan для n-го члена арифметической последовательности, первый член которой a1 = 1a1 = 1, такой, что an + 1 − an = 17an + 1 − an = 17 для n≥1.n≥1.

7.

Найдите формулу anan для n-го члена арифметической последовательности, первый член которой a1 = −3a1 = −3, такой, что an + 1 − an = 4an + 1 − an = 4 для n≥1.n≥1.

8.

Найдите формулу anan для n-го члена геометрической последовательности, первый член которой a1 = 1a1 = 1, такой, что an + 1an = 10an + 1an = 10 для n≥1.n≥1.

9.

Найдите формулу anan для n-го члена геометрической последовательности, первый член которой a1 = 3a1 = 3, такой, что an + 1an = 1 / 10an + 1an = 1/10 для n≥1.n≥1.

10.

Найдите явную формулу для n-го члена последовательности, первые несколько членов которой равны {0,3,8,15,24,35,48,63,80,99,…}.{0,3,8,15,24,35,48,63,80,99,…}. ( Подсказка: Сначала добавьте по одному к каждому члену.)

11.

Найдите явную формулу для n-го члена последовательности, удовлетворяющего условиям a1 = 0a1 = 0 и an = 2an − 1 + 1an = 2an − 1 + 1 для n≥2.n≥2.

Найдите формулу для общего члена каждой из следующих последовательностей.

12.

{1,0, −1,0,1,0, −1,0,…} {1,0, −1,0,1,0, −1,0,…} ( Подсказка: Найдите где sinxsinx принимает эти значения)

13.

{1, −1 / 3,1 / 5, −1 / 7,…} {1, −1 / 3,1 / 5, −1 / 7,…}

Найдите функцию f (n) f (n), которая идентифицирует n-й член anan следующих рекурсивно определенных последовательностей, как an = f (n).ан = f (n).

14.

a1 = 1a1 = 1 и an + 1 = −anan + 1 = −an для n≥1n≥1

15.

a1 = 2a1 = 2 и an + 1 = 2anan + 1 = 2an для n≥1n≥1

16.

a1 = 1a1 = 1 и an + 1 = (n + 1) anan + 1 = (n + 1) an для n≥1n≥1

17.

a1 = 2a1 = 2 и an + 1 = (n + 1) an / 2an + 1 = (n + 1) an / 2 для n≥1n≥1

18.

a1 = 1a1 = 1 и an + 1 = an / 2nan + 1 = an / 2n для n≥1n≥1

Постройте первые члены NN каждой последовательности. Укажите, указывает ли графическое свидетельство, что последовательность сходится или расходится.

19.

[T] a1 = 1, a1 = 1, a2 = 2, a2 ​​= 2, а для n≥2, n≥2, an = 12 (an − 1 + an − 2); an = 12 (an −1 + an − 2); N = 30N = 30

20.

[T] a1 = 1, a1 = 1, a2 = 2, a2 ​​= 2, a3 = 3a3 = 3 и для n≥4, n≥4, an = 13 (an − 1 + an − 2 + an −3), an = 13 (an − 1 + an − 2 + an − 3), N = 30N = 30

21.

[T] a1 = 1, a1 = 1, a2 = 2, a2 ​​= 2, и для n≥3, n≥3, an = an − 1an − 2; an = an − 1an − 2; N = 30N = 30

22.

[T] a1 = 1, a1 = 1, a2 = 2, a2 ​​= 2, a3 = 3, a3 = 3, а для n≥4, n≥4, an = an − 1an − 2an − 3; ан = ан-1ан-2ан-3; N = 30N = 30

Предположим, что limn → ∞an = 1, limn → ∞an = 1, limn → ∞bn = −1, limn → ∞bn = −1 и 0 <−bn 23.

limn → ∞ (3an − 4bn) limn → ∞ (3an − 4bn)

24.

limn → ∞ (12bn − 12an) limn → ∞ (12bn − 12an)

25.

limn → ∞an + bnan − bnlimn → ∞an + bnan − bn

26.

limn → ∞an − bnan + bnlimn → ∞an − bnan + bn

Найдите предел каждой из следующих последовательностей, используя, когда это необходимо, правило L’Hôpital.

28.

(п — 1) 2 (п + 1) 2 (п — 1) 2 (п + 1) 2

30.

n1 / nn1 / n ( Подсказка: n1 / n = e1nlnn) n1 / n = e1nlnn)

Для каждой из следующих последовательностей, для которых указаны n-е члены, укажите, является ли последовательность ограниченной и будет ли она в конечном итоге монотонной, возрастающей или убывающей.

36.

n − 1 / n, n − 1 / n, n≥3n≥3

38.

Определите, имеет ли последовательность, определенная следующим образом, предел. Если да, найдите предел.

a1 = 2, a1 = 2, a2 ​​= 22, a2 ​​= 22, a3 = 222a3 = 222 и т. Д.

39.

Определите, имеет ли последовательность, определенная следующим образом, предел. Если да, найдите предел.

a1 = 3, a1 = 3, an = 2an − 1, an = 2an − 1, n = 2,3,… .n = 2,3,….

Используйте теорему сжатия, чтобы найти предел каждой из следующих последовательностей.

41.

cos (1 / n) −11 / ncos (1 / n) −11 / n

43.

an = sinnsin (1 / n) an = sinnsin (1 / n)

Для следующих последовательностей нанесите первые 2525 членов последовательности и укажите, указывает ли графическое свидетельство, что последовательность сходится или расходится.

Определите предел последовательности или покажите, что последовательность расходится. Если он сходится, найдите его предел.

46.

an = tan − 1 (n2) an = tan − 1 (n2)

47.

an = (2n) 1 / n − n1 / nan = (2n) 1 / n − n1 / n

48.

an = ln (n2) ln (2n) an = ln (n2) ln (2n)

50.

an = ln (n + 2n2−3) an = ln (n + 2n2−3)

53.

AN = (N!) 2 (2n)! An = (N!) 2 (2n)!

Метод Ньютона направлен на аппроксимацию решения f (x) = 0f (x) = 0, которое начинается с начального приближения x0x0 и последовательно определяет последовательность xn + 1 = xn − f (xn) f ′ (xn) .xn + 1 = xn − f (xn) f ′ (xn). Для данного выбора ff и x0, x0 запишите формулу для xn + 1.xn + 1. Если последовательность кажется сходящейся, дайте точную формулу для решения x, x, затем определите предел xx с точностью до четырех знаков после запятой и наименьшее nn, такое, что xnxn согласовывается с xx до четырех знаков после запятой.

54.

[T] f (x) = x2−2, f (x) = x2−2, x0 = 1×0 = 1

55.

[T] f (x) = (x − 1) 2−2, f (x) = (x − 1) 2−2, x0 = 2×0 = 2

56.

[T] f (x) = ex − 2, f (x) = ex − 2, x0 = 1×0 = 1

57.

[T] f (x) = lnx − 1, f (x) = lnx − 1, x0 = 2×0 = 2

58.

[T] Предположим, вы начали с одного литра уксуса и несколько раз удалили 0,1 л, 0,1 л, заменили водой, перемешали и повторили.

  1. Найдите формулу для концентрации после nn шагов.
  2. Через сколько этапов смесь будет содержать менее 10% 10% уксуса?
59.

[T] В озере изначально содержится 2 000 000 особей рыбы. Предположим, что при отсутствии хищников или других причин выведения популяция рыб увеличивается на 6% 6% каждый месяц. Однако с учетом всех причин ежемесячно теряется 150–150 рыб.

  1. Объясните, почему популяция рыб через nn месяцев моделируется как Pn = 1.06Pn − 1−150Pn = 1.06Pn − 1−150 с P0 = 2000.P0 = 2000.
  2. Сколько рыбы будет в пруду через год?
60.

[T] Банковский счет приносит 5% 5% годовых с начислением ежемесячных процентов. Предположим, что 1000 долларов США изначально вносятся на счет, но эти 10 долларов США снимаются каждый месяц.

  1. Покажите, что сумма на счете через nn месяцев равна An = (1 + .05 / 12) An − 1−10; An = (1 + .05 / 12) An − 1−10; A0 = 1000.A0 = 1000.
  2. Сколько денег будет на счету через 11 лет?
  3. Сумма увеличивается или уменьшается?
  4. Предположим, что вместо 10 долларов каждый месяц снимается фиксированная сумма dd долларов.Найдите такое значение dd, чтобы сумма на счете после каждого месяца оставалась 1000 долларов США. 1000 долларов США.
  5. Что произойдет, если dd больше этой суммы?
61.

[T] Студент берет ссуду колледжа в размере 10 000 долл. США 10 000 долл. США с годовой процентной ставкой 6%, 6%, начисляемой ежемесячно.

  1. Если студент платит 100 долларов США в месяц, сколько он должен платить через 1212 месяцев?
  2. Через сколько месяцев кредит будет погашен?
62.

[T] Рассмотрим ряд, сочетающий геометрический рост и арифметическое уменьшение. Пусть a1 = 1.a1 = 1. Зафиксируем a> 1a> 1 и 0 63.

[T] Двоичное представление x = 0.b1b2b3 … x = 0.b1b2b3 … числа xx от 00 до 11 может быть определено следующим образом. Пусть b1 = 0b1 = 0, если x <1 / 2x <1/2, и b1 = 1b1 = 1, если 1 / 2≤x <1.1 / 2≤x <1. Пусть x1 = 2x − b1.x1 = 2x − b1. Пусть b2 = 0b2 = 0, если x1 <1 / 2x1 <1/2, и b2 = 1b2 = 1, если 1 / 2≤x <1.1 / 2≤x <1. Пусть x2 = 2x1 − b2x2 = 2x1 − b2 и, вообще говоря, xn = 2xn − 1 − bnxn = 2xn − 1 − bn и bn − 1 = 0bn − 1 = 0, если xn <1 / 2xn <1/2 и bn− 1 = 1bn − 1 = 1, если 1 / 2≤xn <1.1 / 2≤xn <1. Найдите двоичное разложение 1 / 3,1 / 3.

64.

[T] Чтобы найти приближение для π, π, положите a0 = 2 + 1, a0 = 2 + 1, a1 = 2 + a0, a1 = 2 + a0 и, как правило, an + 1 = 2 + ан.ан + 1 = 2 + ан. Наконец, положим pn = 3.2n2−an.pn = 3.2n2 − an. Найдите первые десять членов pnpn и сравните значения с π.π.

Для следующих двух упражнений предположим, что у вас есть доступ к компьютерной программе или Интернет-источнику, который может генерировать список нулей и единиц любой желаемой длины. Генераторы псевдослучайных чисел (ГПСЧ) играют важную роль в моделировании случайного шума в физических системах, создавая последовательности нулей и единиц, которые выглядят как результат многократного подбрасывания монеты. Один из простейших типов ГПСЧ рекурсивно определяет произвольную последовательность из NN целых чисел a1, a2,…, aNa1, a2,…, aN, фиксируя два специальных целых числа KK и MM и позволяя + 1an + 1 быть остатком после деления К.anK.an в M, M, затем создает битовую последовательность из нулей и единиц, у которых n-й член bnbn равен единице, если anan нечетен, и равен нулю, если anan четно. Если биты bnbn являются псевдослучайными, то поведение их среднего (b1 + b2 + ⋯ + bN) / N (b1 + b2 + ⋯ + bN) / N должно быть аналогично поведению средних значений действительно случайно сгенерированных битов.

65.

[T] Начиная с K = 16 807 K = 16 807 и M = 2 147 483 647, M = 2 147 483 647, используя десять различных начальных значений a1, a1, вычислить последовательности битов bnbn до n = 1000, n = 1000 и сравнить их среднее значение до десяти таких последовательностей, генерируемых генератором случайных битов.

66.

[T] Найдите первые 1000–1000 цифр числа ππ с помощью компьютерной программы или Интернет-ресурса. Создайте битовую последовательность bnbn, положив bn = 1bn = 1, если n-я цифра ππ нечетная, и bn = 0bn = 0, если n-я цифра ππ четная. Вычислить среднее значение bnbn и среднее значение dn = | bn + 1 − bn |, dn = | bn + 1 − bn |, n = 1, …, 999.n = 1, …, 999 . Кажется ли последовательность bnbn случайной? Кажутся ли различия между последовательными элементами bnbn случайными?

бесконечных последовательностей и серий | Безграничное исчисление

Последовательности

Последовательность — это упорядоченный список объектов, который можно рассматривать как функцию, домен которой — натуральные числа.

Цели обучения

Различают последовательность и набор

Основные выводы

Ключевые моменты
  • Подобно набору, последовательность содержит элементы (также называемые элементами). В отличие от набора, порядок имеет значение в последовательности, и одни и те же элементы могут появляться несколько раз в разных местах.
  • Термины последовательности обычно обозначаются одной переменной, например [latex] a_n [/ latex], где индекс [latex] n [/ latex] указывает на [latex] n [/ latex] -й элемент последовательности. .
  • Последовательности, элементы которых напрямую связаны с предыдущими элементами, часто задаются с помощью рекурсии.
Ключевые термины
  • set : набор отдельных объектов, рассматриваемых как самостоятельный объект
  • рекурсия : действие по определению объекта (обычно функции) в терминах самого этого объекта

Последовательность — это упорядоченный список объектов (или событий). Как и набор, он содержит элементы (также называемые элементами или терминами).Количество упорядоченных элементов (возможно, бесконечное) называется длиной последовательности. В отличие от набора, порядок имеет значение в последовательности, и одни и те же элементы могут появляться несколько раз в разных местах последовательности. Точнее, последовательность может быть определена как функция, домен которой является счетным, полностью упорядоченным набором, например натуральными числами.

Примеры: [латекс] (M, A, R, Y) [/ latex] — это последовательность, отличная от [latex] (A, R, M, Y) [/ latex]. Кроме того, последовательность [latex] (1, 1, 2, 3, 5, 8) [/ latex], которая содержит число [latex] 1 [/ latex] в двух разных позициях, является допустимой последовательностью.Последовательности могут быть конечными, как в этом примере, или бесконечными, например, последовательность всех четных положительных целых чисел [latex] (2, 4, 6, \ cdots) [/ latex]. Конечные последовательности иногда называют строками или словами, а бесконечные последовательности — потоками. Пустая последовательность [latex] (\ quad) [/ latex] включена в большинство понятий последовательности, но может быть исключена в зависимости от контекста.

Индексирование

Термины последовательности обычно обозначаются одной переменной, например [latex] a_n [/ latex], где индекс [latex] n [/ latex] указывает на [latex] n [/ latex] -ый элемент последовательности. .2} [/ латекс].

Конвергентная последовательность : График сходящейся последовательности ([latex] a_n [/ latex]) показан синим цветом. Визуально мы можем видеть, что последовательность сходится к пределу [latex] 0 [/ latex] по мере увеличения [latex] n [/ latex].

Указание последовательности с помощью рекурсии

Последовательности, элементы которых напрямую связаны с предыдущими элементами, часто задаются с помощью рекурсии. Это контрастирует с указанием элементов последовательности с точки зрения их положения.Чтобы указать последовательность с помощью рекурсии, требуется правило для построения каждого последовательного элемента в терминах элементов перед ним. Кроме того, необходимо указать достаточное количество начальных элементов, чтобы правилом можно было указать новые элементы последовательности.

Пример

Последовательность Фибоначчи может быть определена с использованием рекурсивного правила вместе с двумя начальными элементами. Правило состоит в том, что каждый элемент представляет собой сумму двух предыдущих элементов, а первыми двумя элементами являются [latex] 0 [/ latex] и [latex] 1 [/ latex]: [latex] a_n = a_ {n-1} + a_ {n-2} [/ latex] и [latex] a_0 = 0, \, a_1 = 1 [/ latex].Первые десять членов этой последовательности ([латекс] 0,1,1,2,3,5,8,13,21,34 [/ латекс]).

серии

Серия — это сумма членов последовательности.

Цели обучения

Укажите требования к сходимости ряда к пределу

Основные выводы

Ключевые моменты
  • Учитывая бесконечную последовательность чисел [latex] \ {a_n \} [/ latex], ряд неформально является результатом сложения всех этих терминов вместе: [latex] \ sum_ {n = 0} ^ \ infty a_n [/ латекс].{\ infty} a_n \ Leftrightarrow L = \ lim_ {k \ rightarrow \ infty} S_k [/ latex].
Ключевые термины
  • последовательность : упорядоченный список объектов
  • Дихотомия Зенона : То, что находится в движении, должно достигнуть промежуточной стадии, прежде чем достигнет цели.

Неформально говоря, серия — это сумма членов последовательности. Конечные последовательности и серии определили первый и последний члены, тогда как бесконечные последовательности и серии продолжаются бесконечно.n} = \ frac {1} {2} + \ frac {1} {4} + \ frac {1} {8} + \ cdots} [/ latex]

Парадокс Зенона : Предположим, вы работаете из места [латекс] x = 0 [/ latex] в направлении [latex] x = 100 [/ latex]. Прежде чем вы сможете добраться туда, вы должны пройти половину пути. Прежде чем вы сможете добраться до середины пути, вы должны пройти четверть пути. Прежде чем проехать четверть, вы должны проехать одну восьмую; перед восьмым — одна шестнадцатая; и так далее.

Члены ряда часто создаются в соответствии с определенным правилом, например, с помощью формулы или алгоритма.Поскольку существует бесконечное количество членов, это понятие часто называют бесконечным рядом. В отличие от конечных суммирований, бесконечные ряды нуждаются в инструментах математического анализа, в частности, в понятии пределов, чтобы их можно было полностью понять и ими можно было управлять. Помимо того, что бесконечные ряды широко используются в математике, они также широко используются в других количественных дисциплинах, таких как физика, информатика и финансы.

Определение

Для любой последовательности рациональных чисел, действительных чисел, комплексных чисел, их функций и т. Д.{\ infty} a_n \ Leftrightarrow L = \ lim_ {k \ rightarrow \ infty} S_k} [/ latex]

Интегральный тест и оценки сумм

Интегральный тест — это метод проверки бесконечной серии неотрицательных членов на сходимость путем сравнения их с несобственным интегралом.

Цели обучения

Опишите цель интегрального теста

Основные выводы

Ключевые моменты
  • В интегральном тесте используется монотонно убывающая функция [latex] f [/ latex], определенная на неограниченном интервале [latex] [N, \ infty) [/ latex] (где [latex] N [/ latex] — целое число) .\ infty \ frac1n [/ latex] расходится.
Ключевые термины
  • неправильный интеграл : интеграл, в котором хотя бы одна из конечных точек принимается в качестве предела либо до определенного числа, либо до бесконечности
  • натуральный логарифм : логарифм по основанию [латекс] e [/ латекс]

Интегральный тест на сходимость — это метод, используемый для проверки бесконечного ряда неотрицательных членов на сходимость. Он был разработан Колином Маклореном и Огюстэном-Луи Коши и иногда известен как тест Маклорена – Коши.\ varepsilon} \ Bigr) \ le \ frac1 \ varepsilon} [/ латекс]

Интегральный тест : Интегральный тест, применяемый к гармоническому ряду. {1+ \ varepsilon}}} = \ infty} [/ латекс]

для каждого [латекса] \ varepsilon> 0 [/ latex], а также того, расходится ли соответствующая серия [latex] f (n) [/ latex].Как только такая последовательность найдена, можно задать аналогичный вопрос о [латексе] f (n) [/ latex], взявшем на себя роль [latex] \ frac {1} {n} [/ latex] или так далее. Таким образом, можно исследовать границу между расхождением и сходимостью бесконечных рядов.

Сравнительные тесты

Сравнительный тест может означать либо тест сравнения пределов, либо тест прямого сравнения, оба из которых могут использоваться для проверки сходимости ряда.

Цели обучения

Различают тесты сравнения пределов и тесты прямого сравнения

Основные выводы

Ключевые моменты
  • Для последовательностей [латекс] \ {a_n \} [/ latex], [latex] \ {b_n \} [/ latex], только с неотрицательными терминами, если [latex] \ lim_ {n \ to \ infty } \ frac {a_n} {b_n} = c [/ latex] с [latex] 0
  • Если бесконечный ряд [latex] \ sum b_n [/ latex] сходится и [latex] 0 \ le a_n \ le b_n [/ latex] для всех достаточно больших [latex] n [/ latex] (то есть для всех [ latex] n> N [/ latex] для некоторого фиксированного значения [latex] N [/ latex]), то бесконечный ряд [latex] \ sum a_n [/ latex] также сходится.
  • Если бесконечный ряд [latex] \ sum b_n [/ latex] расходится и [latex] 0 \ le a_n \ le b_n [/ latex] для всех достаточно больших [latex] n [/ latex], то бесконечный ряд [latex] ] \ sum a_n [/ latex] тоже расходится.
Ключевые термины
  • интегральный тест : метод, используемый для проверки бесконечного ряда неотрицательных членов на сходимость путем сравнения его с несобственными интегралами
  • неправильный интеграл : интеграл, в котором хотя бы одна из конечных точек принимается в качестве предела либо до определенного числа, либо до бесконечности

Сравнительные испытания могут означать либо предельные сравнительные испытания, либо прямые сравнительные испытания. 2} [/ latex].2} [/ latex]) тоже расходится.

Тест прямого сравнения

Тест прямого сравнения позволяет вывести сходимость или расхождение бесконечного ряда или неправильного интеграла. В обоих случаях тест работает, сравнивая данный ряд или интеграл с тем, свойства сходимости которого известны. В этом атоме мы проверим только серийный случай.

Для последовательностей [латекс] \ {a_n \} [/ latex], [latex] \ {b_n \} [/ latex] с неотрицательными терминами:

  • Если бесконечный ряд [latex] \ sum b_n [/ latex] сходится и [latex] 0 \ le a_n \ le b_n [/ latex] для всех достаточно больших [latex] n [/ latex] (то есть для всех [latex] n> N [/ latex] для некоторого фиксированного значения [latex] N [/ latex]), то бесконечный ряд [latex] \ sum a_n [/ latex] также сходится.{n-1} \, a_n [/ latex] с [latex] a_n> 0 [/ latex] для всех [latex] n [/ latex].

    Цели обучения

    Описать свойства чередующегося ряда и требования к сходимости одного ряда

    Основные выводы

    Ключевые моменты
    • Теорема, известная как «тест Лейбница», или тест чередующихся серий, говорит нам, что чередующиеся серии сходятся, если члены [latex] a_n [/ latex] монотонно сходятся к [latex] 0 [/ latex].
    • Знаки общих терминов чередуются между положительными и отрицательными.{n-1} \, a_n} [/ латекс]

      с [latex] a_n> 0 [/ latex] для всех [latex] n [/ latex]. Знаки общих терминов чередуются между положительными и отрицательными. Как и любой ряд, чередующийся ряд сходится тогда и только тогда, когда сходится связанная с ним последовательность частичных сумм.

      Испытание чередующейся серии

      Теорема, известная как «тест Лейбница», или тест чередующихся серий, говорит нам, что чередующиеся серии будут сходиться, если члены [latex] a_n [/ latex] сходятся к [latex] 0 [/ latex] монотонно.k \, a_k \\ & = a_ {m + 1} -a_ {m + 2} + a_ {m + 3} -a_ {m + 4} + \ cdots + a_n \\ & = \ displaystyle a_ {m + 1} — (a_ {m + 2} -a_ {m + 3}) — \ cdots-a_n \ le a_ {m + 1} \ le a_ {m} \\ & \ quad [a_ {n} \ text { убывает}]. \ end {align} [/ latex]

      Поскольку [latex] a_n [/ latex] монотонно убывает, члены отрицательные. Таким образом, мы имеем окончательное неравенство [латекс] S_m — S_n \ le a_ {m} [/ latex]. \ infty \ left | a_n \ right | = L [/ latex] для некоторого действительного числа [latex] L [/ latex].

    • Корневой тест — это тест сходимости бесконечного ряда, который использует предел [латекс] L = \ lim_ {n \ rightarrow \ infty} \ left | \ frac {a_ {n + 1}} {a_n} \ right | [/ латекс].
    • Корневой тест — это критерий сходимости бесконечного ряда, использующий верхний предел [латекс] C = \ limsup_ {n \ rightarrow \ infty} \ sqrt [n] {| a_n |} [/ latex].
    Ключевые термины
    • слагаемое : то, что складывается или суммируется
    • неправильный интеграл : интеграл, в котором хотя бы одна из конечных точек принимается в качестве предела либо до определенного числа, либо до бесконечности
    • верхний предел : верхняя грань набора точек накопления заданной последовательности или набора

    Бесконечный ряд чисел считается абсолютно сходящимся (или абсолютно сходящимся), если сумма абсолютного значения слагаемого конечна.\ infty a_n [/ latex], где каждый член является действительным или комплексным числом, а [latex] a_n [/ latex] отличен от нуля, когда n велико. Тест был впервые опубликован Жаном ле Рондом д’Аламбером и иногда известен как критерий отношения Д’Аламбера.

    Обычная форма теста использует предел, [латекс] L = \ lim_ {n \ rightarrow \ infty} \ left | \ frac {a_ {n + 1}} {a_n} \ right | [/ latex] . Тест соотношения утверждает, что

    • если [latex] L <1 [/ latex], то ряд абсолютно сходится;
    • если [latex] L> 1 [/ latex], то ряд не сходится;
    • если [latex] L = 1 [/ latex] или предел не существует, то проверка не дает результатов, потому что существуют как сходящиеся, так и расходящиеся ряды, которые удовлетворяют этому случаю.

    Корневой тест

    Корневой тест — это критерий сходимости (тест сходимости) бесконечного ряда. Это зависит от количества [латекс] \ limsup_ {n \ rightarrow \ infty} \ sqrt [n] {| a_n |} [/ latex], где [latex] a_n [/ latex] — термины серии, и состояния что ряд сходится абсолютно, если эта величина меньше единицы, и расходится, если она больше единицы. Это особенно полезно при работе с силовыми рядами.

    Корневой тест был впервые разработан Огюстэном-Луи Коши и поэтому иногда известен как тест корня Коши или радикальный тест Коши.\ infty a_n [/ latex], корневой тест использует число [latex] C = \ limsup_ {n \ rightarrow \ infty} \ sqrt [n] {\ left | a_n \ right |} [/ latex], где «lim sup ”обозначает верхний предел, возможно ∞. Обратите внимание, что если [latex] \ lim_ {n \ rightarrow \ infty} \ sqrt [n] {\ left | a_n \ right |} [/ latex] сходится, то он равен [latex] C [/ latex] и может использоваться вместо этого в корневом тесте. Корневой тест утверждает, что

    • если [latex] C <1 [/ latex], то ряд абсолютно сходится;
    • если [latex] C> 1 [/ latex], то ряд расходится;
    • если [latex] C = 1 [/ latex] и предел приближается строго сверху, то ряд расходится;
    • , в противном случае тест будет безрезультатным (ряды могут расходиться, сходиться абсолютно или сходиться условно).2}}} [/ latex]

      и есть другие, для которых [latex] C = 1 [/ latex] и ряд расходится, например:

      [латекс] \ displaystyle {\ sum {\ frac {1} {n}}} [/ латекс]

      Ratio Test : В этом примере соотношение соседних членов в синей последовательности сходится к [latex] L = \ frac {1} {2} [/ latex]. Выбираем [латекс] r = \ frac {L + 1} {2} = \ frac {3} {4} [/ latex]. Тогда синяя последовательность преобладает над красной последовательностью для всех [латекс] n \ geq 2 [/ latex]. Красная последовательность сходится, синяя — тоже.

      Советы по тестированию серии

      Тесты сходимости — это методы проверки сходимости или расходимости бесконечного ряда.

      Цели обучения

      Сформулируйте три метода, которые помогут при проверке сходимости ряда

      Основные выводы

      Ключевые моменты
      • Не существует единого теста сходимости, который работал бы для всех серий.
      • Практика и обучение помогут выбрать правильный тест для данной серии.
      • Мы узнали о тесте корень / отношение, интегральном тесте и тесте прямого / предельного сравнения.
      Ключевые термины
      • условная сходимость : Ряд или интеграл считаются условно сходящимися, если они сходятся, но не сходятся абсолютно.

      Тесты сходимости — это методы проверки сходимости, условной сходимости, абсолютной сходимости, интервала сходимости или расходимости бесконечного ряда.При проверке сходимости ряда следует помнить, что не существует единого теста сходимости, который работал бы для всех рядов. Вы должны угадать и выбрать правильный тест для данной серии. Практика и обучение помогут вам ускорить этот процесс «угадывания».

      Вот краткое изложение теста сходимости, который мы изучили:

      Список тестов

      Предел слагаемого: Если предел слагаемого не определен или не равен нулю, то ряды должны расходиться.

      Тест соотношения: для [латекса] r = \ lim_ {n \ to \ infty} \ left | \ frac {a_ {n + 1}} {a_n} \ right | [/ latex], если [латекс] r <1 [/ latex], ряд сходится; если [latex] r> 1 [/ latex], ряд расходится; если [latex] r = 1 [/ latex], тест считается безрезультатным.

      Корневой тест: для [latex] r = \ limsup_ {n \ to \ infty} \ sqrt [n] {\ left | a_n \ right |} [/ latex], если [latex] r <1 [/ latex], затем ряд сходится; если [latex] r> 1 [/ latex], то ряд расходится; если [latex] r = 1 [/ latex], корневой тест не дает результатов. 3 + \ cdots} [/ latex]

      Эти степенные ряды возникают в основном в реальном и комплексном анализе, но также встречаются в комбинаторике (под названием производящих функций) и в электротехнике (под названием [латекс] Z [/ латекс] -преобразование).Знакомую десятичную запись для действительных чисел также можно рассматривать как пример степенного ряда с целочисленными коэффициентами, но с аргументом [latex] x [/ latex], фиксированным на [latex] \ frac {1} {10} [/ латекс]. В теории чисел концепция [латексных] p [/ latex] -адических чисел также тесно связана с концепцией степенного ряда.

      Радиус схождения

      Степенный ряд сходится для одних значений переменной [latex] x [/ latex] и может расходиться для других. Все степенные ряды [latex] f (x) [/ latex] в степенях [latex] (x-c) [/ latex] сходятся в [latex] x = c [/ latex].r [/ latex], где [latex] c [/ latex] и [latex] r [/ latex] — постоянные действительные числа.

      Цели обучения

      Опишите взаимосвязь между степенными функциями и бесконечно дифференцируемыми функциями

      Основные выводы

      Ключевые моменты
      • Поскольку все бесконечно дифференцируемые функции могут быть представлены в виде степенных рядов, любую бесконечно дифференцируемую функцию можно представить в виде суммы многих степенных функций (целых показателей). {n} [/ латекс].r [/ latex], где [latex] c [/ latex] и [latex] r [/ latex] — постоянные действительные числа. Многочлены состоят из степенных функций. Поскольку все бесконечно дифференцируемые функции могут быть представлены в виде степенных рядов, любую бесконечно дифференцируемую функцию можно представить как сумму многих степенных функций (целочисленных показателей). Иногда область значений степенной функции может состоять только из действительных чисел, но обычно используется неотрицательное значение, чтобы избежать проблем с упрощением. Область определения определяется в каждом отдельном случае.{-4} [/ latex] — все функции мощности.

        Серия Тейлора и Маклорена

        Ряд Тейлора представляет функцию как бесконечную сумму членов, вычисленных из значений производных функции в одной точке.

        Цели обучения

        Определить серию Маклорена как частный случай серии Тейлора

        Основные выводы

        Ключевые моменты
        • Любое конечное число начальных членов ряда Тейлора функции называется многочленом Тейлора.{n} [/ латекс]. Если [latex] a = 0 [/ latex], серия называется серией Маклорена.
        Ключевые термины
        • дифференцируемый : имеющий производную, говоря о функции, область определения и ко-домен которой являются многообразиями
        • аналитическая функция : функция с действительным знаком, которая однозначно определяется через свои производные в одной точке

        Ряд Тейлора — это представление функции в виде бесконечной суммы членов, которые вычисляются из значений производных функции в одной точке.Концепция ряда Тейлора была официально введена английским математиком Бруком Тейлором в 1715 году. Если ряд Тейлора центрирован на нуле, то этот ряд также называется рядом Маклорена в честь шотландского математика Колина Маклорена, который широко использовал это частный случай серии Тейлора в 18 веке.

        Обычно аппроксимируют функцию с помощью конечного числа членов ее ряда Тейлора. Теорема Тейлора дает количественные оценки погрешности этого приближения.Любое конечное число начальных членов ряда Тейлора функции называется многочленом Тейлора. Ряд Тейлора функции — это предел полиномов Тейлора этой функции при условии, что предел существует. Функция может не быть равна своему ряду Тейлора, даже если ее ряд Тейлора сходится в каждой точке. Функция, которая равна своему ряду Тейлора на открытом интервале (или круге на комплексной плоскости), называется аналитической функцией.

        Экспоненциальная функция как степенной ряд : экспоненциальная функция (синим цветом) и сумма первых [латексных] n + 1 [/ latex] членов ряда Тейлора в [латексе] 0 [/ латексе] (красным ) до [латекс] n = 8 [/ латекс].п} {п! }} [/ latex]

        Применения серии Тейлора

        Разложение в ряд Тейлора может помочь в приближении значений функций и вычислении определенных интегралов.

        Цели обучения

        Опишите применения расширения серии Тейлора

        Основные выводы

        Ключевые моменты
        • Частичные суммы ряда, называемые полиномами Тейлора, могут использоваться как приближения всей функции.
        • Дифференцирование и интегрирование степенных рядов может выполняться поэтапно, и, следовательно, это может быть проще, чем напрямую работать с исходной функцией.
        • (Усеченный) ряд может использоваться для численного вычисления значений функции. Это особенно полезно при вычислении специальных математических функций (таких как функция Бесселя).
        Ключевые термины
        • определенный интеграл : интеграл функции между верхним и нижним пределом
        • комплексный анализ : теория функций комплексного переменного; раздел математического анализа, изучающий функции комплексных чисел
        • аналитическая функция : функция с действительным знаком, которая однозначно определяется через свои производные в одной точке

        Использование ряда Тейлора для аналитических функций включает:

        1.Частичные суммы (многочлены Тейлора) ряда могут использоваться как приближения всей функции. Эти приближения часто бывают достаточно хорошими, если включено достаточно много членов. Аппроксимации с использованием первых нескольких членов ряда Тейлора могут сделать неразрешимые в противном случае проблемы возможными для ограниченной области; этот подход часто используется в физике.

        Полиномы Тейлора : Чем больше членов добавляется к многочлену Тейлора, он приближается к правильной функции.3 [/ latex] можно использовать. Поскольку каждый член суммирования можно проинтегрировать отдельно, мы можем вычислить определенный интеграл, если сумма сходится.

        3. Аналитическая функция однозначно продолжается до голоморфной функции на открытом диске комплексной плоскости. Это делает доступным аппарат комплексного анализа.

        4. (Усеченный) ряд может использоваться для численного вычисления значений функции. Это особенно полезно при вычислении специальных математических функций (таких как функция Бесселя).{ix} \ end {align} [/ latex]

        Этот результат имеет фундаментальное значение во многих областях математики (например, в комплексном анализе), физике и технике.

        Суммирование бесконечного ряда

        Бесконечные последовательности и серии могут сходиться или расходиться.

        Цели обучения

        Описание свойств бесконечной серии

        Основные выводы

        Ключевые моменты
        • Бесконечные последовательности и серии продолжаются бесконечно.
        • Говорят, что ряд сходится, если последовательность частичных сумм имеет конечный предел.
        • Считается, что ряд расходится, если предел бесконечен или не существует.
        Ключевые термины
        • предел : значение, к которому сходится последовательность или функция
        • последовательность : упорядоченный список объектов

        Серия — это сумма членов последовательности. Конечные последовательности и серии определили первый и последний члены, тогда как бесконечные последовательности и серии продолжаются бесконечно.{\ infty} a_n \ Leftrightarrow L = \ lim_ {k \ rightarrow \ infty} S_k} [/ latex]

        Если предел бесконечен или не существует, говорят, что ряд расходится.

        Бесконечная серия : бесконечная последовательность действительных чисел, показанная синими точками. Эта последовательность не является ни возрастающей, ни убывающей, ни сходящейся, ни коши. Однако он ограничен.

        Простой способ сходимости бесконечного ряда — это если все [latex] a_ {n} [/ latex] равны нулю для достаточно больших [latex] n [/ latex] s.Такой ряд можно отождествить с конечной суммой, поэтому он бесконечен только в тривиальном смысле. Выявление свойств рядов, которые сходятся, даже если бесконечно много членов не равны нулю, поэтому составляет суть изучения рядов. В следующих атомах мы изучим, как определить, сходится ли ряд или нет, и как вычислить сумму ряда, если такое значение существует.

        Сходимость рядов с положительными членами

        Для последовательности [латекс] \ {a_n \} [/ latex], где [latex] a_n [/ latex] — неотрицательное действительное число для каждого [latex] n [/ latex], сумма [latex] \ sum_ {n = 0} ^ {\ infty} a_n [/ latex] может либо сходиться, либо расходиться к [latex] \ infty [/ latex]. {k} a_n = a_0 + a_1 + \ cdots + a_k [/ latex]

        не убывает.n} + \ cdots} [/ латекс]

        сходятся? Можно ли «визуализировать» его схождение на прямой числовой? Мы можем представить линию длиной [латекс] 2 [/ латекс], с последовательными отрезками, обозначенными длиной [латекс] 1 [/ латекс], [латекс] \ frac {1} {2} [/ латекс], [латекс ] \ frac {1} {4} [/ latex] и т. д. Всегда есть место, чтобы отметить следующий сегмент, потому что количество оставшейся линии всегда такое же, как и в последнем отмеченном сегменте: когда мы отметили [латекс] \ frac {1} {2} [/ latex], у нас все еще есть кусок [латекс] \ frac {1} {2} [/ latex] без маркировки, так что мы определенно можем отметить следующий [латекс] \ frac { 1} {4} [/ латекс].Этот аргумент не доказывает, что сумма равна [latex] 2 [/ latex] (хотя это так), но доказывает, что это не более [latex] 2 [/ latex]. Другими словами, у ряда есть верхняя граница. Однако для доказательства того, что серия равна [latex] 2 [/ latex], требуется только элементарная алгебра. Если серия обозначена [латекс] S [/ латекс], можно увидеть, что:

        [латекс] \ displaystyle {\ frac {S} {2} \, = \ frac {1+ \ frac {1} {2} + \ frac {1} {4} + \ frac {1} {8} + \ cdots} {2} \\ \ quad = \ frac {1} {2} + \ frac {1} {4} + \ frac {1} {8} + \ frac {1} {16} + \ cdots} [/ латекс]

        Следовательно:

        [латекс] \ displaystyle {S- \ frac {S} {2} = 1 \\ S = 2} [/ латекс]

        Геометрическая сумма : Визуализация геометрической суммы в примере 2.Длина линии ([latex] 2 [/ latex]) может содержать все последовательные отрезки с размеченными длинами [latex] 1 [/ latex], [latex] \ frac {1} {2} [/ latex], [latex] \ frac {1} {4} [/ latex] и т. д.

        Для этих конкретных примеров есть простые способы проверить сходимость. Однако может оказаться, что простых способов проверить сходимость не существует. Для этих общих случаев мы можем поэкспериментировать с несколькими хорошо известными тестами сходимости (такими как тест отношения, интегральный тест и т. Д.). Мы изучим некоторые из этих тестов в следующих атомах.

        пределов последовательностей | Блестящая вики по математике и науке

        Здесь мы обсудим аспекты, которые вам необходимо знать для понимания концепции сходимости последовательности. Мы предоставим вам пошаговую презентацию всех концепций. Во-первых, что такое последовательность?

        Последовательность — это функция f: N → Rf: \ mathbb N \ rightarrow \ mathbb Rf: N → R, определенная как f (n) = xnf (n) = x_nf (n) = xn, и обычно обозначается на x1, x2, …, xnx_1, x_2 ,.3} {3 + 1}, \ ldots1 + 113, 2 + 123, 3 + 133,….

      • Поскольку мы теперь знакомы с последовательностями, давайте попробуем понять, что представляет собой предел последовательности. Проще говоря, предел — это математически точный способ говорить о приближении к значению без необходимости оценивать его напрямую.

        Действительное число LLL — это предел последовательности xnx_nxn, если числа в последовательности становятся все ближе и ближе к LLL, а не к какому-либо другому числу. В общем смысле предел последовательности — это значение, к которому она приближается с произвольной близостью.

        Например, если xn = cx_n = cxn = c для некоторой константы c, c, c, то lim⁡n → ∞xn → c, \ displaystyle \ lim_ {n \ to \ infty} x_n \ to c, n → ∞lim xn → c, и если xn = 1n, x_n = \ frac 1n, xn = n1, то lim⁡n → ∞xn → 0 \ displaystyle \ lim_ {n \ to \ infty} x_n \ to 0n → ∞lim xn → 0.

        Когда предел последовательности при n → ∞n \ to \ inftyn → ∞ приближается к одному значению, мы говорим, что последовательность сходится. Определим сходимость последовательности формально:

        Мы говорим, что последовательность xnx_nxn сходится к , если существует x0∈Rx_0 \ in \ mathbb Rx0 ∈R такое, что для любого ϵ> 0 \ epsilon> 0ϵ> 0 существует натуральное число NNN такое, что xn∈ (x0 −ϵ, x0 + ϵ) x_n \ in (x_0 — \ epsilon, x_0 + \ epsilon) xn ∈ (x0 −ϵ, x0 + ϵ) или ∣xn − x0∣ <ϵ | x_n -x_0 | <\ epsilon∣xn −x0 ∣ <ϵ для всех n≥Nn \ geq Nn≥N.

        Легко проверить, что если такое число x0x_0x0 существует, то оно уникально. В этом случае мы говорим, что последовательность xnx_nxn сходится к x0x_0x0, и называем x0x_0x0 пределом последовательности xnx_nxn. Если x0x_0x0 является пределом xnx_nxn, мы пишем lim⁡n → ∞xn = x0 \ displaystyle \ lim_ {n \ to \ infty} x_n = x_0n → ∞lim xn = x0.

        Примечание: Сходимость каждой последовательности, приведенной в приведенных выше примерах, проверяется непосредственно из определения. В общем, проверка сходимости непосредственно из определения — сложная задача.Мы увидим некоторые способы найти пределы определенных последовательностей и некоторые достаточные условия сходимости последовательности.

        Теперь, когда мы получили концепцию конвергенции в теоретическом плане, пришло время разработать несколько примеров и построить прочный фундамент конвергенции последовательностей. Поехали ::

        Сходится ли следующая последовательность:

        11,12,13,…, 1n,…? \ frac {1} {1}, \ frac {1} {2}, \ frac {1} {3}, \ ldots, \ frac {1} {n}, \ ldots \ ,? 11, 21, 31,…, n1,…?


        Последовательность приближается к нулю.Чем больше становится nnn, тем меньше и меньше член становится равным нулю. Таким образом, последовательность сходится. □ _ \ квадрат □

        Доказательство:
        Для произвольного ϵ> 0 \ epsilon> 0ϵ> 0 неравенство ∣xn∣ = 1n <ϵ | x_n | = \ frac 1n <\ epsilon∣xn ∣ = n1 <ϵ верно для всех n> 1ϵn> \ frac {1} {\ epsilon} n> ϵ1 и, следовательно, для всех n> Nn> Nn> N, где NNN — любое натуральное число такое, что N> 1ϵ N> \ frac {1} {\ epsilon} N> ϵ1. Таким образом, для любого ϵ> 0 \ epsilon> 0ϵ> 0 существует натуральное число NNN такое, что ∣xn∣ <ϵ | x_n | <\ epsilon∣xn ∣ <ϵ для любого n≥Nn \ geq Nn≥N.n} f (n) = 1 + 10n1 сходятся:

        1.1, 1.01, 1.001,…? 1.1, \ 1.01, \ 1.001, \ \ ldots \,? 1.1, 1.01, 1.001,…?


        В этой последовательности мы видим, что значения уменьшаются по мере увеличения nnn и в конечном итоге приближаются к единственному значению. Чем больше мы берем значение nnn, тем ближе и ближе член становится к 1. Следовательно, элементы данной последовательности приближаются к 1, когда nnn приближается к бесконечности. n} {n} f (n) = n (−1) n:

        −11,12, −13,…? \ Dfrac {-1} {1}, \ dfrac 12, \ dfrac {-1} {3}, \ ldots \,? 1−1, 21, 3− 1,…?


        Последовательность приближается к нулю.n} {n} n (−1) n колеблются, они «в конце концов приближаются» к единственной точке 0. Общей чертой этих последовательностей является то, что члены каждой последовательности «накапливаются» только в одной точке. □ _ \ квадрат □

        журнал⁡2 \ журнал 2log2 ln⁡2 \ ln 2ln2 111 222

        Пусть g (n) = n − ⌊n2⌋ + ⌊n3⌋ − ⌊n4⌋ + ⋯. {n + 1}, \ ldots \,? 1, -1,1, -1,1, -1,… , (- 1) n + 1,…?


        Ясно, что последовательность колеблется между 1 и -1 и не сходится к значению.n (-1) n колеблется между двумя разными точками -1 и 1, что означает, что элементы последовательности приближаются к -1 и 1 «часто» по мере увеличения nnn. □ _ \ квадрат □

        Мы говорим, что функция расходится до бесконечности , если она стремится к положительной бесконечности или отрицательной бесконечности.

        Например, такими функциями являются f (n) = nf (n) = nf (n) = n и f (n) = ln⁡nf (n) = \ ln nf (n) = lnn.

        Сходится ли следующая последовательность:

        1,2,3,…, п,…? 1, 2, 3, \ ldots, n, \ ldots \,? 1,2,3,…, n,…?


        Последовательность целых чисел выше не ограничена.\ text {nd} 2-й пример), \ big),) то такая последовательность расходится. Ниже будет показано, что если последовательность сходится, то предел разницы между последовательными членами равен 0.

        Примечание 2 : Это правда, что если положительная последовательность не убывает, то предел существует. Однако, возможно, нам не удастся легко определить предел.

        формальных доказательств — Доказательство сходимости последовательностей

        Я постараюсь помочь вам, насколько смогу.

        Напоминаю определение:

        Мы говорим, что последовательность $ (x_n) _ {n \ in \ mathbb {N}} $ сходится к действительному числу $ x $, если для всех $ \ varepsilon> 0 $ существует $ N \ in \ mathbb {N} $ такое, что для всех $ n> N $, $ | x_n — x | <\ varepsilon $.Формально
        \ begin {gather *} \ forall \ varepsilon> 0, \ существует N \ in \ mathbb {N}: | x_n — x | <\ varepsilon, \ \ forall n> Н. \ end {gather *}

        Первое, на что следует обратить внимание в связи с этим определением, — это то, что неравенства могут быть строгими или нет (кроме $ \ varepsilon $). Если вы не понимаете этого факта, просто скажите мне, но я думаю, вам не так сложно понять, почему.

        Как вы поняли, вы должны исправить $ \ varepsilon> 0 $ и выбрать (или найти) целое число $ N $ такое, что условие $ | x_n — x | <\ varepsilon $ выполняется для всех целых чисел $ n> N $.

        $ \ varepsilon $ является произвольным, потому что вы хотите, чтобы ваша последовательность $ (x_n) $ была как можно ближе к $ x $. И это то, что говорится в определении. Если вы сделаете картинку, это означает, что для каждого уровня $ \ varepsilon> 0 $, после определенного целого числа $ N $, расстояние от $ x_n $ до $ x $ меньше, чем $ \ varepsilon $, т.е. для всех $ n > N $, $ x_n \ in] x- \ varepsilon, x + \ varepsilon [$. Логично, что чем меньше $ \ varepsilon $, тем большее целое число $ N $ нужно найти. Это в некотором смысле принцип ограничения: «Быть ​​как можно ближе к числу, как $ n \ rightarrow \ infty \ (\ text {то есть} \ forall n> N) $».

        Обратите внимание, что определение особенно полезно для доказательства теоретических результатов, потому что для его использования вы должны сначала знать значение предела. Но во многих частных случаях у нас есть некоторые методы расчета и приемы, чтобы найти предел (возможно, для последних трех примеров, которые вы упомянули). Вероятно, вы должны знать некоторых из них по старшей школе.

        Теперь давайте попробуем это определение на ваших примерах.

        1) $ x_n = \ frac {1} {n} $ (Я беру $ k = 1 $, но это та же идея для других $ k $…)
        Пусть $ \ varepsilon> 0 $. Выбираем $ N = \ left \ lfloor \ frac {1} {\ varepsilon} \ right \ rfloor + 1 $. Обратите внимание, что $ N $ зависит от $ \ varepsilon $ (не обязательно, но может, и это очевидно из-за логического порядка кванторов: forall .. (фиксировано) существует ((почти) один) ..). Тогда для всех $ n> N $, $ | x_n — 0 | = | x_n | = \ frac {1} {n} <\ frac {1} {N} <\ varepsilon $.

        Ну, так во всех книгах написано .. Но вы остаетесь в той же точке .. Почему ему так легко? Я знаю, что был похож на тебя.. Итак, одна из стратегий — думать противоположным образом.

        Какой $ N $ нужно взять, чтобы было равенство $ \ frac {1} {N} = \ varepsilon $? Неудивительно, что ответ — $ N = \ frac {1} {\ varepsilon} $. Но на самом деле мы не уверены, что это целое число, поэтому берем целую часть этого действительного числа. Но по определению $ \ lfloor x \ rfloor \ leq x $ или эквивалентно $ \ frac {1} {\ lfloor x \ rfloor} \ geq 1 / x $. Итак, с выбором $ N = \ left \ lfloor \ frac {1} {\ varepsilon} \ right \ rfloor $, у нас есть $ \ frac {1} {N} \ geq \ varepsilon $! Итак, вы берете $ N = \ left \ lfloor \ frac {1} {\ varepsilon} \ right \ rfloor + 1 $, чтобы получить $ \ frac {1} {N} <\ varepsilon $, потому что по определению $ x <\ lfloor x \ rfloor + 1 $.k} \ leq \ frac {1} {n} <\ varepsilon $ для всех $ n> N $. Так что достаточно взять $ N $ точки 1.

        3) Наконец, $ x_n = \ frac {4n + 1} {n + 3} $. Интуитивно мы можем записать это как: $ x_n = \ frac {4 + \ frac {1} {n}} {1 + \ frac {3} {n}} $, так что предел составляет $ 4 $. Мы также можем использовать пункт 1. Но давайте просто сделаем это как следует. Пусть $ \ varepsilon> 0 $. Мы выбираем $ N = \ left \ lfloor \ frac {11} {\ varepsilon} — 3 \ right \ rfloor + 1 $, если $ \ varepsilon \ leq \ frac {11} {3} $, и $ 0 $ в противном случае. Тогда для всех $ n> N $ имеем $ | x_n — 4 | = \ left | \ frac {4n + 1} {n + 3} — 4 \ right | = \ left | \ frac {-11} {n + 3} \ right | = \ frac {11} {n + 3} <\ frac {11} {N + 3} <\ varepsilon $.

        Постарайтесь во всем этом разобраться .. Если у вас есть вопросы, не стесняйтесь спрашивать!

        Для обучения этому определению попробуйте доказать:

        Если $ (x_n) _ {n \ in \ mathbb {N}} $ и $ (y_n) _ {n \ in \ mathbb {N}} $ — две последовательности, сходящиеся к $ x $ и $ y $ соответственно, тогда $ x_n + y_n $ и $ x_ny_n $ сходятся к $ x + y $ и $ xy $ соответственно. (Что касается второго, напомню, что каждая сходящаяся последовательность ограничена, т.е. $ \ exists M> 0 $: $ | x_n |

        С уважением,

        Маркус

        страница не найдена — Williams College

        ’62 Центр театра и танца, 62 Центр
        касса 597-2425
        Магазин костюмов 597-3373
        Менеджер мероприятий / Помощник менеджера 597-4808 597-4815 факс
        Производство 597-4474 факс
        Магазин сцен 597-2439
        ’68 Центр карьерного роста, Мирс 597-2311 597-4078 факс
        Academic Resources, Парески 597-4672 597-4959 факс
        Служба поддержки инвалидов, Парески 597-4672
        Прием, Вестон-холл 597-2211 597-4052 факс
        Программа позитивных действий, Хопкинс-холл 597-4376
        Africana Studies, Hollander 597-2242 597-4222 факс
        Американские исследования, Шапиро 597-2074 597-4620 факс
        Антропология и социология, Холландер 597-2076 597-4305 факс
        Архивы и специальные коллекции, Sawyer 597-4200 597-2929 факс
        Читальный зал 597-4200
        Искусство (История, Студия), Spencer Studio Art / Lawrence 597-3578 597-3693 факс
        Архитектурная студия, Spencer Studio Art 597-3134
        Фотография Студия, Spencer Studio Art 597-2030
        Printmaking Studio, Spencer Studio Art 597-2496
        Студия скульптуры, Спенсер Студия искусства 597-3101
        Senior Studio, Spencer Studio Art 597-3224
        Видео / фотостудия, Spencer Studio Art 597-3193
        Азиатские исследования, Холландер 597-2391 597-3028 факс
        Астрономия / астрофизика, Thompson Physics 597-2482 597-3200 факс
        Департамент легкой атлетики, физическое воспитание, отдых, Ласелл 597-2366 597-4272 факс
        Спортивный директор 597-3511
        Boat House, Озеро Онота 443-9851
        Автобусы 597-2366
        Фитнес-центр 597-3182
        Hockey Rink Ice Line, Lansing Chapman 597-2433
        Intramurals, Атлетический центр Чандлера 597-3321
        Физическая культура 597-2141
        Pool Wet Line, Атлетический центр Чандлера 597-2419
        Sports Information, Hopkins Hall 597-4982 597-4158 факс
        Спортивная медицина 597-2493 597-3052 факс
        Площадки для игры в сквош 597-2485
        Поле для гольфа Taconic 458-3997
        Биохимия и молекулярная биология, Thompson Biology 597-2126
        Биоинформатика, геномика и протеомика, Бронфман 597-2124
        Биология, Thompson Biology 597-2126 597-3495 факс
        Охрана и безопасность кампуса, Хопкинс-холл 597-4444 597-3512 факс
        Карты доступа / системы сигнализации 597-4970 / 4033
        Служба сопровождения, Хопкинс-холл 597-4400
        Офицеры и диспетчеры 597-4444
        Секретарь, удостоверения личности 597-4343
        Коммутатор 597-3131
        Центр развития творческого сообщества, 66 Stetson Court 884-0093
        Центр экономики развития, 1065 Main St 597-2148 597-4076 факс
        Компьютерный зал 597-2522
        Вестибюль 597-4383
        Центр экологических исследований, выпуск 1966 года Экологический центр 597-2346 597-3489 факс
        Лаборатория экологических наук, Морли 597-2380
        Экологические исследования 597-2346
        Лаборатория ГИС 597-3183
        Центр иностранных языков, литератур и культур, Холландер 597-2391 597-3028 факс
        Арабские исследования, Холландер 597-2391 597-3028 факс
        Сравнительная литература, Холландер 597-2391
        Критические языки, Холландер 597-2391 597-3028 факс
        Языковая лаборатория 597-3260
        Русский, Холландер 597-2391
        Центр обучения в действии, Brooks House 597-4588 597-3090 факс
        Библиотека редких книг Чапина, Сойер 597-2462 597-2929 факс
        Читальный зал 597-4200
        Офис капелланов, Парески 597-2483 597-3955 факс
        Еврейский религиозный центр, Стетсон-Корт 24, 597-2483
        Мусульманская молитвенная комната, часовня Томпсона (нижний уровень) 597-2483
        Католическая часовня Ньюмана, часовня Томпсона (нижний уровень) 597-2483
        Химия, Thompson Chemistry 597-2323 597-4150 факс
        Классика (греческий и латинский языки), Холландер 597-2242 597-4222 факс
        Когнитивная наука, Бронфман 597-4594
        Маршал колледжа, Thompson Physics 597-2008
        Отношения с колледжем 597-4057
        Программа 25-го воссоединения, Фогт 597-4208 597-4039 факс
        Программа 50-го воссоединения, Фогт 597-4284 597-4039 факс
        Advancement Operations, Мирс-Вест 597-4154 597-4333 факс
        Мероприятия для выпускников, Фогт 597-4146 597-4548 факс
        Фонд выпускников 597-4153 597-4036 факс
        Связи с выпускниками, Мирс-Уэст 597-4151 597-4178 факс
        Почтовые службы выпускников / разработчиков, Мирс-Уэст 597-4369
        Девелопмент, Фогт 597-4256
        Отношения с донорами, Vogt 597-3234 597-4039 факс
        Офис по планированию подарков, Фогт 597-3538 597-4039 факс
        Grants Office, Мирс-Уэст 597-4025 597-4333 факс
        Программа крупных подарков, Фогт 597-4256 597-4548 факс
        Фонд родителей, Фогт 597-4357 597-4036 факс
        Prospect Management & Research, Мирс 597-4119 597-4178 факс
        Начало занятий и академические мероприятия, Jesup 597-2347 597-4435 факс
        Communications, Hopkins Hall 597-4277 597-4158 факс
        Sports Information, Hopkins Hall 597-4982 597-4158 факс
        Web Team, Southworth Schoolhouse
        Williams Magazines (ранее Alumni Review), Hopkins Hall 597-4278
        Компьютерные науки, Thompson Chemistry 597-3218 597-4250 факс
        Conferences & Events, Парески 597-2591 597-4748 факс
        Запросы Elm Tree House, Mt.Ферма Надежды 597-2591
        Офис контролера, Хопкинс-холл 597-4412 597-4404 факс
        Счета к оплате и ввод данных, Хопкинс-холл 597-4453
        Bursar & Cash Receipts, Hopkins Hall 597-4396
        Financial Information Systems, Hopkins Hall 597-4023
        Purchasing Cards, Hopkins Hall 597-4413
        Студенческие ссуды, Хопкинс-холл 597-4683
        Dance, 62 Центр 597-2410
        Центр Дэвиса (ранее Мультикультурный центр), Дженнесс 597-3340 597-3456 факс
        Харди Хаус 597-2129
        Jenness House 597-3344
        Райс Хаус 597-2453
        Декан колледжа, Хопкинс-холл 597-4171 597-3507 факс
        Декан факультета, Хопкинс-холл 597-4351 597-3553 факс
        Столовая, капельницы 597-2121 597-4618 факс
        ’82 Гриль, Парески 597-4585
        Кондитерская, Парески 597-4511
        Общественное питание, факультет 597-2452
        Driscoll Dining Hall, Дрисколл 597-2238
        Eco Café, Научный центр 597-2383
        Grab ‘n Go, Парески 597-4398
        Lee Snack Bar, Парески 597-3487
        Обеденный зал Mission Park, Mission Park 597-2281
        Whitmans ‘, Парески 597-2889
        Economics, Schapiro 597-2476 597-4045 факс
        Английский, Холландер 597-2114 597-4032 факс
        Сооружения, служебное здание 597-2301
        College Car Request 597-2302
        Скорая помощь вечером / в выходные дни 597-4444
        Запросы на работу производственных объектов 597-4141 факс
        Особые мероприятия 597-4020
        Кладовая 597-2143 597-4013 факс
        Клуб преподавателей, Дом факультетов / Центр выпускников 597-2451 597-4722 факс
        Бронирование 597-3089
        Fellowships Office, Hopkins Hall 597-3044 597-3507 факс
        Financial Aid, Weston Hall 597-4181 597-2999 факс
        Geosciences, Clark Hall 597-2221 597-4116 факс
        Немецко-русский, Голландер 597-2391 597-3028 факс
        Глобальные исследования, Холландер 597-2247
        Дипломная программа по истории искусств, Кларк 458-2317 факс
        Службы здравоохранения и благополучия, Thompson Ctr Health 597-2206 597-2982 факс
        Медицинское просвещение 597-3013
        Услуги интегративного благополучия (консультирование) 597-2353
        Чрезвычайные ситуации с угрозой жизни Позвоните 911
        Медицинские услуги 597-2206
        История, Холландер 597-2394 597-3673 факс
        История науки, Бронфман 597-4116 факс
        Хопкинс Форест 597-4353
        Центр Розенбурга 458-3080
        Отдел кадров, B&L Building 597-2681 597-3516 факс
        Услуги няни, корпус B&L 597-4587
        Льготы 597-4355
        Программа помощи сотрудникам 800-828-6025
        Занятость 597-2681
        Заработная плата 597-4162
        Ресурсы для супруга / партнера 597-4587
        Занятость студентов 597-4568
        Линия погоды (ICEY) 597-4239
        Humanities, Schapiro 597-2076
        Информационные технологии, Jesup 597-2094 597-4103 факс
        Пакеты для чтения курса, Dropbox Office Services 597-4090
        Центр кредитования оборудования, Додд Приложение 597-4091
        Служба поддержки преподавателей / сотрудников, [адрес электронной почты защищен] 597-4090
        Медиа-услуги и справочная служба 597-2112
        Служба поддержки студентов, [электронная почта] 597-3088
        Телекоммуникации / телефоны 597-4090
        Междисциплинарные исследования, Холландер 597-2552
        Международное образование и учеба, Хопкинс-холл 597-4262 597-3507 факс
        Инвестиционный офис, Хопкинс Холл 597-4447
        Бостонский офис 617-502-2400 617-426-5784 факс
        Еврейские исследования, Мазер 597-3539
        Справедливость и закон, Холландер 597-2102
        Latina / o Studies, Hollander 597-2242 597-4222 факс
        Исследования лидерства, Шапиро 597-2074 597-4620 факс
        Морские исследования, Бронфман 597-2297
        Математика и статистика, Bascom 597-2438 597-4061 факс
        Музыка, Бернхард 597-2127 597-3100 факс
        Concertline (записанная информация) 597-3146
        Неврология, Thompson Biology 597-4107 597-2085 факс
        Окли Центр, Окли 597-2177 597-4126 факс
        Управление институционального разнообразия и справедливости, Хопкинс-холл 597-4376 597-4015 факс
        Управление счетов студентов, Хопкинс-холл 597-4396 597-4404 факс
        Performance Studies, ’62 Центр 597-4366
        Философия, Шапиро 597-2074 597-4620 факс
        Физика, Thompson Physics 597-2482 597-4116 факс
        Планетарий / Обсерватория Хопкинса 597-3030
        Театр старой обсерватории Хопкинса 597-4828
        Бронирование 597-2188
        Политическая экономия, Шапиро 597-2327
        Политология, Шапиро 597-2168 597-4194 факс
        Офис президента, Хопкинс-холл 597-4233 597-4015 факс
        Дом президента 597-2388 597-4848 факс
        Услуги печати / почты для преподавателей / сотрудников, ’37 House 597-2022
        Программа обучения, Бронфман 597-4522 597-2085 факс
        Офис Провоста, Хопкинс Холл 597-4352 597-3553 факс
        Психология, психологические кабинеты и лаборатории 597-2441 597-2085 факс
        Недвижимость, B&L Building 597-2195 / 4238 597-5031 факс
        Ипотека для преподавателей / сотрудников 597-4238
        Профессорско-преподавательский состав Аренда жилья 597-2195
        Офис регистратора, Хопкинс Холл 597-4286 597-4010 факс
        Религия, Холландер 597-2076 597-4222 факс
        Romance Languages, Hollander 597-2391 597-3028 факс
        Планировщик помещений 597-2555
        Соответствие требованиям безопасности и охраны окружающей среды, класс ’37, дом 597-3003
        Библиотека Сойера, Сойер 597-2501 597-4106 факс
        Службы доступа 597-2501
        Приобретения / серийные номера 597-2506
        Каталогизация / Службы метаданных 597-2507
        Межбиблиотечный абонемент 597-2005 597-2478 факс
        Исследования и справочные службы 597-2515
        Стеллаж 597-4955 597-4948 факс
        Системы 597-2084
        Научная библиотека Шоу, Научный центр 597-4500 597-4600 факс
        Исследования в области науки и технологий, Бронфман 597-2239
        Научный центр, Бронфман 597-4116 факс
        Магазин электроники 597-2205
        Машинно-модельный цех 597-2230
        Безопасность 597-4444
        Специальные академические программы, Харди 597-3747 597-4530 факс
        Sports Information, Hopkins Hall 597-4982 597-4158 факс
        Студенческая жизнь, Парески 597-4747
        Планировщик помещений 597-2555
        Управление студенческими центрами 597-4191
        Организация студенческих мероприятий 597-2546
        Студенческий дом, Парески 597-2555
        Участие студентов 597-4749
        Программы проживания для старших классов 597-4625
        Студенческая почта, Парески, офис 597-2150
        Устойчивое развитие / Центр Зилха, Харпер 597-4462
        Коммутатор, Хопкинс Холл 597-3131
        Книжный магазин Уильямса 458-8071 458-0249 факс
        Театр, 62 Центр 597-2342 597-4170 факс
        Trust & Estate Administration, Sears House 597-4259
        Учебники 597-2580
        вице-президент по кампусной жизни, Хопкинс-холл 597-2044 597-3996 факс
        Вице-президент по связям с колледжем, Мирс 597-4057 597-4178 факс
        Вице-президент по финансам и администрированию, Хопкинс-холл 597-4421 597-4192 факс
        Центр визуальных ресурсов, Лоуренс 597-2015 597-3498 факс
        Детский центр Williams College, Детский центр Williams 597-4008 597-4889 факс
        Музей искусств колледжа Уильямс (WCMA), Лоуренс 597-2429 597-5000 факс
        Подготовка музея 597-2426
        Служба безопасности музея 597-2376
        Музейный магазин 597-3233
        Уильямс Интернэшнл 597-2161
        Williams Outing Club, Парески 597-2317
        Оборудование / стол для студентов 597-4784
        Проект Уильямса по экономике высшего образования, Мирс-Вест 597-2192
        Уильямс Рекорд, Парески 597-2400 597-2450 факс
        Программа Уильямса-Эксетера в Оксфорде, Оксфордский университет 011-44-1865-512345
        Программа Williams-Mystic, Mystic Seaport Museum 860-572-5359 860-572-5329 факс
        Исследования женщин, гендера и сексуальности, Schapiro 597-3143 597-4620 факс
        Написание программ, Хопкинс-холл 597-4615
        Центр экологических инициатив «Зилха», Харпер 597-4462

        странно сходящихся последовательностей | Статьи

        Классический математический анализ включает интригующее взаимодействие между конечными и бесконечными коллекциями, а также между дискретными и непрерывными структурами.Что делает взаимодействие интригующим, так это то, что на основе самых элементарных соображений возникают парадоксальные или, по крайней мере, совершенно противоречащие интуиции результаты. Какова, например, сумма 1 + 2 + 3 +… + n +…?

        Сумма ?

        Несомненно, эта последовательность чисел становится все больше и больше; а кроме этого, ни к чему не сходится.

        Не совсем так. Сумма –1/12.

        Нильс Абель назвал расходящиеся серии изобретением дьявола.Сделав выдающийся вклад в эту тему как игрушку дьявола, он знал, о чем говорит.

        Бесконечные последовательности

        Математический анализ начинается с рассмотрения бесконечных последовательностей действительных чисел и определения их предела. На данный момент я буду рассматривать реальные числа и только реальные числа. Бесконечная последовательность — это произвольная функция, область определения которой совпадает с множеством натуральных чисел ℕ = {0, 1, 2,…, n ,…} или с некоторыми из его бесконечных подмножеств, а диапазон значений которой содержится в вещественном линия R.Таким образом, an: n∈N или более явно, если менее компактно

        a0, a1, a2,…, an,….

        Термины an таких последовательностей часто генерируются по фиксированному правилу или путем применения определенного алгоритма к каждому n.

        Пределы следующие. Действительное число a является пределом последовательности an: n∈N, если для любого ε> 0 существует натуральное число n0 = n0 (ε) такое, что an-a <ε при n> n0. Если это так, последовательность сходится (или стремится) к a, когда n стремится к бесконечности: a = liman: n∈N. Легко показать, что a уникально.

        . Поскольку определение предела было дано — так случилось, что это сделал Огюстен-Луи Коши, — математики, естественно, хотели знать, существуют ли такие пределы и при каких условиях они существуют. Классический критерий — снова Коши, как это повторяется снова — утверждает, что an: n∈N сходится тогда и только тогда, когда для любого ε> 0 существует натуральное число n0 = n0 (ε) такое, что an-am < ε, если n> n0 и m> n0.

        Среди всех достоинств математического анализа никто, кроме Коши, никогда не считал эти идеи легкими, и никто, кроме Леонарда Эйлера, никогда не считал их очевидными.

        Критерий Коши отражает свойство полноты действительных чисел, из которого практически сразу вытекают необходимое и достаточное условие существования предела. Многие другие условия сходимости более удобны на практике. Если последовательность действительных чисел монотонна и ограничена, то она обязательно имеет предел.

        Если две последовательности an: n∈N и bn: n∈N сходятся к a и b соответственно, то для любых вещественных r и t последовательность ran + tbn: n∈N сходится к ra + tb.

        Предположим, что для двух последовательностей an: n∈N и bn: n∈N могут быть найдены два натуральных числа m и n, такие что am + i = bn + i всякий раз, когда i∈N. Тогда an: n∈N сходится тогда и только тогда, когда сходится bn: n∈N, и в этом случае сходится равенство

        лиман: n∈N = limbn: n∈N

        Сходимость последовательности действительных чисел зависит только от ее хвоста.

        Пример 1.

        Последовательность 1,21 / 2,31 / 3,…, n1 / n,… сходится и ее предел равен 1.

        Пример 2.

        Последовательность an: n∈N, где an = 1 + 1 / nn для n> 0, сходится к постоянной Напье, e = 2.71828….

        Пример 3.

        Последовательность an: n∈N, где an = 1 + 1/2 +… + 1 / n-ln⁡ (n) для n> 0, сходится к постоянной Эйлера, C = 0,57721….

        Если не существует предела последовательности an: n∈N, то последовательность расходится. В некотором смысле большинство реальных последовательностей расходятся.

        Пример 4.

        Любая неограниченная последовательность расходится; в частности, неограниченная последовательность bn: n∈N, где bn = 1 + 1/2 +… + 1 / n для n> 0.

        Пример 5.

        Последовательность an: n∈N, где an = 1 для всех четных n и an = -1 для всех нечетных натуральных n, расходится.

        Пример 6.

        Последовательность an: n∈N, где an = (n!) 1 / n для n> 0, не ограничена и, следовательно, расходится.

        Иногда удобно рассматривать некоторые расходящиеся последовательности, как если бы они сходились к бесконечности. Последовательность an: n∈N стремится к + ∞, если для любого r> 0 существует n0 = n0 (r) такое, что an> r при n> n0. Последовательность bn: n∈N стремится к –∞, если для любого r <0 существует n0 = n0 (r) такое, что bn n0.

        Для данной последовательности an: n∈N можно связать с ней другую последовательность sn: n∈N, где sn = Σak: k≤n обозначает сумму первых n + 1 членов an: n∈N.Обратите внимание, что исходная последовательность an: n∈N полностью определяется sn: n∈N, потому что an = sn-sn-1, точка, действительная для всех n∈N. Пара an: n∈N, sn: n∈N называется бесконечной серией, для краткости серией. С гораздо меньшим беспорядком в обозначениях математики пишут

        а0 + а1 + а2 +… + ан +….

        Ряд a0 + a1 + a2 +… + an +… сходится, если соответствующая последовательность sn: n∈N имеет конечный предел s, сумму ряда; в противном случае он расходится.

        Теперь следует критерий сходимости Коши: бесконечный ряд имеет сумму тогда и только тогда, когда для любого ε> 0 существует n0 = n0 (ε) такое, что an + 1 + an + 2 +… + an + m <ε всякий раз, когда n> n0 и m∈N.

        Из сходимости a0 + a1 + a2 +… + an +… следует, что

        lim⁡an: n∈N = 0.

        В области, где мало времени, используются более простые тесты: тест Даламбера, тест n -го корня, тест Лейбница, тест Раабе, тест Куммера, тест Бертрана, тест Гаусса, интегральный тест, тест Абеля, Проба Дирихле, проба Ермакова.

        Тестов достаточно, чтобы провести бесконечную серию экзаменов на бакалавриат.

        Пример 7.

        Ряд 1/12 + 1/22 + 1/32 +… + 1 / n2 +… сходится, поскольку все его члены 1 / n2, где n> 1, меньше 1 / (nn-1) = 1 / (n-1) -1 / n, а серия

        1 + 1/2 + 1/6 + 1/12 +… + 1 / (nn-1) +…

        сходится к 2.Здесь работает так называемый сравнительный тест. Более тонкий расчет показывает, что сумма 1/12 + 1/22 + 1/32 +… + 1 / n2 +… равна π2 / 6. Это результат, хорошо скрытый от чьей-либо интуиции.

        Как и следовало ожидать, серии бывают особых типов. Есть геометрические ряды, гармонические ряды, гипергеометрические ряды, чередующиеся ряды и многие другие. Во многих случаях серия формы

        a0x + a1x + a2x +… + тревога +…,

        находится на рассмотрении, где тревожно∈N — действительные функции действительной переменной x.Для каждого значения r x есть обычный ряд

        a0r + a1r + a2r +… + anr +….

        Я упоминал степенной ряд, ряд Тейлора, ряд Лорана, ряд Дирихле, тригонометрический ряд, ряд Фурье и постоянно популярный биномиальный ряд?

        Пример 8.

        Геометрический ряд 1 + q1 + q2 +… + qn +…, где q <1.

        Пример 9.

        Ряд Гранди: 1-1 + 1-1 +… + -1n-1 +… и, в более общем смысле, для любого строго положительного целого k, ряд 1-k1 + k2-k3 +… + -1n-1kn-1 + ….

        Пример 10.

        Серия 0! -1! +2! -3! +… + -1nn! +….

        Пример 11.

        Серия Дирихле:

        fx = a1 / 1x + a2 / 2x + a3 / 3x +… + an / nx +…,

        , где x∈R, а an: n∈N — последовательность действительных чисел. В частности, обобщенный гармонический ряд, который является частью дзета-функции Римана, появляется почти везде:

        ζx = 1 / 1x + 1 / 2x + 1 / 3x +… + 1 / nx +…,

        и обобщенный ряд переменных гармоник, который является частью эта-функции Дирихле

        ηx = 1 / 1x-1 / 2x + 1 / 3x-… + (- 1) n-1 / nx +…,

        Таким образом получается

        .Ряд 1 / 1x + 1 / 2x + 1 / 3x +… + 1 / nx +… сходится для всех действительных x> 1, а ряд 1 / 1x-1 / 2x + 1 / 3x-… + (- 1) n- 1 / nx +… сходится для всех действительных x> 0.

        Пример 12.

        Ряд a0 + a1 + a2 +… + an +… называется абсолютно сходящимся, если сходится ряд a0 + a1 + a2 +… + an +…. Всякий абсолютно сходящийся ряд сходится.

        Увы, обратное неверно. Переменная гармоническая серия

        1-1 / 2 + 1 / 3-1 / 4 +… + (- 1) n-1 (1 / n) +…,

        сходится, и его сумма равна ln (2), но этот же ряд не сходится абсолютно, как может указывать пример 4.

        Сходящийся ряд a0 + a1 + a2 +… + an +… условно сходится, если связанный с ним ряд a0 + a1 + a2 +… + an +… расходится.

        Если ряд сходится условно, то для любого действительного r существует такая перестановка его членов, что новый ряд сходится к r. Это классическая теорема Римана, допускающая нетривиальное обобщение на серию векторов в конечномерном евклидовом пространстве.

        Странно расходящиеся серии

        Странно расходящиеся ряды, такие как 1 + 2 + 3 + 4 +…, просты, поскольку они расходятся; они странны, поскольку это не так.Это утверждение, казалось бы, идет на цыпочках до самого края парадокса. Тем не менее Эйлер систематически имел дело со странно расходящимися рядами; он смог успешно ими манипулировать. После множества хитроумных манипуляций Эйлер решил заявить, что любой расходящийся ряд должен иметь определенную сумму. Позицию Эйлера критиковали придирчивый Жан Д’Аламбер, столь же привередливый Абель и другие математики, которым все безразлично, что Эйлер мог быть прав, а они — нет.Лишь намного позже какой-то рациональный фрагмент идеи Эйлера превратился в богатую теорию суммируемости расходящихся рядов.

        Многие расходящиеся серии; и столько сумм; много сумм и так много методов суммирования: метод суммирования Абеля, метод суммирования Бореля, метод суммирования Чезаро, метод суммирования Эйлера, метод суммирования Холдера, метод суммирования Куммера, метод суммирования Ламберта, метод суммирования Линделофа, метод суммирования Пуассона, метод суммирования Вороного и многие другие.

        Самый простой метод суммирования назван в честь Эрнесто Чезаро. Если дан ряд a0 + a1 +… + an +…, то иногда может случиться — не всегда и не во всех случаях — что соответствующая последовательность средних арифметических его частичных сумм

        s0, s0 + s1 / 2, (s0 + s1 + s2) / 3,…,
        (s0 + s1 +… + sn) / (n + 1),…,

        сходится к определенному действительному числу. Это его сумма Чезаро.

        Понятие суммы Чезаро совместимо с обычным понятием суммы сходящегося ряда.Действительно, согласно одной из классических теорем Коши, если последовательность действительных чисел имеет предел, то последовательность средних арифметических чисел этих чисел также сходится и имеет тот же предел. Простые контрпримеры показывают, что обратное неверно.

        Следует также отметить, что метод суммирования Чезаро вдохновил тауберовы результаты в общей теории рядов. Эти результаты позволяют математикам установить сходимость заданного ряда a0 + a1 +… + an +…, если a0 + a1 +… + an +… суммируется некоторым методом и удовлетворяет некоторым дополнительным ограничениям.

        Побывав в сарае с инструментами дьявола, Авель предложил метод, ныне известный как метод суммирования Абеля или, иногда, метод суммирования Абеля-Пуассона. Дьявол, как известно, работает без дела.

        Предположим, что дан ряд a0 + a1 +… + an +…, и для переменной x, находящейся в открытом интервале (0,1), рассмотрим соответствующий степенной ряд

        a0 + a1x1 + a2x2 +… + тревога +….

        Может случиться так, что этот ряд сходится для каждого r в (0,1) и, следовательно, определяет конкретную функцию f (x) на интервале.Может случиться так, что существует предел f (x), когда x стремится к 1. Такова сумма Абеля ряда. Метод Чезаро совместим с обычным понятием суммы сходящегося ряда. Таков метод Авеля. Если ряд a0 + a1 +… + an +… сходится к s, то в силу классической теоремы Абеля сумма Абеля a0 + a1 +… + an +… также равна s. Обратное неверно.

        Чеки дьявола никогда не бывают пустыми.

        Пример 13.

        Для любого строго положительного целого k из метода Абеля следует, что расходящийся ряд

        1-k1 + k2-k3 +… + -1n-1kn-1 +…,

        суммирует до 1 / (k + 1), и, в частности, серия Гранди

        1-1 + 1-… + -1н-1 +…

        сумм до 1/2.Ряд Гранди также суммируется методом Чезаро, и его сумма по Чезаро снова равна 1/2. Это не просто счастливое совпадение. Метод Абеля значительно сильнее, чем метод Чезаро. С другой стороны, для натурального числа k, строго большего 1, ряд

        1-k1 + k2-k3 +… + -1n-1kn-1 +…,

        не суммируемый по Чезаро.

        Есть много других примеров рядов, суммируемых по методу Абеля, а не по методу Чезаро.

        Пример 14.

        Рассмотрим ряд 1-2 + 3-4 +… + -1n-1n +…, который тривиально расходится и не суммируется по Чезаро.Метод Абеля работает, но метод Чезаро не работает. Действительно, если 0

        Метод Абеля очень полезен при работе с продуктами бесконечного ряда. Ниже представлена ​​одна формула, но две серии:

        a0 + a1 +… + an +…,
        b0 + b1 +… + bn +….

        Их произведение c0 + c1 +… + cn +… определяется равенством

        cn = a0bn + a1bn-1 + a2bn-2 +… + anb0 (n∈N)

        Произведение двух условно сходящихся рядов, как правило, не сходится.Но если оба ряда a0 + a1 +… + an +… и b0 + b1 +… + bn +… суммируемы по Абелю, то их произведение c0 + c1 +… + cn +… также суммируемо по Абелю и равно произведению сумм Абеля a0 + a1 +… + an +… и b0 + b1 +… + bn +….

        Это удивительно элегантный результат и, возможно, самый простой результат, доступный интуиции, который предполагает, что методы, изобретенные по определению, имеют богатую внутреннюю структуру.

        Пример 15.

        Используя тот факт, что сумма произведений является произведением сумм — выше, чуть выше — сразу получаем равенство

        1-1 + 1-1 +… 1-1 + 1-1 +…
        = 1-2 + 3-4 +….

        Пример 16.

        Эмиль Борель создал или открыл несколько важных методов суммирования. Если

        а0 + а1 +… + ан +…,

        — это серия, уходящая в неизвестность, затем функция

        fx = e-x (Σsnxn / n!: N∈N),

        имеет смысл — точнее говоря, совершенный — применительно к набору действительных чисел x, для которого эта функция хорошо определена. Может случиться так, что f (x) имеет конечный предел y, когда x стремится к + ∞. В таком случае y называется суммой ряда Бореля. Если ряд a0 + a1 +… + an +… сходится, то он также суммируем по Борелю и его обычная сумма совпадает с его суммой по Борелю.

        Доказательство несложно.

        Далее используется тот же Борелевский, но другой метод: суммирование борелевских интегралов.

        Пример 17.

        Ряд 0! -1! +2! -3! +… + (- 1) nn! +… Имеет сумму Бореля

        ∫0∞e-t1 + tdt.

        Пример 18.

        Рассмотрим частичную дзета-функцию Римана ζ (x) и частичную эту функцию Дирихле η (x) для всех действительных чисел x, строго превышающих 1. Сходимость

        1 / 1x + 1 / 2x + 1 / 3x +… + 1 / nx +…,
        1 / 1x-1 / 2x + 1 / 3x-… + -1n-1 / nx +…,

        сразу подразумевает:

        ηx = (1-21-x) ζ (x).

        Еще лучше, можно рассмотреть 1 / 1x-1 / 2x + 1 / 3x-… + -1n-1 / nx +… для комплексных чисел z. Заменяя x на z, получаем 1 / 1z-1 / 2z + 1 / 3z-… + -1n-1 / nz +…, которое сходится к комплексному значению, когда действительная часть z строго больше 1. Это η ( x) допускает аналитическое продолжение на всю комплексную плоскость. C является доказуемым фактом жизни. Кроме того, есть

        ηz = (1-21-z) ζ (z),

        где ζ (z) — аналитическая функция на открытом множестве C \ {1}, продолжающая ζ (x). Помня, что η-1 = 1/4, можно, наконец, обнаружить, что вопреки всем ожиданиям,

        1 + 2 + 3 +… + n +… = ζ-1 = -1 / 12.

        Это выдающийся результат в математике, но это также выдающийся результат в математике, имеющий нетривиальные приложения в современной теоретической физике.

        Видел ли вдохновленный Эйлер, слепой со средних лет и так много видевший , это ?

        Теория бесконечного суммирования

        Предположим, что последовательность функций gn: n∈N задана на некотором подмножестве D в R такая, что существует точка накопления d в D. Предположим также, что для каждого n∈N существует предел gnt как t∈ D стремится к d, и что этот предел равен 1.

        Рассмотрим ряд a0 + a1 +… + an +… и пусть функциональный ряд

        gt = a0g0t + a1g1t +… + angnt +… (t∈D)

        будет его естественным партнером.

        Может случиться, что функция g (t) корректно определена в некоторой окрестности точки d и, кроме того, существует предел s = sa0 + a1 +… + an +… для g (t) при t∈D имеет тенденцию к d. Ряд S = a0 + a1 +… + an +… суммируем, и его обобщенная сумма равна s.

        Пример 19.

        Предположим, что последовательность функций gn: n∈N удовлетворяет этим двум условиям:

        1. все функции из gn: n∈N равномерно ограничены; и
        2. для каждого t∈D последовательность gnt: n∈N монотонна.

        Если S сходится к s, то сумма s равна s.

        Многие классические методы суммирования соответствуют соответствующему выбору последовательности функций, удовлетворяющих 1) и 2). Для метода суммирования Чезаро функции gnn∈N могут быть определены на N следующим образом:

        gnm = m + 1-n / m + 1, если n≤m,
        и gnm = 0, если n> m

        Для метода суммирования Абеля функции gnn∈N могут быть определены на открытом интервале (0,1)

        gnt = tn 0

        Взяв положительный луч (0, + ∞) R в качестве D и определив функции gnn∈N формулой

        gnt = 1-∑tk / k!: K 0

        получается определенная версия метода суммирования Бореля.

        Есть и другие общие схемы. Не все идет гладко. Две разные схемы могут оказаться несовместимыми в том смысле, что для некоторых конкретных серий они дают две различные обобщенные суммы.

        Пример 20.

        Довольно часто схема суммирования формулируется в терминах вещественной матрицы T со счетным числом строк и столбцов. Методы T-суммирования преобразуют одну бесконечную последовательность в другую в ожидании или надежде, что это преобразование даст последовательность, которая будет лучше себя вести в плане сходимости.Знаменитая теорема Сильвермана – Теплица устанавливает необходимые и достаточные условия, при которых класс всех сходящихся последовательностей переводится посредством T в себя.

        Среди множества интересных и важных утверждений о суммируемости матриц я хотел бы упомянуть один результат. Для данной матрицы T и ряда a0 + a1 +… + an +…, рассмотрим все те перестановки членов a0 + a1 +… + an +…, которые дают T-суммируемый ряд, и обозначим символом S (a0 + a1 +… + an +…, T) множество всех T-сумм, полученных таким образом.Оказывается, что, изменяя a0 + a1 +… + an +… и T всеми возможными способами, семейство всех множеств S (a0 + a1 +… + an +…, T) дает семейство всех аналитических, или суслинских, подмножеств R Это семейство намного шире, чем семейство всех борелевских подмножеств R.

        .

        . Различные схемы суммирования естественным образом приводят к большому семейству точечных множеств в R и связаны с более или менее тонкими фактами классической описательной теории множеств. Подобные связи затрагивают даже проблему измеримости по Лебегу всех подмножеств R.

        С более общей точки зрения, любой метод суммирования может быть описан как функционал f, определенный на некотором векторном подпространстве L = dom (f) пространства RN всех бесконечных вещественных последовательностей. Элементы L имеют вид sn: n∈N, где sn (n∈N) — частичные конечные суммы ряда. Как правило, методы суммирования удовлетворяют условиям как линейности, так и регулярности. Линейность означает, что f — линейный функционал на L; регулярность, что f расширяет стандартный линейный функциональный предел, определенный на пространстве c всех сходящихся последовательностей в RN.

        Иногда накладывается третье условие, некий аналог трансляционной инвариантности. Если s является суммой ряда a0 + a1 +… + an +… и r является суммой a1 +… + an +…, то s = a0 + r.

        Пример 21.

        Ни один метод суммирования, удовлетворяющий условию трансляции, не может быть применен к ряду 1 + 1 + 1 +… + 1 +…. Доказательство очевидно: s = 1 + 1 + 1 + 1 +… + 1 +… = 1 + s, откуда следует, что 0 = 1.

        Теперь, если последовательность tn: n∈N не принадлежит L, а r — произвольное действительное число, функционал f может быть расширен до линейного функционала g, определенного на большем векторном подпространстве K в RN, так что

        tn: n∈N∈K, gtn: n∈N = r.

        Это утверждение допускает конструктивное доказательство, не опирающееся на аксиому выбора.

        Эйлер подтвержден

        Прекрасная интуиция Эйлера теперь подтверждена:

        Для любого конкретного расходящегося ряда существует линейный и регулярный метод суммирования, который присваивает ряду действительное число в качестве его суммы.

        Сразу следует резервирование. Не существует линейного метода суммирования , удовлетворяющего условию сдвига, который присваивает конечную сумму на 1 + 2 + 3 +… + n +….

        Предположим, что

        1 + 2 + 3 +… + n +… = s,

        , где s — действительное число. Тогда:

        с-1 = 2 + 3 +… + n +…, и

        -1 = s-1-s = 2-1 + 3-2 +… + n + 1-n +…
        = 1 + 1 + 1 +… + 1 +…,

        что невозможно.

        Если кто-то желает приписать определенные обобщенные суммы сразу всем расходящимся рядам, то он должен обратиться к сильным формам аксиомы выбора. Стандартное применение леммы Цорна, например, показывает, что существует линейный регулярный метод суммирования, область определения которого составляет всю RN.На практике такой результат бесполезен из-за крайне неконструктивного характера. С другой стороны, из фундаментального результата Роберта Соловея следует, что при счетных формах выбранной аксиомы логически непротиворечиво предполагать, что область применения любого линейного регулярного метода суммирования является подпространством первой категории в RN. Поскольку RN — польское топологическое пространство, RN само по себе является второй категорией. Следовательно, область применения любой конструктивно определенной линейной и регулярной схемы суммирования является очень малой частью RN.В рамках того же слабого фрагмента теории множеств можно предположить, что класс всех линейных регулярных методов суммирования, снабженный своим естественным частичным упорядочением, не имеет максимальных элементов.

        Используя бесчисленные формы избранной аксиомы, математики смогли построить несколько замечательных линейных функционалов на различных векторных подпространствах RN. Среди них наиболее известен функциональный предел Банаха, или Lim, определенный на пространстве всех ограниченных вещественных последовательностей, Lim является линейным, расширяет стандартный функциональный предел и инвариантен относительно сдвигов.Но существование таких функционалов всегда подразумевает существование в конечном итоге патологических подмножеств R, а именно множеств, неизмеримых в смысле Лебега, или множеств, не обладающих так называемым свойством Бэра.

        Пример 22.

        Рассмотрим следующие функции серии FS:

        cos (t) ⁡ + cos⁡2t + cos⁡4t +… + cos⁡2nt +…,

        , где переменная t принимает значения, превышающие R. Предположим, что для каждого t из измеримого по Лебегу подмножества R со строго положительной мерой FS суммирует f (t) с помощью некоторой линейной регулярной схемы суммирования, удовлетворяющей условию сдвига.Отсюда следует, что f неизмеримо по Лебегу. Такова теорема Колмогорова.

        Глубокий результат Соловея показывает, что нет никакой надежды на эффективное построение измеримого по Лебегу подмножества R или, что то же самое, не измеримой по Лебегу функции, действующей из R в R.

        Комплексный анализ

        ← Комплексный анализ →


        Сходимость последовательностей

        Бесконечная последовательность комплексных чисел $ \ left \ {z_1, z_2, z_3 \ ldots \ right \} $ имеет предел $ z $, если для каждого положительное число $ \ varepsilon $, существует натуральное число $ n_0 $ такое, что \ begin {eqnarray} \ label {seq} \ left | z_n-z \ right | <\ varepsilon \ quad \ text {when} \ quad n> n_0.\ end {eqnarray}

        Геометрически это означает, что при достаточно больших значениях $ n $ точки $ z_n $ лежат в любом заданном Окрестность $ \ varepsilon $ точки $ z $ (рисунок 1). Поскольку мы можем выбирать размер $ \ varepsilon $ сколь угодно маленьким, он Отсюда следует, что точки $ z_n $ становятся сколь угодно близкими к $ z $ по мере увеличения их индексов. Обратите внимание, что необходимое значение $ n_0 $ будет, как правило, зависеть от значения $ \ varepsilon $.

        Рисунок 1: Геометрическая интерпретация.{\ infty} $ может иметь не более одного ограничения. То есть предел $ z $ уникален если он существует. Когда этот предел существует, говорят, что последовательность сходится к $ z $; и мы пишем \ begin {eqnarray *} \ lim_ {n \ rightarrow \ infty} z_n = z \ end {eqnarray *} Если последовательность не имеет предела, она расходится на .

        Теорема 1: Предположим, что $ z_n = x_n + iy_n $ ($ n = 1,2,3, \ ldots $) и $ z = x + iy $. потом \ begin {eqnarray} \ label {teoseq01} \ lim_ {n \ rightarrow \ infty} z_n = z \ end {eqnarray} если и только если \ begin {eqnarray} \ label {teoseq02} \ lim_ {n \ rightarrow \ infty} x_n = x \ quad \ text {и} \ quad \ lim_ {n \ rightarrow \ infty} y_n = y.\ end {eqnarray}

        Проба

        Чтобы доказать эту теорему, сначала предположим, что выполнены условия (\ ref {teoseq02}). То есть существуют, для каждого $ \ varepsilon> 0 $ положительные целые числа $ n_1 $ и $ n_2 $ такие, что \ [ | x_n-x | <\ frac {\ varepsilon} {2} \ quad \ text {when} \ quad n> n_1 \] а также \ [ | y_n-y | <\ frac {\ varepsilon} {2} \ quad \ text {when} \ quad n> n_2. \] Следовательно, если $ n_0 $ больше двух целых чисел $ n_1 $ и $ n_2 $, \ [ | x_n-x | <\ frac {\ varepsilon} {2} \ quad \ text {и} \ quad | y_n-y | <\ frac {\ varepsilon} {2} \ quad \ text {when} \ quad n> n_0.\] С \ [ | (x_n + iy_n) — (x + iy) | = | (x_n-x) + (y_n-y) | \ leq | x_n-x | + | y_n-y |, \] тогда \ [ | z_n-z | <\ frac {\ varepsilon} {2} + \ frac {\ varepsilon} {2} \ quad \ text {when} \ quad n> n_0. {\ infty} z_n = S.{N} y_n. \] Теперь утверждение (\ ref {teo01}) истинно тогда и только тогда, когда \ begin {eqnarray} \ label {teo04} \ lim_ {N \ rightarrow \ infty} S_N = S; \ end {eqnarray} и ввиду соотношения (\ ref {teo03}) и теоремы 1 о последовательностях предел (\ ref {teo04}) выполняется, если и только если \ begin {eqnarray} \ label {teo05} \ lim_ {N \ rightarrow \ infty} X_N = X \ quad \ text {и} \ quad \ lim_ {N \ rightarrow \ infty} Y_N = Y. \ end {eqnarray} Пределы (\ ref {teo05}), следовательно, подразумевают оператор (\ ref {teo01}), и наоборот.Поскольку $ X_N = X $ и $ Y_N = Y $ — частичные суммы ряда (\ ref {teo02}), теорема доказана. $ \ blacksquare $

        Эта теорема может быть полезна для демонстрации того, что ряд знакомых свойств рядов в исчислении переносятся на ряды, члены которых являются комплексными числами.

        Свойство 1: Если серия комплексных чисел сходится, $ n $ -й член сходится к нулю как $ n $ стремится к бесконечности.